You are on page 1of 615
Oy, TT (74, ANE, ba Zi ( CHEMISTRY ae / For \ EEE ; » Published by : GOLDEN BELLS 94-B, Hindi Park, Daryaganj, New Delhi-1 10002. Phones ; {01*2326 01 90 * |o112328 71 26 EMAIL ; col WEBSITE : laxmi@hotmail.com wwwlaxmipublicationscom Branches Offices : Bangalore Chennai Cochin Guwahati Hyderabad : Jalandhar Kolkata Lucknow Mumbai Ranchi :Pan Bazar, Rani Bari, Guwahat 129/, Ullrd Main Road, IX Cross Chamrajpet, Bangalore (Phone : 080-2661 1961-62) £26, Damodaran Street, ! Nagar, Chennai (Phone : 044-2434 47 26) 43/1394 D, St. Benedict's Road, Ernakulam North, Cochin (Phone : 0484-239 70 04) ‘(Phones : 0261-254 38 69, 251 98 81) 4-2-453, Ist Floor, Ramkote, Hyderabad (Phone : 040-2475 02 47) ‘Adda Tanda Chowk ND-366, Jalandhar City (Phone : 0181-222 12 72) :R.DB. Chambers (Formerly Lotus Cinems) 106/A, 1st Floor, S.N. Banerjee Rogd, Kolkata, Ph. : 033-22275247 Pax. : 22273773 E-mail : nailcal@vsnl.net #18, Madan Mohan Malviya Marg, Lucknow (Phone : 0522-220 95 78) E-mail : newsgelko@sifycom Fax : 220 40 98 : 128A, Block 3, First Floor, Noorani Building, Opp. Old Mahim PO. Ld. Road, Mumbai (Phone : 022-24 46 39 98), 24 46 16 32 Fax : 91-22-24 46 16 32 E-mail : nevagemum@tstanova.com Radha Gobind Street, Tharpagna, Ranchi (Phone : 0651-230 77 64) GBC—5021—295-CHEMISTRY FOR AIEEE All Rights Reserved with the Publishers 5th Edition Edited by : Sh. Rajesh Aggarwal Kota Price : Rs. 295.00 only C—1152410808 Lover Typesetted at : Monu Printographies, Delhi-03 Printed at : MEHRA PREBS, Delhi Contents Units SOLVED PAPERS — AIEEE 2005, 2004, Pages SOME BASIC CONCEPTS (Atoms, Molecules and Chemical Arithmetic) Important Terms, Facts and Formulae a5 Question Bank : Level I G11 Level IL 15 Answers, Hints/Solutions 122 STATES OF MATTER (Gaseous State) Important Terms, Facts and Formulae 29-27 Question Bank : Level! 2en24 Level IL 440 Answers, Hints/Solutions a7 STATES OF MATTER (Solid State) ‘Important Terms, Facts and Formulae 4851 Question Bank : Level I 52-86 Level IL 56—60 Answers, Hints/Solutions ama ATOMICSTRUCTURE Important Terms, Facts and Formulae e486 Question Bank: Level I sr Level IL Daa Answers, HintwSolutions 782 Ei sams Important Terms, Facts and Formulae 8386 Question Bank: Level I 792 LeveL I 99 Answers Hinte/Solutions 29-108 Unite Ey crea, ENERGETICS AND THERMODYNAMICS Important Terms, Facts and Formulae 107—111 Question Bank: Level I 112-118 pale voce host Hata joe Important ‘Terms, Facts: and Formulae 133~136 Question Bank: Level I 137-143 a ae Answers, Hints/Solutions 151—157 Ee REACTIONS ANDELECTROCHEMISTRY Tern Tr Ps nt oma Te Question Bank; Level [ 162—167 tou ae snr Hau a El 2225 ceca encrons wo cveea wens Important Terms, Facts and Formulae 180—162 gontabeet bot pear tae eos os ows : eres Important Terms, Facts and Formulae 200—202 Question Bank: Level 203207 oe moa Answers, Hints/Solutions: ‘211-213 es Important Terms, Facts and Formulae ‘214216 nub col a par ae Answers, Hints/Solutions (226-229 er Mcn FONDA MOLETILAR TROSTORE Tapert Ti os ad Pome a onto be teal =a fae ao ices Answers, Hints/Solutions Unite Pages CHEMISTRY OF NON-METALS-1 (Hydrogen, Oxygen, Nitrogen and thelr Compounds) Important Terms, Facts and Formulae (255-268 Question Bank: — Level I 269-277 Level It (277-281 GiEASTRY OF WOWETALE-# uo Sion Phsphen Sub oda ere) Tes Pa Ral ae Question Bank: — Level I 298-305 Level IT 305—309 a: CHEMISTRY OF LIGHTER METALS Il. ELEMENTS, THEIR OCCURRENCE AND EXTRACTION Top ea Fl ma mae Question Bank: Level J 318321 toi sat vcr, an ae a CHEMISTRY OF HEAVIER METALS Ta ou as oa oo Question Bank: — Level I 334-337 Level IT 337-341 es xe El scstar ornepresonarve zn Important Terms, Facts and Formulae 345363 Question Bank: — Level I 364-376 teat a ee asiaer i TRANSITION METALS INDUCING LANTHANIDES Tmportant Terms, Facts and Formulae (387-392 Question Bank: Level I 393397 teat oe Air te a i CO-ORDINATION CHEMISTRY AND ORGANO METALLICS- Important Terms, Facts and Formulae 405—409 Question Bank: — Level I 410—415 coat ten NUCLEAR CHEMISTRY Important Terms, Fae's ard Formulae | 425427 Question Bank: Level | 423-433, Level it 499497 Answers, Hinte/Solutions 497-442 PURIFICATION AND CHARACTERISATION OF ORGANIC COMPOUNDS Important Terms, Faets and Formulae 440447 Question Bank : Level 443452 Level It 459457 Answers, HinteSolutions 487-462 PRR] -UNDAMENTAL CONCEPTS OF ORGANIC CHEMISTRY | Important Terms, Facts and Formulae 463471 Question Bank: Level J an—a05, Level It 475—488 Answers, Hints/Solutions 483—484.D 1 EE] souensu AND STEREOCHEMISTRY Important Terms, Facts and Formulae 485492 Question Bank 493497 ‘Answers, Hints/Sslutions 497600 BPE CHEMISTRY OF HYDROCARBONS Important Terms, Facts and Formulae 501—606 Question Bank : Level I 50711 Level IE 511-615 Anowers, Hinte/Solut 51552 BEM ORGANIC COMPOUNDS CONTAINING HALOGENS (Alkyl Halides and Aryl Halides) Important Terms, Foets and Formulae 522825 Question Bank: Level 1 526-530 Level It 530634 Answers, Hinta/Sslutions 596-528 IC COMPOUNDS CONTAINING OXYGEN (Alcohols and Phenol ORGANIC COMPOUN: (Alcohols and Phenols) - Important Terms, Faets and Fermulae 530544 Question Bank: Level I 545—548 Level It 549-553 564657 Answers, Hints/Sclutions Units ee PBB tes cance correc 6 can om Important Terms, Facts and Formulae 558566 cee ot oe tot cna Answers, Hints/Solutions 577-483 Bob or sounomnos conte ni eaten Ai nia itn Tp rs a oi sa Question Bank: — Level I s94—098 Level It 599603 a conn Sncae conan ranma TORS (Snes ens coy Rad hn ae) rt ees Pl re cae Question Bonk: — Level I 623—628 tui aaa es om 1. SYNTHETIC AND NATURAL POLYMERS «Ss a Importent Terms, Facts and Formulae 642—648 Question Bank: — Level I 649653 ict one Pal 26 woizcutes mo noroacas proces Inportant Terms, Frets and Formulae Question Bank: Level J Level IT Answers, Hinte'Solutions Amportont Terms, Frets and Formulae Question Bank: § Level L Love! IE Answers, Hints/Solutions 676-680 631683 oa4—ea6 686—688 Units Pages PRACTICE PAPERS. Practice Paper 1 crs Practice Paper 2 c-6-C-10 Practice Paper 3 110-15 AIEEE (AET) 2005—Solved 1. 2. 4, 9. 10. The oxidation state of Cr in [Cr(NH,),Cly]' is (@o wea (+2 (+3 Which one of the following types of drugs reduces fever ? (a) Tranquiliser (®) Antibiowe (6) Antipyretie (d) Analgesic. Which of the following oxides is amphoteric in character ? (a) 8n0; (@) Sid, (©) CO, (d)Ca0. Which one of the following species is diamagnetic in nature ? iy ony (Hy (a) Hey’ If cis the degree of dissociation of Na,SO,, the vant Hoff’s factor (#) used for calculating the molecular mass is (a) ~20 (b) 1420 =a isa. Which of the following is a polyamide ? (a) Bakelite (b) Terylene (e) Nylon-66 (a) Teton. Due to the presence of an unpaired electron, free radicals are : ta) Cations (0) Anions (©) Chemically inactive (d) Chemically reactive. For a spontaneous reaction the AG, equilibrium constant (K) and E°,. will be respectively @)-ve ch ve (a) -ve,>1,-ve (©) + ¥0,> 1, ve Hydrogen bomb is based on the principle of (a) Artificial radioactivity (b) Nuclear fusion (e) Natural radioactivity (d)—ve, > 1, +8. (d) Nuclear fission. An ionic compound has a unit cell consisting of A ions at the corners of a cube and B ions on the contres of the faces of the cube. The empirical for- mula for this compound would be n. 12, 13. 14, 15. @AB AB, AB (a) AB, The highest electrical conductivity of the follow- ing aqueous solutions is of (a) 0.1 M diftuoroacetie acid (6) 0.1 M fluoroacetic acid (€} 0.1 M chloroacotic acid (@) 0.1M acetic aci. Lattice energy of an ionic compound depends upon (a) Charge on the ion and size of the ion (®) Packing of ions only {c) Size of the ion only (d) Charge on the ion only. Consider an endothermic reaction X + Y with the activation energies E, and Ey for the backward and forward reactions, respectively. In general (a) there is no definite relation between E, and E, (0) B= By (E,> E, (d) By < Ep Aluminium oxide may be electrolysed at 1000°C to furnish aluminium metal (At. Mass = 27 amu; 1 Faraday = 96,500 Coulombs). The cathode reaction is AI + Se" Al? ‘To prepare 5.12 kg of aluminium metal by this mathod would require (a) 5.49 x 10' C of electricity (b) 5.49 x 10° C of electricity (c) 1.83 x 10" C of electricity (a) 5.49 x 107 C of electricity. ‘The volume of a colloidal particle. Ve as compared tothe volume of a solute particle in a true solution Vg, could be Ne yo-8 ©) yy 10 Ne @ ye = 2005/2 16. 17. 18. 19. a1. Consider the reaction : Ny + 3H, > 2NH, carried out at constant temperature and pressure. If AH and AU are the enthalpy and internal energy changes for the reaction, which of the following ‘expressions is true? (@) AH > AU (b) AH < AU (©) aH = AU (@) aH =0. ‘The solubility product of a salt having general formula MX, in water is : 4 x 10"? The concentration of M* ions in the aqueous solution of the salt is (a) 4.0 x 10° M, (6) 1.6 x10" M © 10x 104M (a) 20 x 10M. Benzene and toluene form nearly ideal solutions. ‘At 20°C, the vapour pressure of benzene is 75 torr and that of toluene is 22 torr. The partial vapour pressure of bonzene at 20°C for a solution contain- ing 78 g of benzene and 46 g of toluene in torr is (@) 535 (6) 37.5 (© 25 (a) 50. Which one of the following statements is NOT true about the effect of an increase in temperature on the distribution of molecular speeds in a gas ? (e) The area under the distribution curve remains the same as under the lower temperature ) The distribution becomes broader (©) The fraction of the molecules with the most prob- able npeed increases (d) The most probable speed increases. For the reaction 2NOdg) = 3NO@) + OFg), (K, = 1.8 x 10-8 at 184°C) (R = 0.0831 kJmol.K)) nes K, and K, are compared at 184°C it is found rat (¢) Whether K, is greater than, less than or equal to K, depends upon the total gas pressure OK=K (6) Ky is less than K, (2) K, is greater than K. ‘The exothermic formation of CIF, is represented by the equation : Clg) + 3F,fg) = 21g) ArH = ~ 329 kad Which of the following will increase the quantity of CIF, in an equilbrium mixture of Cl, F, and ar,? 2. (a) Adding F, (0) Increasing the (©) Removing Cl, (a) 3.98 10% (©) 3.88 10° (@) 3.68 x 10-6 | 3.98 x 108. two different reactants can (@ snimolecalar ‘Two solutions of 4 substance (non-clectrolyte) are mixed in the follbwing manner. 480 ml of 1.5 M first solution + 5$0 mL of 1.2 M second solution. ‘What is the molafity of the final mixture ? (a) 2.70 M 134M (e) 1.50 M | @)1.20M. During the of electrolytic refining of cop- per, some metals prosont as impurity settle as‘an- ‘ode mud’. These (a) Fe and Ni (6) Agand Au (©) Pb and Za (@) Sn and Ag. “Blectrolyte KNO, | HCI | NaOAe | NeCi a(S em? mot) vaso | 262] mo | 155 Calculate Ayo using appropriate molar con- ductances of the diecttolytes listed above at infinite dilution in H,O dt 25°C (@) 2175 (6) 390.7 (©) 552.7 (5172, If we consider that 1/6, in place of 1/12, mass of carhon atom is tafcen to be the relative atomic mass unit, the mass of one mole of a substance will (a) be a function of the molecular mass of the substance (6) remain, " ld mn atom, which of the following by the three quantum members 1e energy in the absence of mag- fields ? =0 =o AIBEE—2005 PAPER 2005/3 a1. 32. ()n=3, (a) (4) and (6) (6) (@)and (4) (c) (2) and (3) (di and 2), Based on lattice energy and other considerations, which one of the following alkali metal chlorides is expected to have the highest melting point ? (a) RCL (o) KCL, (©) NaCl (a) Licl A schematic plot of In K,, versus inverse of tem- perature for a reaction is shown below 60, 1 20 7 : 15x10 FE) 20x10 ‘The reaction must be (a) highly spontaneous at ordinary tomperature (@) one with negligible enthalpy change (c) endothermic (@) oxothermic. Heating mixture of Cu,O and Cu,S will give (a) Cu,80, () CuO + Cus () Cus so, (a) Cus 80, ‘The molecular shapes of SF,, CF, and XeP, are (a) different with 1, 0 and 2 lone pairs of electrons on the central atom, respectively (®) different with 0, 1 and 2 lone pairs of electrons on. the central atom, respectively (c) the same with 1, 1 and 1 lone pair of electrons on the central atoms, respectively (d) the same with 2,0 and 1 lone pairs of electrons on the central atom respectively. ‘The disperse phase in colloidal iron (III) hydroxide and colloidal gold is positively and negatively charged, respectively. Which of the following statements is NOT correct ? (a) Coagulation in both sols can be brought about by electrophoresis (®) Mixing the sols has no effect {c) Sodium sulphate solution causes coagulation in both, sols (d) Magnesium chloride solution coagulates, the gold sol ‘more readily than the iron (IID hydroxide eo. 34. 85. 87. 41. The number of hydrogen atom(s) attached to phos- phorus atom in hypophosphorous acid is (@) three @) one ©) two (a) zero. What ie the conjugate base of OH”? (a) OO (©) H,0 (a) 0,. Heating an aqueous solution of aluminium chloride to dryness will give (a) ANOHICL, (6) ALO, ALC, (@) AICI, ‘The correct order of the thermal stability of hydrogen halides (HX) is (@) HI > HOI < HF > HBr (®) HOI < HF > HBr < HI (©) HP > HCl > HBr > HI (@) Hl > HBr > HO > HF. Calomel (Hg,Cl.) on reaction with ammonium hydroxide gives (a) Hg () Hg,0 (© Ni-Hg-Hg~ Cl (@) HgNH,Cl. ‘The number and type of bonds between two car- bon atoms in calcium carbide are (a) Two sigma, two pi (8) Two sigma, one pi (©) One sigma, two pi (@) Ore sigma, one pi. ‘The oxidation state of chromium in the final prod- uct formed by the reaction between KI and acidi- fied potassium. dichromate solution is @)+3 (by +2 ore (a+. In silicon dioxide (a) There are double bonds between silicon and oxygen atoms () Silicon atom is bonded to two oxygen atoms (©) Bech silicon atom is surrounded by two oxygen atoms and each oxygen atom is bonded ta two silicon atoms (q) Bach silicon atom is surrounded by four oxygen atoms and each oxygen atom is bonded to two silicon atoms. ‘The lanthanide contraction is responsible for the fect that (a) Zr and 2n have the same oxidation state (8) Zrand Hf have about the same radius (© 2r and Nb have similar oxidation state (d) Zr andY have about the same radius. 2005/4 43, The TUPAC name of the coordination compound K,(Fe(CN) gl is (a) Tripotassium hexacyanciron (Il) () Potaasiom haxacyanoiron (Il) (c) Potassium hexaeyanoferrate (111) (@) Potassium hexacyanoforrate (M1). 44, In which of the following arrangements the order is NOT according to the property indicated against it? (@) Lic NacK @ and B%y > 0. 9. (6) H-Bomb is based on the principle of nuclear fusion, ‘ 10. (b) Number of A ions per unt cell= $ = 2 Number of B ions per unit cell = 2 = Hence empirical formula is AB, 11, (@) HCF,COOR is strongest acid and thus highest de- gree of ionisation. Thus its eondvetivity ie maximum. 12, (6) Magnitude of lattice energy of ionie compound de- pends upon charge on the ions and size of the ions. 18. (d@) Energy profile diagram for endothermic reaction is as shown, Bvidently, E, AP Blectricity required for production of 27g of Al = 3 x 96500 6 Electricity quired for production of 6.12 x 10g of AP = 396500 «5:12.10? a = 5.49 x 107C. Mécottoial) Vico) 16. (a) N; (g) + 3H, @) + 2NH, (@) 16. (a) An=2-4 4H =aU-( 2) RT = aU +2RT ‘Thus AH > AU. 1D. (0) Ky = (MPU f= 4x0 ‘Now, at equilibrium (M?] = (% ]x 3 or [X]=(M*' x2 ‘Thus , [M2] (237? = 4 10-8 15" 75 1x1 Po= Boas > 57 50 tom: 19, (¢) The statement 3 is egainst the fact. Actually on in creasing the temperature, the fraction of molecules with ‘ish probable speed decreases. 20. (d) 2NOsla) == SNO, (2) + O52) y= hy (RT, (An = 4-2 = 2) 8 x 10° x (0.0881 x 457? 59x 10-8 Thus &, >, 21. (@) Addition of F would shift the equilibruim in for ward direction in accordance with Le-Chatelier’s prin- ciple. 22, (a) pH = —log (H,0"1 Jog {H,0"1 = - pH =-54= 86 [H,0*] = antilog 6.6 = 5.98 x 10°. 23. (d) Unimolecular reaction. 24, (0) MV, +MV_ = MAY) 1.5 x 480 + 1.2 520 = My x 1000 My= ag99 = 25. (B) Less reactive metals such as Ag and Au settle down as anode mud. 26. (8) A” yone = A wsoa: + MHC A pact = 910 + 4262 ~ 126.5 390.7. . (e) Mass of one mole of the substance will increase two fold 28 (a) These correspond to orbitals of the same subshell Gd. 8 20058 COMPREHI CHEMISTRY FOR AIEEE 30. 31. 32. 33, 34. 36. 37. 38. 30, 40. ai. 42. 43. 44, (@) NaC has smallest ions. LiCl is covalent in charac- ter due to high polarization. (@) Since K,, is increosing with decrease in (increase in 1/1), the reaction must be exothermic. (d) Cu,0 + Cu,S -> 6Cu + SO, Cuprous sulphide brings about reduction of cuprous oxide. This reaction takes place in Bessemer converter ‘during metallurgy of copper. (a) SF, is see-saw shaped with 1 lone pair around S. GF, is tetrahedral with no lone pair around C. Xe, is square planar with 2 lone pairs around Xe. (®) Mixing these two oppositely charged sols would re- sult in coagulation. (e) Hyrophosphorus acid is HPO, { ois sie cso aa ats phosphorus atom, (@) OW > OF + HY Conjugate base of OH is O*. ()AL05. (©) HF > HCl > HBr > HI ‘Thermal stability decreases with increase in atomicsize of the halogen. (d) Hg,Cl, + 2NH,OH — Hg + Hg(NH,)Cl + NHCI + 2H,0. fe) TC = CF. There are two pi and one sigma bond. (a) K,Cr,0; + 4H,S0, > K,80, + Cr{SOpy + 44,0 + 310) 2KI + K,S0, + 10} + K,80, + H,0 +1) «3 Oxidation state of Gr in Crg{SO,) is +3. wm -0- (8) Zr and Hf are the corresponding eloments belonging to second and third transition series. (©) KelFetCN),) Potassium hexacyanoferrate (IID) (c) Theionization enthalpy of N should be greater than that of O due to its stable electronic configuration, (6) SO,* contains 42 electrons whereas CO;* and NOs” contain 32 electrons each. (©) CH Her + By ck, Br CH= C~ CH= CH + HBr cH, Ease of substitution of H atoms is 3° > 2” > 1°, a. . (e) 1, 3-butadiene under OY R-X + Be . (a) For alkenes other cohtaining three bidentate Modicwatenperin ech henge shows optical noe (®) An octahedral compl ligands has a chiral stract on ite mirror image and (@ CO” ion has configufation 3a. Hence in the pres- exce of strong ligand field all the electrons get paired. (q) PCC can oxidise the llylic aleobol to ketone with- out effecting the double: ©) Hg sys b+ bromo-2-butene predom (6) Wurtz reaction is tr | + Re —Cux. ting two moles of R - X with aa ¢ : cerca Gaetan cen me aba crn asa can show optical activity (@) Primary Aralkyl anfines are strongly basic due to absence of conjugation yfith the ring, wan ethene, addition follows Markowinkow's rule, (e) Lor, cF;], ie Tethn poly tetraftuor ethylene, (©) CH,CHCH,CH,, (@) The bp and fp are diffe (@ Clis a good lee displaced by the nucleo ‘AIEBE—2005 PAPER 2005/9 ee w.@ Sw 8G . to Se te Be bond a7 Naa 1H 3e—2e bond 67. (@) H atom is a wnielectron ryrtom and the oneray of arbitals depends only upon the value of x 68, (0) The 4f-cloctrons constitute inner shells and ere in- effective in sereening the nuclear charge 68. (2) theinonr = Yn G42) BLM. The value of 2.84 BM cor- responds to the presence of 2 unpaired electrons. é sys- tem in strong ligand field will pair one of the electrons and give two unpaired electrons. cH, 10. ) 0+ OK End (enamine) 71. (d) When two electron releasing groups are present the incoming group will occupy para or artho position to the group which has more + R effect. yd 72. (a) BXi+ GY XY a= 2% + F0-H-A-¥) or ~ 200 = 1.00% ~X) + 0.5K - X)~- 1.0X-X) = 0.5K - X) 200 a or X=X)= TP = 400 ke mol 73. (a) NH,ES(s) = H,Sig) + NHs@) (@) @) Initial pressure = 0.50 atm due to presence of ammo- nia. Final pressure = 0.84 = 067 x0.17 = 0.1139 atm. 74. (b) The E. Formula from given %age data is NaHCO NH, C—NH,—*> 5. (c) For a first order reaction 2.303, 2.308 og Aa) te eta AIEEE (AET)2004—Solved 1. Which of the following sets of quantum numbers is correct for an electron in 4 f orbital ? (a)n=4[23,mae4sne (yn = 3,1 =2, m= —2, ©n=4l=3merhent @n=4l=4m 2. Consider the ground state of Cr atom (Z = 24). The numbers of electrons with azimuthal quantum num- bers, ! = 1 and 2 are respectively, (@) 12 and 4 ()16 and 5 (16 and 4 12 and 5, 38, Which one of the following ions has the highest value of ionic radius ? @ur or or @B™ ‘The wavelength of the radiation emitted, when in a hydrogen atom cloctron falls from infinity to station- ary state 1, would be (Rydberg constant = 1.097 x 10? 4aee mr) (@) 91 nm 9.110% nm (6) 406nm @1920m. 5. The correct order of bond angles (smallestfirst) in H,S, NA, BF, and SiH, is (a) H,S < Sif, < NH, < BF, (6) H,S< NH, requires first an exothermic and then an endothermic step as shown below OW+ = 142 mo? O@) aH O@+e=O@F; AH" = 664 ks molt ‘This is boceuse (@) Oxygen is more electronegative (©) 0" ion hus comparatively larger size than oxygen atom (c)O- ion will tend to resist the addition of another electron (@) Oxygen has high electron affinity. ‘The states of hybridization of boron and oxygen atoms in boric acid (H,BO,) ere respectively (@) sp? and sp* () sp? and sp? (isp? and sp? (a) op? and op? Which one of the following has the regular tetrahedral structure ? (o) Ker, © EKER, BFE SF, (Atomic nos : B = 5, S = 16, Ni = 28, Xe= 56 Of the following outer eleccronic configurations of at- coms, the highest oxidation state is achieved by which one of them ? (o) (n= detns* (n= Dehn? (0)(n— Deng? @in- Dens, As the temperature is raised from 20°C to 40°C, the ‘average kinetic energy of neon atoms changes by a fec- tor of which of the following ? 1 ws oe 313 oF @ p1s7e35 ‘Tne maximum number of 90° angles between bond pair- bond pair of electrons is observed in (a) dep? hybridization (6) apd* hybridization (dept hybridization __(d) spe hybridization. Which one of the following aqueous solutions will exhibit highest boiling point 7 (2) 0.01 MNa,S0, 5) 0015 M glucose (0.015 Mures (@) 001 M KNO,, Which among the following factors is the most important in making fluorine the strongest oxidizing halogen ? 20042 ‘COMPREHENSIVE CHEMISTRY FOR AIEEE a. 18, 19, 2. (a) Electron affinity (6) Bond dissociation energy (c) Hydration enthalpy —_(d) Ionization enthalpy. In van der Waals equation of state for real gases, the constant °’ is a measure of (a) intermolecular repulsions (6) intermolecular collisions per unit volume (©) volume eecupied by the molecules (c intermolecular attraction. ‘The conjugate base of H,PO, is (@) PO HPO CHO, PQ, 6.02 x 10 molecules of urea are present in 100 mL of its solution. The concentration of urea solution is (0.001 M. 01M @001M = 6.02 x 10 mol", ‘Toneutralise completely 20 ml of 0.1 M aqueous solu- tion of phosphorous acid (H,PO,), the volume of 0.1 M. aqueous KOH solution required is (@)10 mL @)60 mL (140 mL @) 20 mL. For which of the fellowing parameters, the structural isomers C,H,OH and CH,OCH, would be expected to have the same values ? (Assume ideal behaviour) (q) Heat of vaporization (@ Vapour pressure at the same temperature. Which of the following liquid pairs shows a positive deviation from Raoult’s law ? (a) Water—hydrochlorie acid () Acotone—KC1> CH,COOH > sucrose (@) The osmotic pressure (x) of a solution ia given by the ‘equation x= MRT, where M is the melarty ofthe sckution. ‘What type of erystal defectis indicated in the diagram below? Na* Cl Na* Cl Na* Cl- Cl-oCr Na*o Nat Na‘ Cl a Cl Nat Cl- Cl Na* Cr Na* 0 Na* in volume from 1+ 109 m! to K st @ constant pressure of 1 x 10 Nm. The Jrork done is, (a)—8005 ©) 9904 (©) 270 kt (@)- 900 ks. 26, In ahydrogen-oxyger fuel-cell, combustion of hydrogen occurs to (a) generate heat (6) remove adsorbed oygen from electrode surface (@) produce hich purity}water ‘between the twa electrodes ‘the concentration of the reac- ).8 M to 0.4 Min 15 minutes. The time taken for the cofcentration to change from 0.1 M i} |(®) 60 minutes (@ 16 minutes, expression for the reaction neat 2} (Oy) (@)25 x10" 0.02 (4x 104 @50. 81. Therate equation for fhe reaction 2A + B—> Cis found (f) value off is independgntof the initial concentrationsof A. and B (e) rate of formation of Chis twice the rate of disappearance ofA (d) ty, in content, 82, Consider the followinglE Erpavjpgs £0.77 Brggygq =- 0.16 ‘AIEEE—2004 PAPER 2004/3 ‘Uniler standard conditions the potential for the reaction Sais) + 2Fe(aq)—+ 2 (aq) + Sn*(aq) is (@)1.68V ogy @onv @iey. 88. The molar solubility (in mol L-¥)of a sparingly soluble salt MX, is‘s’. The corresponding solubility product is ,'8'i8 given in terms of K,, by the relation Geek si28 2 = 2565 (5 = 056K)" @s= (28K! 84, The standard emf of a cell, involving one electron changeis found to bo 0.501 V at 25°C. The oquilibrium constant of the reaction is (F = 96,500 C mol-1 8,314 JK“ mol) (0)1.0» 108 @) 1.0% 108 (6) 1.0 10” (21.0% 108, 35. The enthalpies of combustion of carbon and carbon monoxide are 393.6 and —289 kJ mot" respectively. ‘The enthalpy of formation of carbon monoxide per mole is (a) 1105 by - NOs ks (©)~ 676.5 is (675. 36. Tho limiting molar conductivities A° for NaC}, KBr and KCI are 126, 152 and 150 S em? mol! respectively. ‘The A° for NaBr is (@)128 $ em mot! () 302 S em? mot (©1278 cm* mot" (@)176 8 ca® mot, 87. Inacell thst utilises thereaction Zn(s) + 2H"(oq)—> 2n(aq) + Hy(g) addition of H,SO, to cathode compart- ‘ment, will (a)lower the E and shift equilibrium tothe left @)increase the B and shit equilibrium to the lot (©) inerease the B and shift equilibrium to the right (d)lower the E and shift equilibrium tothe right, 38, Which one of the following statements regarding he- ium is incorrect ? (@) It i used to fil gas balloons instead of hydrogen because itis lighter and non-inflammable (6) It is used in gas-cooled nuclear reactors () Tt ia used to produce and custain powerful cuperconduc:- ing magnets (a) It is used as a cryogenic agent for carrying out expert- ‘ments at low temperature: 99, Identify the correct statement regarding enzymes : (a) Enzymosare specific biological cetalysta that can normaly function at very high temperatures (T ~ 1000) (b) Enaymesare specific biological catalysts that posses well- defined active sites (©) Enzyines are specific biological catalysts that cannot be poisoned (a) Enzymes are normally heterogeneous catalysts that are very specific in their action. 40. i. 45. a. ‘One mole of magnesium nitride on the reaction with an excess of water gives (@) one mole of ammonia (6) two moles of nitric acid (two mcles of ammonia (a) one mole of nitric ecid. Which one of the following ores is best concentrated by froth-floatation method ? (2) Magnetite () Malachite (©) Galena (a) Cassiterite, Beryllium and aluminium exhibit many properties which are similar. But, the two elements differ in (a) exhibiting maximum covalency in compounds (©) exhibiting amphoteric nature in their oxides (6) forming covalent halides (d) forming polymeric hydride, Aluminium chloride exists as dimer, Al,Cl, in solid state as well as in solution of non-polar solvents such as benzene, When dissolved in water, it givos (a) APY + 301" (®) AIO, + HCI (IAMOH)I* + 3HCL_ (a) IAKH,O),"* + 3C ‘The soldiers of Napolean’s army while at Alps during freezing winter suffered a serious problem as regards to the tin buttons of their uniforms. White metallic tin buttons got coverted to grey power. This transforma- tion is related to (@) an interaction with nitrogen of the air at very low tem- peratures. (6) an interaction with water vapour containedin the humid aie (6) a change in the partial pressure of oxygen in the air (d) achange in the crystalline structure of tin. ‘The Fg; Values for Cr, Mn, Fe and Co are—-0.41, +157, +0.77 and + 1.97 V respectively. For which one of these metals the change in oxidation state from + 2 10 +3 is casiest ? Gr (co Fe (Mn. Excess of KI reacts with CuSO, solution and then Na,S,0, solution is added to it. Which of the stazements is incorrect for this reaction? (a) Cah is formed (6) Bvoled I, is reduced (©) NajS,0, wsoxidised (4) Cul, is formed. ‘Among the properties (a) reducing (63 oxidising (c) complexing, the set of properties shown by CN ion towards metal species is, (aab abe Oca @be. ‘The coordination number of e central metal atom in a complex is determined by (@) the number of ligands around a metal ion bonded by sigma bonds 2004/4 TENSIVE CHEMISTRY FOR AIEEE (®) the number of only anionic ligands bonded to the metal ion (6) the number ofligands around a metal ion bonded by sigma and pi-boads both (@) the nuinber of ligands around a metal ion bonded by pi- bonds. 49, Which one of the following complexes is an outer orbital complex ? (a) iFe(CND, © INN) © (com) @ IMarem+. (Atomic nos. : Mn = 25 ; Fe = 26; Co= 27 ; Ni = 28) 50. Coordination compounds have great importance in bic- logical systems. In this context which of the following statements is incorrect ? (@) Chlorophyll are green pigments in plants and contain (b) Carboxypeptidase-A is an enzyme and contains zine (©) Cyanocobalamin is B,, and contains cobalt (@) Haemoglobin’ the red pigment cf blood and contains iron. Cerium (Z = 58) is an important member of the lanthanoids. Which of the following statements about cerium is incorrect ? (a) The common oxidation states of cerium are +3 and + 4 ()Ceriam (IV) ects as an oxidizing agent (© The + 4 oxidation state of cerium is not known in sote tions (@) The + 9 oxidation state of cerium is more stable than the +4 exidation state. Which one of the following has largest number of iso- mers? Ga) (RutNHt) Cl) ©) (Colem,c (@)MAPR,)H(CON ——_—_(d) (CotNH,),C™, (R= alky! group, en = ethylenediamine) ‘The correct order of magnetic moments (spin only val- ‘ues in B.M) among is (0) (MnCl > [CoC * > (ReICN) (@)IFe(CNDg > iCoCI* > EMCI) (0) [Re(CN),)* > [MnCl,}? > [CoC (@) IMnCl,J* > (Fe(CND,}* > (CoC. (Atomic nos. : Ma = 25, Fe = 26 Co= 27) 54. Consider the fellowing nuclear reactions 51. ‘The number of neutrons in the element L is (id 145 140 ss, ‘The half-ife ofa radioisotope is four hours, Ifthe initial mass of the isotope was 200 g, the mess remaining undecayed after 24 hours: 57. 5B. 1. 4. (@) 1042 g 416) g ©3125 5 (2.08 ‘The compound formed retest for nitrogen , inte post with the Lassaigne solftion of an organic compound is G) Fog FACN, (8) Na,{Fo(CN),NOS} © FecN, @ NaglFe(CN),. ‘The ammonia evolved from the treatment of 0.90 g of fan organic compound for the eetimation of nitrogen was passed in 100 mij of 0.1 M sulphuric acid, The for 20 mL of 0.5 M sodium hy- droxide solution for eqmplate neutralization. The or- ganic compound is (a) acetamide (6) thiourea (@) urea (@) benzamide. ‘Which one of the follofving has the minimum boiling point? | (@) nButeno (@) isobutane (©) 1Batene (@)1-Batyne, The TUPAC name of tHe compound is (@) 3, Sdimotiyl-14 cyplchexane Which one of the following |does not have sp? hybrid- ized carbon ? (a) Acetone (8) Acotamide (c) Acetonitrile (@) Acetic acid Which of the followinghwill have a meso isomer also? (a) 2.Chlorobutane (8) 2-Hiydroxypropancie acid (€)2,3-Dichloropentane | (4) 2, 3-Dichlorobutane. Rate of the reaction is fastest when Zis ()0cocH, (a) CH,-CH,—CHl,--CH, (6) CH,—CH,—C= CH H CH ! 1 (©) Cy (@)CH,—CH,—CH—CH. Di Call, Consider the acidity ofthe carboxylic acids (® PhCOOH, 0-NO,C,H,COOH (id) p-NO,C,4,COOH (iv) m-NO,CH,COO! ‘ATEEE—2004 PAPER 2004/5 66. 67. ‘Which of the following order is correct ? Gla>b>erd O)b>e>d>a @bodrare Dbsdsers. Which of the following is the strongest base? @) Sm, © CS cus, CH ‘Which base is present in RNA but not in DNA? (a) Uracil () Thymine (©)Guanine (@) Crtosine, ‘The compound formed on heating chlorobenzene with chloral in the presence of concentrated sulphuric acid, (@) gammexene (©) hexachloroethane (6) freon @ DDT. On mixingethyl acotate with aqueous sodium chloride, the composition of the resultant solution is (@) CH,COOC,Hi, + NaCl (6) CH,Cl+ C,H,COONa (©CH,COCI + €,H,0H + NaOH (@) CH,COONa + C,H,08. ‘Acetyl bromide reacts with excess of CH,Mgl followed by treatment with a saturated solution of NH,Ci gives 1. Which one ofthe following undergoes reaction with 50% sodium hydroxide solution to give the corresponding alcohol and acid? (0) Phenol (@)Bensoic acid (© Butanal (@)Bensaldehyde. ‘Among the following compounds which can be dehy- drated very easily is (0) CH,CH,CH,CH,CH,OH (menyen,ererch DH tty ay I (©) CH,CH, CCH,CH fl oH on (@CH,CH,CH,CHCH, 13. Which of the following compounds is not chiral ? (a) 1-chleropentane (6)8-chloro-2-methyl pentane () -chloro-2-methyl pentane (@) 2chloropentane. 74, Insulin production end its action in human body are responsible for the lovel of diabetes. This compeund belongs to which of the following categories ? — oa kaha erie a ‘70. Which'one of the following is reduced with zinc and 1 " hydrochloric acid to give the corresponding hydrocar- 75+ The smog is essentially caused by the presence of (a) Ethyl acetate () Butan-2-one (©) Oxides of sulphur and nitrogen omges er oan ANSWERS 1. (©) 2. @) 3% fe) 4. (a) B. (ce) 8. (a) 7. (b) 8. (d) a ee peep et ee a a ee ee a 4 1 a 10. a cm Solutions (©) For 4felectrons, the set ofquantum number aren (@) Blectronic configuration of Cr is 15%, 26728, 6°Sp*3d, do!, 1 = 1 represent p-level and! = 2 repre- sents d-level. Electrons in p-level (= 1) = 32 Electrons in d-level (I= 2) = (©) Among the given species, the increasing order of ionic radil is BY < Lit < Fo < of @ = 1.097 x 107 mt 1 m= 91.18 «10m 1097 x10" e or =91nm (©) The correct bond angles along with the increasing order is = HS < NH < Si < BR (0h ~ eed ~ cowety ~ cach (G)The isoelectronic species (containing 18 electrons each) are K*, Ce, Sc, Cl- ©) Al, < SiO, < ‘Amphateric Weatly mane PQ; < S80; Weakscid — Moderntely sone (@) Higher the bond order, lower will bethe bond length. ‘Thus, bond length in NO is > than in NO*, (2) The step 0") + — 0%) is endothermic. ‘The energy is required to overcome the replusive in- teraction between negatively charged electron and the monovalent antion, () Boric acid has a network structure in which each boron is trigonal (sp*-hybridised) and each O atoms tetrahedral (sp*-hybridised) (©) Among the given species BF,“ has tetrahedral struc- ture, XeF, is squareplanar ; SF, issee-saw shaped and INi(CND, is also square planar. (©) Among the given electronic configurations, +7 state ‘can be attained by (n — 13d8 ns? K.Euo _ 313 Kon” 298 © KE«T « 4. 16. 1. 18, 19. () spd ordsp! (©) High value of fluorine to the (©) B is called co-vol volume occupied by @) HPO" rer N N @ Na Nev or M= =0.01M (€) HPO, isa dibasi is 2H,0 Now, MiMi. “EE eS, or Ve =40mL 256, ‘AIEEE—200¢ PAPER 2004/7 22. (@) A mixture of benzene-methanol among the given 84, (c) ‘AG? = — 1 x 96500 « 0.591, pairs will exhibit positive deviation from Raoult’s law. =~ 2.308 RT log &, 28. (6) The freezing point depression will depend on the ‘96500x 0.591 value of eryoscopic constant of the solvent. or logh,= Ssescpartcae 79.997 10 44, (d) Here, equal number of Na* and Cl-ions aremissing - x10 from their regular lattice positions in the rock-salt % See ee ere at suas. Osh peraiea s ouely ete ® ¢+0,— = AH, + a8, 25. (a) Ws 105 (1 107-1109) jae 2 AB AR = ABs =-9005 CO+-0; — 393.5 - (- 283) . aH, =- 110.5 26. (c) The product of fuel cell reaction is pure water. C0, ” 27 (@) The given data reveals that typ = 15 min. Thus, the concentration of reactant will fall from 0.1 M to (a) 4° (NaBr) = A* (NaCl) 4 A* (KBr) — A° (KCL) 0.025 M in two half lives. Thus, time required = 126 + 152-150 = 128 sem? mot =2%15= 30min, .059 | TP 1. (e) 2080 |, BEE 28. 6) Kg= LPO oat [P,lO,P On adding H,30,, the [H*] will increase. As a result Now [P,Q] = [P,| = 1 being solids E,au Will also increase and shift the equilibrium to right. 1 38. (a) Helium is not lighter than H,. Xo" fo, 39, (b) Enzymes are shape selective biological catalysts 39. (a) For the reaction, which normally functions effectively at body tempera- (COg) + Cl jg) —+ COCI{g), An = 1-2-1 ‘ture. e Kp=Kg(RT)* or ra «ery 0) Mase 1.0 — MEOR + aN INoF 41, (©) Froth floatation process is used to concentrate gen- 30, (d) Given, ROE 24x 104 erally sulphide ores. ae 42, (a) Be is divalent wheras Al is trivalent. 1 Lowy 48, (d) Al,Cl, + 12H,0 ——+ 2{AKCH,O),)* + 6CI- — > yey} Bo forrnction, NOG) <——= gNsl)* OMe). Gay wise tn gotncrivared inno grey hh which bali brittle crumbles down to a powder in cold climate. 45. (a) E",,, is minimum for Cr, consequently its, highest. Hence its oxidation will be easiest.’ 81, (6) Thevalue ofrateconstant is independent of initial 4g, (d) 4KI + 2CuSO, —> Cul, +K,S0, + 1, concentration at a given temperature. 1,+NaS,0,——>Na,8,0, + 2Nal 82, (¢) The given reaction reveals that Sn is oxidised and Gul, is not formed at any stage. Fe® jons are reduced. Thus standard potential forthe 2 . nen (©) ON- ion acts as a reducing agent as well as reaction is Epp ~ Eagtfaq complexing agent. = 0.77 —(-0.14) = 0.91 V (@) The number of ligands around the metalion bonded uaeieenieesan by sigma bonds. 33. ©) MX, =——> Mt + 4 ; . (6) Complex rid state of Centrat hyp = (MIE [ If ts solubility in ° ——“—" mol Lt (Fe(cNy]- Peps ean a wets | then nM) = Macon oe | ee = 256 5° Ki=4s {CoH} asp? us an outer orbital - -(% INH optat | Dover [ze 51 (a) Chlorophyll, a green pigment of plant does not ‘contain calcium but instead it contains Mg. (©) The + 4 oxidation state of cerium is also known in solutions. However it tends to change into + 9 state. Hence the statement (3) is false. (@) Among the given species, {Co(en),Cl,]* cation exhibit, cig, trans as well as optical isomers. (@) Complex No of unpaired electrons IFe(cNy + Zero (coc Three inci Five ‘Now, larger the number of unpaired electrons, higher is the spin only magnetic moment, Hence, [MnCl > [CoCl,?>[Fe(CN)]* @ 738M ——+728N +24He 28N + 20 + 2° ‘Thus, number of neuirons in element L = 230-86 = 144. © ty=4br. «. 24hr=6 halflives Ef « oe. came (@) 8Na,{Fe(CN)g + 4Fe* ———+ Fe[Fe(CN)glp + 12 Nat ‘Prussian bie N=N, 20x08 ox 0.1 20x05 2 TO OP ine Vol. of H,SO, used by ammonia = 100—50 = 50 mL, Miltimoles of H,SO, used by NH, = 50 x 0.1=5 Millimoles of NH, formed = 2x 5 = 10 10x10" x14 %ofN a ‘This percentage refers to % age of N in urea 4 x 2x 100 ie (222E30 1008) will be shown by isobutane. (6) Among the given species, the lowest boiling point. (c) The IUPAC name of the compound is 3, 3-Dimethyt-I-cyclohexanol. Sif © =50mL x 100= 46.6% dis€ =0 on oti aid 2 # CHy—E =n cH,—€ <0 ‘Acronis | NH, Acstanide G1. (c) 2, 3 Dichlorobut options Cl is the best leaving group hence the ratelof reaction will be fastest in case 9 | ofe—c1, | 63. (c) Among the give: bai ‘2-eyclopropylbutane has chiral struct of increasing acid strength is COOH COOH No,, * ®» 65. (6) Lone pair of N ‘not involved in pronation is, 66. (a) Uracil is presea on. © @ benzyl amine C,H,CH,NH, is ance. Hence its availability for 1 om Dbutnot in DNA, @oT) on mixing NaCl and ethyl ‘AIEEE—2004 PAPER 2004/9 ° I ICH Mel (excoo) 69. (¢) CHy—C—Br ——— CH I CH,—C—OH + MplBr + MefOHIBr CH ‘2Methy12 propanol Il 2a-Hg 10. @) CH,—C—G,y ——% CH,CH,CH,CH, ‘2Butanone Batane It refers to Clemensen’s reduction 74, %. @ Benzaldchyde undergoes disproportionation with 50% NaOH to given benzyl alcohol and sodium benzoate 508 W208 CeH,CHO GjH,CH,OH + C,4,COONa} ‘This is Cannizzaro's reaction. (©) 3-Methylpentan-3-ol will be dehydrated most read- ily as it gives 3° carbocation intermediate. (@) Among the given species 1-chloropentane isachiral structure (CH,CH,CH,CH,CH,C)) @ Insulin is a biochemically active peptide hormone secreted by pancreas. (@) Photochemical amog is caused by oxides of sulphur and nitrogen. AIEEE (AET) 2003—Solved In Bohr series of lines of hydrogen spectrum, the third line from the red end corresponds to which ane of the following inter-orbit jumps of the electron for Bohr or- bits in en atom of hydrogen ? (32 1532 41 (245. ‘The de Brogile wavelength of a tennis ball of mass 60 g moving with a velocity of 10 metres per second is approximately (a) 10 metres, ©) 10" metres () 10 metres (@) 10 metres. Planck's constant, h = 8.69 x 10 Js. ‘The orbital angular momentum for an electron revolving in an orbit is given by id+D. 2, This momentum for an s-electron will be given by 1h Oa aR ©) zero k A OR Of -%- How inany unitoells are present in a cube-shaped ideal crystal of NaC! of mass 1.00 g? {Atomic masses : Na = 23, Cl=95.51 (0)2.57 x 10" unit cols @)5.14 x 10" unit cells (©) 1.28 « 108 nit colt (1.71 x 10" unit eal. Glass isa (aq) micro-crystalline solid (6) super-cooled liquid ge (@ polymeric mixture. ‘Which one of the following statements is correct ? (@) Manganese salts give a violet borax bead test in the re- ducing flame @) From a mixed precipitate of AgC] and Agl, ammonia so- lution dissolves only AgCI () Ferric ions give a deep green precipitate on adding potas- sium ferrocyanide solution (a) On boiling a solution having K*, Ca® and HCO, ions we get a precipitate of K,Ca(CO,)y, According to the Periodic Law of elements, the varia tion in properties of elements is related to their (a) atomic masses () nuclear masses (©) atomic numbers (@) nuclear neutron-proton numbers ratio, 10, L 18. ue Graphite is a soft solid lubricant extremely difficult to melt, The reason for this anomalous behaviour is that graphite @) ia non-crystalline rubstance (©) isan allotropic form of diamond (c) has melecules of variable molecular masses like polymers @) has carbon atoms arranged in large plates of rings of strongly bound carbon atoms with weak interptate bonds ‘The IUPAC name of CH,COCH(CH,), is (a) Inopropylmethyi ketone (6) 2-methyl-2-butanone (@) 4-methylisopropy! ketone(d) 3-methyl-? butanone. ‘When CH, = CH — COOH is reduced with LiAlH,, the compound obtained will be (@)CH,—CH,—COOH (b) CH, = CH—CH,OH (@CH,—CH,—CH,OH (@) CH,—CH,—CHO. ‘According to the kinetic theory of gases. in an ideal gas, between two successive collisions a gas molecule A reduction in atomic size with increase in atomic number is a charactaristic of elements of (e)high atomic masses (b) d-block (@) block (@) radioactive series. ‘The general formula C,H,,0, could be for open chain (= aicetonee (@) carboxylic acide @) dialdehydes, ‘An ether is more volatile than an alcohol having the ‘same molecular formula. This is due to G) dipolar character of ethers ) aleohols having resonance structures @)intor-molecalar hydrogen bonding in other (@) inter-molecular hydrogen bonding in alcohels. Among the following four structures Ito IV, ca, poe oatt—bi—o,, cto © ay wb A JE I CH,—CH—o.H, an ™ COMPRERENSIVE CHEMISTRY FOR AIEEE 18. 47. 18. 2. itis true that (@) all four are chiral compounds () only I and II are chiral compounds © only 11 is a dhiral compound @ only Il and IV are chiral compounds. Which one of the following processes will produce hard water? () Saturation of water with CaCO, (@) Saturation of water with BigCO, (©)Saturation of water with CaSO, (@) Addition of Na,80, to water. ‘Which one of the following compounds has the emall- ‘est bond angle in its molecule ? (@)S0, ®) OH, SH, (d) NH, ‘Which one of the following pairs of molecules will have permanent dipole moments for both members ? (@) SiF, and NO, (®) NO, and CO, (@)NO, and 0, (d) Sir, and CO, ‘Which one of the following groupings represents a col- lection of isoelectronic species ? (At. nos, : Os : 55, Br : 38) (a) Nav, Ca, Me ON FL Ne (Be, AI, Cr (@) Ca®, Co", Br. In the anion HCOO- the two carbon-oxygen bonds are found to be of equal length. What is the reason for it? (q) Electronic orbitals of carbon atom are hybridised () The C = 0 bond is weaker than the C—O bond (© The anion HCOO- has two resonating structures (@ The anion is obtained by removal of a proton from the ‘acid molecule. ‘The pair of species having identical shapes for mol- ecules of both species is (a) CF, SF, (6) XeF,, CO, (BF, PC, Pry Fy, ‘The atomic numbers of vanadium (V), chromium (Cr), manganese (Mn) and iron (Fe) are respectively 23, 24, 28 and 26. Which one of these may be expected to have the highest second ionization enthalpy ? @y Cr (Mn (de. Consider the reaction equilibrium 280,(g) + 0,{g) ——» 280,() ; AH? =- 198 kJ. On the basis of Le Chatelier’s principle, the condition favourable for the forward reaction is (a) lowering of temperature as well as pressure (increasing temperature as well as pressure () lowering the temperature and increasing the pressure (@ any value of temperature and pressure, 27, a1. What volume of pressure will be co: elemental boron (at en as, at 273 K and 1 atm. jumed in obtaining 21.6 g of mig mass = 10.8) from the reduction of boron tridhloride by hydrogen ? (e961 ihgat @ieL daze, For the reaction equilibrium, N,0,¢) <= 2NO,(@)the\concentrations ofN,O, and NO, at equilibrium ard4.8 x 10+ and 1.2 x 10mol Lt respectively. The valup of K, for the reaction is (@)33 x 10'mol L?* (6)3 x 107 mal L (6)3 x 10 mol L* (d)3 x 10 mol L, of k sparingly soluble AB, is solubility product number will 410 (1x10 @ 1x10" ‘When during electrol coulombs of charge pj is of a solution of AgN, 9650 rough the eleczroplating bathe the mass of silvgr deposited on the cathode will ues ine nes tous For the redox reactior Zn(s) + Cu* (0.1 M) -Zn** (1 M) + Cu(s) taking place in a coll, E°,qy is 1.10 volt. Egy for the coll will be RT (cou oas) coast wr40% tne) ml! quemlnota enka he degree of ionization is ).3. Taking , for water as 1.85, ‘solution will be nearest to ®)-0.360C (@) + 0.480°C. ion botwoon the substances A and B is given by Rate = IAI” [B}™ (On doubling the conc concentration of B, earlier rate of the re tration of A and halving the ratio of the new rate to the ion will be as 1 (0) Sons Oman (n-m on, jum hydroxide on titration n of hydrochloric acid gave litre value of 85 ml:Thp molarity of barium hydroxide solution was (a)0.07 os (0.28 035, ‘AIEEE—2003 PAPER 3 82. The correct relationship between free energy change in a reaction and the corresponding equilibrium con- stant K, is @sG=RTInK, @-sG=RTAK, ()8G* = RT In K, (@)-0G" = RTInK, 38. If at 298 K the bond energies of C—-H,C—C, C=C and H — H bonds are respectively 414, 347, 615 and , the value of enthalpy change for the HC = CH) + H,@)—> H,C— CH) at 298 K will be (a) + 250 ks (8) ~ 260 13 fo) +125 kd 125. 84, The enthalpy change for a reaction does not depend upon () the physical states of reactants and products (©) use of different reactants forthe same product (©) the nature of intermediate reaction steps (@) the differences in initial or inal temperaturesof involved substances. 35. A pressure cockor reduces cooking time for food be- cause (a)heat is more evenly distributed in the cooking space (6) boiling point of water involved in cooking is increased (©) the higher pressure inside the cooker crushes the food material (@) cooking involves chemical changes helped by a rise in temperature 96. If liquide A and B form an idea! solution, (a) the enthalpy of mixing is zero (©) the entropy of mixing is zero (c) the free energy of mixing is zero (d) the free onorgy as wel asthe entropy of mixing are onch 37. For the reaction systom, 2NO() + O,{g)-» 2NO,lg), volume is suddenly reduced tohalfits value by increasing the pressure on it.I the reaction is of first order with respect to O, and second ‘order with reapoct to NO, the rate of reaction will (a) diminish to ono-feurth ofits initial value (6) diminish to one-eight ofits initial value ‘increase to eight times ofits initial value (@) increase to four timos of its initial value 38, Fora cell reaction involving a two-electron change, the standard «m4, of the cell is found to be 0.295 V at 25°C. The equilibrium constant ofthe reaction at 25°C will be (1x10 @) 20.8 «107 10 @1x 10", 4. 45. 48. In an irreversible process taking place at constant T and P in which only pressure-volume work is being done, the changoin Gibbs froe onorgy (dG) and change in entropy (dS), satisfy the criteria (a(S), <0,(d0),,< 0 (6)(dS), 2 >0, (dG), <0 VAS, g=O,GGIg =O (2)(ASH,g =O, (4G), > 0. Which one of the following cheractoristies is not correct, for physical adsorption ? (a) Adsorption on solids is reversible (6) Adsorption increases with increase in temperature (6) Adsorption is spontaneous (d) Both enthelpy and entropy of adsorption are negative. In respect of the equation & = Ae-®¥*T in chemical kinetics, which one of the following statements is correct? (o) kis equilibrium constant (6) A is adsorption factor (©), is energy of activation (a) R is Rydberg’s constant. Standard reduction electrode potentials of three met- als A, B and C are respectively + 0.5 V.- 30 V and~ 1.2 V. The reducing powers of these metals are (@)B>C>a @)A>B>C (C>B>A @A>C>B ‘Which one of the following substances has highest pro- ton affinity ? (@)#,0 ons (NH, PH, Which one of the following is an amphoteric oxide? (a) 20 Nao (so, zo, ‘A red eclid is insoluble in water. However it becomes soluble if some KI is added to water. Heating the red solid in a test tube results in liberation of some violet coloared fumes and droplets of a metal appear on the cooler parts of the test tube. The red solid is (a) (NH,,Cr,0, Hel, (© Hgo Pro, Concentrated hydrochloric acid when kept in open air sometimes produces a cloud of white fumes. The ex- planation for it is that (a) concentrated hydrochloric acid emits strongly amelling HCI gas all the tire (© Oxygen in aie reacts with the emitted HICL gas to form a loud of chlorine gas (c) strong affinity of HCl gas for moisture in air results in forming of droplets of liquid solution which appears like a cloudy smoke (2) due to strong sfinity for water, cancentratad hydrochloric acid polls moisture of air towards ite. This moisture forms roplata of water nod hence the elena COMPREHENSIVE CHEMISTRY FOR AIEEE 41. 49. 51. ‘What may be expected to happen when phosphine ges is mixed with chlorine gas ? (a) The mixture only coals down (6)PCI, end HCl are formed and the mixture warms ap (€) PCI, and HC} are formed and the mixture cools down (d) PH, Cl, is formed with warming up. ‘Tho number ofd-cloctrons retained in Fe (At no. of Fe = 28) ion is @s ws os 6 What would happen when a solution of potassium chro- mate is treated with an excess of dilute nitric acid ? (@)C2 and Cr,0," are formed ()Cr, 0 and H,0 are formed ()CrOF is reduced to + 3 state of Cr (@)CrO}* is oxidized to +7 state of Cr. Inthe coordination compound, K, [Ni(CN),], the oxida- tion state of nickel is @-1 wo +1 @s2 Ammonia forms the complex ion [Cu(NH,),]” with cop- per ions in alkaline solutions but not in acidic solu- tions. What is tho reason for it? (a) In acidic solutions hydration protects copper ions (6)In acidic solutions protons coordinate with ammonia mlacle forming NE," ions and NH, mneele are mot (o)tn tains stutions insoluble C(O%D, is pected which issoluble in extess of any alkali (@) Copper hydroxide is an amphoteric substance One mote of the complex compound Co(NH,),Cl,, gives, 3 moles of ions on dissolution in water. One mole of the same complex reacts with two moles of AgNO, 00- lution to yield two moles of AgCl(s). The structure of the complex is (@)ICoNH,CH C1, ) (CoNH,),Cl) NH, ((CoNH),CLICL NH, @ IC(NH,),CI Cl, . NH. The radius of La® (Atomic number of La = 57) is 1.06, A. Which ono of the following givon valuoe will be cloc- est to the radius of Lu. (Atomic number of Lu = 71)? (a) 160A, @)140A 106A 0854 ‘The redionueleidé "34h undergoes two successive Bedecays followed by one c-decey. The atomic number and the mass number respectively of the resulting radionucleide are (a) 92 and 234 (©)90 and 230 (6) 94and230 (a) 92 and 230. a. The half-life of a radiqactive isotope is three hours. If the initial mass of thefisotope were 256 g, the mass of it remaining undecayed after 18 hours would be (a) 408 (805 (© 120g id) 16.0. Several blocks of megpesium are fixed to the bottom ofaship to | (a) keep away the shark | ()make the ship lighted | (c) prevent action of wat to time. This helps in (a) keeping it coot () developing interlock silicates GNCH NICS Nis cc ee cavcaNe, ccc) fH, Butene-1 may be convefted to butane by reacting with (@)za—HeI @sn—HOL © Zn—He Pan, The solubilities of carbbnates decrease down the mag- nesium group due te aftecrease in (a) lattice energies of 0 (©) hydration energies of| (©) interionic attraction (2) eatropy of solution fo During dehydratic with cone. H,SO,, the (©) elimination of water (@) formation of an ester. Which one of the follow)ng nitrates will leave behind a ‘metal on strong heating? {q) Ferric nitrate ©) Copper nitraie () Manganese nitrate | (d) Silver nitrate. ‘AIEEE—2003 PAPER G4. When rain is accompanied by a thunderstorm, the collected rain water will have a pH value G@) slightly lower thaa that of rain water without thunder. storm (slightly higher than that when the thunderstorm is not there (¢) uninfluenced by o:currence of thunderstorm mL. (© ethanoic acid and ammonium salt (d) mathylamine salt and ethanoic acid. ‘The intornal energy change when a system goes from state A toB is 40 kJ/mole. Ifthe system goes from A to B by a reversible path and returns to state A by an irreversible path what would be the net change in (d) which depends on the amount of dustin ar. en eoornr? ayaa Complete hydrolysis of cellulose gives Oasis (oie. (a) D-fructose (©) Deibose ° . glace (igloos. 72, The reaction of chloroform with alcoholic KOH and 66. For making good quality mirrors, plates of float glass P-toluidine forms are used. These are obtained by floating molten glass ‘overa liquid metal which does not solidify before glass. @ Hy IN ‘The metal used can be (@) mercury tin Pepe (magnesium. one O)we 67. The substance not likely to contain CaCO, is (a)a marble statse (©) calcined gypsum Coen sels @ dolomite © ne-{O)-wncuer, 68. The reason for double helical structure of DNA is operation of (a)van der Waals’ forces (6) dipole-ipole interaction w Hy Ne hydrogen bonding _(d) electrostatic attractions. 69. Boitles containing CgH,] and CgH,CH! lost their origi- ¥ nal lables. They were labelled A and'B for testing. A 7% Nylon threads are made of and B were separately taken in test tubes and boiled ee with NaOH solution. The end solution in each tube polyamide polymer —_(d) polyethylene polymer. was made acidic with dilute HNO, and then some 74, On mixing a certain alkane with chlorine and AgNO, solution was added. Substance B gave a yellow irradiating it with ultraviolet light, it forms only one prociphate. Which ono of tho fllowing statamanta ie monechlcroalkane. This alkane could be true for this experiment ? @) propane ©) pentane Aw CHI (©) isopentane (4) neopentane, ()A was CACHE ‘76. Which of the following could act as propellant for (©B wes CHI rockets ? (d) Addition of HNO, was unnecossary. G) Liquid hydrogen + liquid nitrogen 70, Reto] fseyene.on hytrotyd in aedlomadiom gon: Liquid oxyoon + liquid nitrogen be (©) Liguid hydrogen + liquid oxygen Ce Tt ee: (d) Liquid nitrogen + liquid oxygen. ANSWERS a) z@ =O ze @ @o 7 ©. = @ 2@ 1. ® © Be B ne 1% @ | BO 1. @ Ts @ 18.) 3._@ a) 0) BO | *@ 35) 36.) 7 Be) 30._(o) 30 ae) | = @ 3 @ uO 3. @) 36. ta) me 38 @) a @ | ow ae ao a ©) te) CoC) 8) ao | &@ @ Compnenehisive CHEMISTRY FOR AIEEE 49. (6) 50.) | OL. 62. (a) 53. (d) ‘64 (c) 85. (a) 56. (c) wo] 58 oO | ao a) @@_ | aw) oo] 8 @ @@ | 8 70 tar a_@_ | mo [me @ Solutions 1, (6) The lines falling in the visible region comprise Balmer series. Hence the third line would be n, = ng = Bie. 52 “h__66¥10-# B= To” Box Id x10 3. (b)Fors-electron,!=0. .. angularmomentam =zero 4. (a) Number of formulas in cube shaped crystals 10 = Fay ¥ 6.02 «10% Since in NaCl type of structure 4 formula units form a call, =10-m, 10x 602x107 B85 x4 = 2.57 x 10"! unit cells. isa transparent or translucent super cooled units cells = 5. (6) Glass is liquid. 6. (6) Ammonia can dissolve pat. of AgCl only due to for- ‘mation of complex as given below : AgCl +2.NH,— [Ag (VH,IC1 7. (e) The properties of elements change with a change in at. no. 8. (@) Chcice itsetis the answer, 8. (d) Seo TUPAC rules, 10. (6) LiAIH, can reduce COOH group and not the dou- blebond. cH, =cH—coon “> criy=cH—cH,0n 11, (According tokinetic theory the gat meloculos travel ‘in a straight line path but show haphazard motion due ‘to collisions. 12. (¢)fbleck olements show a rogular decrease in at. sizo due to lanthanide/actinide contraction. 13, (e) Selfexplanatory. 14, (@) Selfexplanatory. 15. (B)A chiral objector structure has four different groups attached to the carbonation. 16, (e) Permanent hardness of water is due to chlorides and sulphates of calcium and magnesium. 17, (©) In H,S, due to lowerfelecironegativity of sulphur the LP.-L.P repulsion isfnore than B.P-B.P repulsions and hence the bond anglp is 92°. * 18. (c) Both NO, and O, will have net dipole m 19.) N*, F- and Na* cont 20. (©) Solf explanatory, H- 21. (6) Both XeF, and CO, 22, (6) In chromium the removal of first electron removed from stable orbitals and therefore 28. (e) Due to exothermicit ‘temperature will be req ing to 2 moles. ‘High pressure will 2A, (6)2BC,+ 3H, —> sia. INO,)? 3 to) Be TNO = 310-9 mol L* 28. (a) AB, —= AY42.B- K,, = (Al BPs [1 x 1 27. (b) AgNO, —> Agt + NI figuration is 3d°.4s!. After 6 second electron has to be of reaction low or optimum es arechang- [2x 2110 28 tf oe1s0-ommsse 201. 29. (a) HK + xt fnew 10 ‘ At Eql. {1 - ar) ca ‘Total no, of fons = o(1 — se(l+a) +ea+casc[l-atara] == a1. 88 Effective cone. in solution =e[1 +a] = 0.2 (1 + 0.3] = 0.2 + 0.06 = 0.26 AT) = ~(1.85 x 0.26) = - 0.480" C. (@) Rate, =k [AP [BI Rate, = # (2A)" 1B)" Rate; _ Fi2AI" BY * Rate; # TAT IBY = BY [pir = 20.2- = are, (a) Ba (OH), + 2HC1—> BaCl, + 2,0 Applying Molerity equation, MN = Ma¥2 or 25 xM, = (Ba(OH),) GEC 01x35 _ 07 “ 2x25 10 (@) Self explanatory. (@ CH, = CH, (g) + H, (g)— CH, - CH, 6H =1(C=C)+4(C-H)+1(1-H) -1(C-C)-6(C-E =1(C=C)+1(H-H)- \(C-C)-2(C-) = 615 + 434 -347-2x 414 = 1050 ~ 1175 = = 125 kd. (©) Self explanatory. (@®) On increasing pressure, the temperature is also increased. Thus in pressure cooker due to increase in pressure the b.p. of water increases. (a) In the formation of ideal solutions. The forces of attraction (old) are equal to forces of attraction (new). (©) r= k [04] [NO}? . when the volume is reduced to ‘1/2, The conc. will double 2. New rate = k [20,) [2 NO}? = 8& [0,] [NO}? ‘The new rate increases by eight times. (d) The oquilibrium constant is related to the stand- ard emf of cell by the expression 01x35 2 2 - nn log K = Bea x Gogg = 0295x ee or Keilxio% (@) For spontaneous reactions, dS > 0 and AG or dG should be negative ie. <0. (©) As adsorption is an exothermic process. .. Rise in temperature will decrease adsorption. (©) Self explanatory. (a) Lower is the value of reduction potential more is the reducing power. 48. 41. 49. 7 (€) More basic is the compound, greater isits tendency to accept protons Nn,+H’ — Nu (a) It is amphoteric because it reacts with acids as well as bases ZnO +2NaOH > Na,ZnO, + HO nO + H,SO, — ZnSO, +H,0 (6) Hgl, + KI —> KyHgly ‘daetiabie (cabs On heating Hel, decomposes as Hel, — Hg +1, (a) 4801 +0, —> 201, + 21,0 Cloud of white fumes () PH, + 4C1, — PCI, + SHCL (d) Fer? = 348.409, (0) Cry 077+ OH —* 20102" + Ht ‘The above equilibrium shifts to L.HLS. on addition of acid. (yA Det DA = Sexe (®) Selfexplanatory. (@) CO (NH,),Cl, ==* [CO (NE), Cll"? + 2c1- + Strueture is [CO (NH,), Cll Cl. (e) The ionic radii of Lu’? ion is 0.92 A which is close to 0.85 A tm —, ex 4, ay — om. (@) typ = Bh. Initial mass (C,) = 256 g a ©, (©) Magnesium provides cathodic protection and pre- vent resting or corrosion. (®) Selfexplanatory. (©) An acidie solution cannot have a pH > 7. (©) Move is the no. of + I groups attached to N atom greater is the basic character. @ Factual. (®) The solubility is governed by Asien i, Auton * Aratiee ~ SBriptrton Due to increase in size the magnitude of hydration energy decreases and hence the solubility. 67. 68. 69. 10. ©) cH,cH,OH+H’ “24, oncH,—b—H K Pratonated shah (@ AgNO, —> Ag + NO, + } 9, (a) The rain water after thunderstorm contains dissolved acid and therefore the pH is less than rain water without thunderstorm. (©) (Celis), + 2H20 2+ nCgH 1205 D-chcoe (a) It is because mercury exists as liquid at room tem- perature, () Gypsum is CaSO, . 24,0. (c) Factual. (@) CHI will not respond to silver nitrate test be- catise CI bond has a partial double bend charactor. (@) CH,CH.N 5 C+H,0 ", (CH,CH,NH, + HCOOH. “. 5. ‘SIVE CHEMISTRY FOR AIEEE (e) For a cyclic process: energy is zoro because does not depend on the, @ ie net change in the internal ange in internal energy th) NH, NSc +CHCI, + 3KOH —> O + 3KC1+3H,0 cH, cH, (e) Nylon is a polyamide polymer. (d) In neopentane all tHe H atoms have same environ- cH, I mentCH, —C—CH; | Therefore it gives only one | CH, mono chloroalkane. (e) Liquid hydrogen. cellent fuel for rockets} liquid oxygen are used as ex- H() has low mass and high whoroas oxygen is a strong AIEEE (AET)2002—Solved 5 1 ‘Which of the following is a redox reaction ? (a) NaCl + KNO, —> NaNO, + KCI ©) CaC,0, + 2HC—+ CaCl, + H,C,0, (©) Ca(OH), + 2NH,Cl—+ CaCl, + 2NH, + 2H,0 (@) 2RIAGCN),) + 2a —s 2Ag + K, nlCN),) For an ideal gas, number of mol per litre in terms of its pressure P, temperature T and gas constant R is @ Pr @PRr ort @RTP. Number of P—O bonds in P,O,9is @i7 16 ow as KO, is used in space and submarines because it (a) absorbs CO, and inereases O, concentration (©) absorbs moisture (e) absorbs CO, (2) produces ozone, Which of the following ions has the maximum mag- netic moment ? @) Ma" Oh on @or. ‘Acetylene does not react with (a) Na (©) ammonical AgNO, HCI @NeOH. ‘Compound A given below is ococn, COOH x (a) antiseptic ©) antibiotic (@) analgesic (a) pesticide, For the following coll with hydrogen cloctrodos at two different pressures p, and p, Pe ii) | H* (aq) | Peet) pM emfis given by BE ig, A BE og, Ph cy Fee or Fpbe rig Bl iog (op Meer ah apie 10. we 12. 18. “a 15, 16. a. Acetylene reacts with hypochlorous acid to form (@) C\CHCHO (@) CCH,COoR (0 0H,c001 (@ acH,CHO, On heating benzylamine with chloroform and ethanolic KOH, the product obtained is, (6) benzyl sleokol (oy benzaldehyde (© benzonitrile (a) benzyl isoryanide, ‘hic othe lowing ronetion in one at anode 7 one sae toate 1 ogee 0 (20, TH+, +10-+ er reste ve, Which of the following concentration factor is affected bby change in temperature? (@) molarity (b) molality (©) mal fraction (d) weight fraction. Cyanide process is used for the extraction of (@) barium {) silver (© boron (a) zine. Following reaction (CH,),CBr + H,0— (CH,),COH + HBr, is an example of @) elimination reaction ©) Sree radial substitution (© nucleophilic substitution @) electrophilic substitution. ‘A metal M forms water soluble MSO, and inert MO. MO in aqueous solution forms insoluble M(OH), salu- ble in NaOH. Metal M is (Be Mg Ca ws. Half ife of a substance A following first order kinetics 105 cays Starting with 100 gof 4, ancunt loft afer 15 days is @2g 508 1258 (62s. ‘The most stable ion is () (RAOED,* (recy © IF«Cm> @) [FeGH,0),*. SIVE CHEMISTRY FOR AIEEE 18. 19. a1. ‘A substance forms zwitter fon. Tt ean have functional groups (@)—NH,, COOH NH, —80, (c)both (a) and (6) G@) none of these. AfFe anc Cr** both are present in group III of quali- tative analysis, then distinction can be made by (@) adéition of NH,OH in presence of NH,Cl when only Fe(OM, is precipitated (©) addition of NH,OH in presence of NH,Cl when COH), and Fe(OH), both are precipitated and on adding Br, water and NaOH, COED, dissolves {c) precipitate of Cx(OH), and Fe(OH), as obtained in (b) fre treated with cone HCI whon only Fo(OH), dissolves (@) (®) and (e) aro correct. In an organic compound of molar mass 108 g mol*C, Hand N atoms are present in 9:1: 3.5 by weight. Molecalar formula ean be @CHN, ©) CHAN CH, @OH,.N, Solubility of Ca(OH), is S mol litre, The solubility product (K,,) under the same condition is @4s @ ast ast as. Heat required to raise the temperature of 1 mol of a substance by 1° ia called (@) specific heat ©) molar heat capacity (©) water equialent (2) specific gravity S-particle is emitted in a radioactive reaction when (@).a proton changes to neutron (6) a neutron changes to proton (e)a neutron changes to electron @ am clectron changes t nextroo, In a mixture of A and B, components show negative deviation when (@) AB interaction is stronger than A—A.and B—B inter- action (6) AB interaction is weaker than A—A and B—B inter- action (@AV,.>0, 8,,>0 @avni=0, 380. >0. Refining of impure copper with zine impurity is to be done by electrolysis using electrons as Cathode Anode (a) pare copper pure zine (@) pare rine pure copper (e) pars copper impure copper (@)_ pare zine impure zine. Aluminium is extracted by the electrolysis of (@) alumine (®) bauxite (c) motten cryolite (@) alumine mized with molten eryolite. 27. , freezing point is ~ 0186°C. Point of the same solution is (@)0.186" (© 1.86" : wet aa ae Fora reaction, A+2B}—» C, rateis given by+ IA) IB], hence the orger ofthe reaction is @s @2 (1 wo. 31. CH,Mgl is an organothetallic compound due to (a) Mg—I bond (©) 0-1 bond (© 0—Mg boad (a) 0-1 bona. 82. Ono of tho following spociog acts as both Bronsted acid and base (@) H,PO, (HPO (© HOY (a) all ofthe above. 83. Hybridisation of the ps ‘atom changes in (a) AIH, changes to AIH|- (6) HO changes to H,O* (ONH, changes toNH{ (dh in all cases. 34. Racomic mixture is frmed by mixing two (a) icomerie compossnds| (c) meso compounds {b) chiral compounds (@) enantiomers with cifral carbon. 85. The number of lone pqirs on Xe in XeF,, XeF,, and XeF,, respectively are | (0)3,2,1 ln246 fe)1,2,3 D 6, 4,2 36, An aqueous solution pf 1M NaCl and 1M HCl is (@) aot a buffer but pH £7 (2) nota buffer but pH >7 (e)a buffer with pH <7] (@) a baffer with pH> 7. 87. Consider following tfo reactions A—> Produat; - 414 oa, tale B—> Produ; - 2B) _ 4, 1) hy and k, aro express and time (see) as (a) soe, M sec (©) sec", M sec" in torms of molarity (mol =) [OM sec, M sec (@ Meee see, AIEEE —2002 PAPER 3 38. RNA contains (@) ibowe sugar and thymine (@) ribore sugar and uracil (e)deoxyribose sugar and uracil (d) deoryritose suger and thymine. 39. For a cell given below, Ag | Ag* |] Cu® | Cu o w Agt +e" Ag, E* Cu + 2e°-—+ Cu, Ev =y E*cellis Gist Oday ely -2 Gy ~ 2x. 40, Based on kinetic theory of gases following laws can be proved (e) Boyle's aw ©) Charles nw (© Avogadro's lew all of these, 41. MnO, is a good oridising agent in different medium changing to ‘MnO-—+ Ma® —+Mn02- —+Mn0, —>Mn,0, changes in oxidation number respectively are : (@)1,3,4,5 6) 5,4, 3,2 05,1,3,4 @)2,6,4,3. ‘HL, + 1, —+ 2H, the differential 40) __ 4 Oy} _ atm de dt at 94 Hal __ 9 ¢ Up), dt WR gy Rae ae @) ce 2lBal __ fla. ¢ 0 at dt dt (q ~ 21a. a) | adt ~~ 2dt dt 43. Number of atoms is 560 g of Fe (atomic mass = 56 g mor”) is (oie twice that of 70 gN G) ishalfthat of 20 git (@) both are correct (@) nene is correct. 44, Geometrical isomerism is not shown by (a)1, tdichloro-L-pentene () 1,2-dichloro-Lpentene (01, Bdichloro2-pmene (1, 4-dishloro2-pentene. 45. Number of atoms in the unit cell of Na (BCC type crys- tal and Mg (FCC type crystal) are respectively @aa 4,2 24 @141 48. Which of the following compounds has incorrect IUPAC nomenclature ? 9 wanerzcrztocat, weayeucnci etiyibulanoete ty Seti! batanl ° @oncnlcnycr, a crycricticty 2 nt ba 2mehytyentanone methyl btanel 47. End product of the following reaction is Cla cH,cH,CooH 5 lesholieKOHt (a) CHjCHCOOH (@) CH,CH,COOH ba a (c) CH,=CHCOOK (d) CH,CHCOOH. 1 OW 48. For the following reaetion in gaseous phase 0+ 40,—+C0, Is @ an oan Rt @err. 49. Energy of H-atom in the ground state is ~ 19.6 ¢ hhence energy in the second excited state is (2)-680 @)-84eV (@-1510V (~ 453 0. 50, A square planar complexis formed by hybridisation of the following atomic orbitals (@) 5.25 2,P, Os. Do Py Pod ©4.s.P5p, QDsP Py Pohd. 51. ‘Type of isomeriam shown by [CrINH,),NO,ICl, is (a) optical (b)onisation () goornetrical (a) tinkage. 52, Oneof the following equilibra is not affected by change ‘in volume of the flask G@) PCIe) arate PCI 5) + Clip) (8) N,@) + 3H,@) <= 2NH) (Ng) + 0,8) et 2NOW) {d) $0,C(g) <=® 806) + Chi) 5s. 87. 61. ‘Uncertainty in position ofa particle of 25 gin spaceis 10+ m, Hence uncertainty in velocity (ms“!)is (Planck's constant A = 6.6 x 10“ Js) @2ix10" 21x 10% (05x 10% 50x10. Consider the following reactions at 1100°C, 1.20 + 0,—+ 200, AG = ~ 480 kJ mal“? IL 22n + 0,—» 2Zn0, AG" = ~ 360 kJ mot Based on these, select correct altemative : ‘@) zine can be oxidised by CO (©) zine oxide can be reduced by carbon (©) both are correct (@) none ia correct. Arosetion is non-spontancous at the freezing point of water but is spontaneous at the boiling point of water then aH as. (a) +v0 +¥e @) -ve ~ve @ -ve +ve @ +v0 -ve. ‘Monomers are converted to polymer by (a) hydrolysis of monomers ©) condensation reaction between motomers (©) protonation of monomers (@) none is correct. Increasing order of bond strength of O,, 0," 0,2- and Bor<, (CH, (0) CH,CH— > (CH, Cf > (CH,,C— (© (CH,,CH— > CH,CH,— > (CH).C— (q) (CH,),C— > CH,CH- > (CH),CH- PCI, and PCl, both ofist ; NCI, oxists bat NCI, dose not exist. Itis dae to (@) lower elecronegativpy of? than N (a) chain isomerism (b) position isomerism ©) conformers (@) stereoisomerism. Conductivity (Seimep's S) is directly proportional to area of the vessel ard the concentration of the solu- tion in it and 1y proportional to the length of nsthntof proportionality is expressed in (@)8 mmol" (0) S'm mol* (8 m* mol* @)S* mt mol A heat engine absorbs heat q, from a source at tem- perature T, and heat q, from a source at temperature ‘AIEEE—2002 PAPER “Ty, Work done is found to be J(q, + 4)- Thi cordance with (@) Grat law of thermodynamics (©) second law of thermodynamics (6) Soules equivalent aw (4) none ofthe above. 70. Select correct statement, (@) when # covalent bond is formed, transfer of electrons takes place (@) pure H,0 does not contain any ion (6) a bond is formed when atiractve forces overcome repal- sive forces (é) HP is less polar chan HBr. 71. Tne metallic sodium dissolves in liquid ammonia to form a deep blue coloured solution. The deep blue col- our is due to formation of (@) solvated electron, ¢ (NH) 73. Sy, reaction is more probable in @ > c+ KoH—> © A+ KoA © Ore 108n— @ Oy ense1+ Kon —> 74. Oxidation number of C1 in CaOCl, (bleaching powder) (@) roro, since it contains Cl, (b)= 1, since it contains Cr (©) +1, sineeit contains C10 (©) solvated atomic sodium, Na(NH,), (@) +1 and—1 since it contains C10" ané Cr () (Nav + Nev) 75. Picric acid is (@NONH, + Hy cooH COOH 72. Maximum dehydration takes place thet of ° ° on J NO, OH ® © OH © © coon ‘coon 9 cH, NO ‘NO, NE, ‘OH NO, iH © @ @ @ ANSWERS L@ 2 @ 3% 4@ & a) a@ 7 © &@) 10. @ i. @ 12. @) 3. ¥ 1%. @ | 16 © i © 6 ©) @ 20. @) Ha) 2) 3 @ | 2% @ 2) 26 7_®, 2 Dd 30.) 30 | 2 © 3. @) 3 EG) 36. @) a) 8. Be | 0 @ a. © 2 a © 4 @) BO 6a) wo | @ 2B. © 50. (@) a. @ 2) ia) a &@) | sO maw 38. (a) we @ oo. @ a. © ao So | a@ = 6 o_@ a ® le) 70. te) 7 @) | 2 ® 73. @ a 0. err 5 a 8. 10. i 12, 13. 14. 15. 16. 17. 18. 19. Solutions (d) Zn is oxidised to Zn” and Ag is reduced to Ag. (d) PV=nRT or Vin =RT/P. (6) Each P atom is directly bonded to four O atoms in P, Oj, thus, total number of P-O bonds is 16. (@ 4KO, + 260, + K, C0, + 50,. (a) Mn** : (Ar) 3d* has five unpaired electrons hence ‘its maximum magnetic moment is highest. (@) Acetylene does not have enough acidic strength so as to react with NaOH. (c) Factual question. (®) It is a type of concentration cell. The emf is given Pay stanede 9, BE tog Pi, ne Fa a, 3 as EE‘ + 7p!" 5, atcathode te) cuacats noc —+ cca» exon | oa Cl, CH-CHO , =H _ Cl, CH—-CH(OH),, (@) Cli, CH, NH, + CHCl, + 3KOH — C,H,NC + KCl» $,0° Itis carbylamine reaction. (@) Oxidation takes place at anode. () Is not feasible, ie., Cr is not oxidised to Cr,0;- under given condition, (@) Among tho given cone factors, molarity changes with temperature. © Factual question. (©) Its a nucleophilie substitution reaction. (@) Bo forms soluble rulphate BeSO,. It reacts with steam HH,0) to form BeO which forms B(OH), in aque- ‘ous solutions. Be(OH), is amphoteric and dissolves in [NaOH to form sodium berrylate. wc =0,(3) where, C = cone. left, Cy= initial cone. nenumber of balflives 1 =100(3) =125¢. (©) It has highest value of stability constant. (©) Zwitter ion can form from both the combinations (—SO,H and -NH,) and also (—COOH and —NiL,). (®) Question is based on facts. Now, a = total time/half life = 2% is 23,4 20. (@) Mol ratio is O°! or 3:4: or 0.75 :1:0.25 ‘The empirical formula LN, nai =2 Mol. Formula is (Cy3,N), or CgEl,No. 21. (a) Ca(OH), <=> Ca? | 20H- Kp = [(Ca®] [OH]? = (6) (25)? = 45°, 22, (6) Faciualquestion. | 28, @) jn——> Jt + 2p 24. (a) Bnergy roleasod in 4B interactions ie morethan that required for overcoming A—A and B—B interactions. 25. (c) Factual question. (@) Cryplite makes molten alumina conducting. 0.612 Fae * 0-186 0.0612". 28, (6) C atom of CH, (d) In NH,’ the angle (POS ae 33. (a) Hybrid state of Al fhanges from sp? to 34. (@) Thi ‘with definition of Racemic mixture, | 38. (a) XeF, (p (ip =2);XeF, Up =D. 86. (a) Mixture solution not a buffer as per definition but due to HCl its peips < 7. . (d) Reaction 1 is zero frder but reaction 2 is Iat order. ‘inode =~ E | (©) O.N. in Mn0,-(+ ; MnO, (+ 6); MnO, (+ 4) and in Mn,0, (+ 3). 42. () This is as per the! 560 48. (6) 560g Fe = = pt of rate law. = 10 mol or 10N, 20gH H jor 20N,. me = 49. 51. £28 (a) It is due toattachment of similar groups to double bonded carbon atom. (c) BOC (8/8 +1 =2). FCC (8/8 + 6/2 = 4). (a) Tt does not have correct numbering of parent chain. o (©) CH,cH,coom —, cH,cH_cooH TOR CH, = CHCOOH. (@) K,=K mn, 4n,=1-15=-05 1s kK, KK aD = Boor . & uz, iv x, “&D , 136 (B= rev For second excited statn =3, E, = 38 1s1ev. (@) For square planar complex, the hybrid state isd,?, (@ NO, being ambidentate ligand can link through N as well as through oxygen. (©) The value of Any, =0. (a) Apply Ax . Ap = A/ax. (®) AGis -ve for 2n0 + C—>Zn + CO. (@) Higher the bond order greater is bond strength. (@) Ibis based on lanthanide contraction. oo pit = Re 5 WES A 7y 297-1 897, 14. 15. (e) Factual question. (6) It is evident from the equation that and log c, are linear functions. cnet cnentex coe cx, HOOCCH, COOH. (a) Thisis to hypereonjugation. (c) Statement is self expanatory. (q) They fallin the category of stereoisomerism, in gen- eral OF 1 (b) 8 = area(m?) «cone (moVm?) ooh (8) 8 = area (m?) (oe 8) aa Se kon? me, k= 8 molt m?, (c) Iti a factual statement, e) Factual statement, (a) It is dueto the formation of ammoniated electrons. (©) C= 0 groupis electron withdrawing hence —CH, ‘group in between —OH and C = 0 is acidic, hence de- hydration is maximum 9 9 H Hy on ~H#:0 (@) Self explanatory answer. (©) It is 2, 4, 6-trinitrophenol. CHEMISTRY ordre Copyrighted material PART-A x SOME BASIC CONCEPTS (Atoms, Molecules and Chemical Arithmetic) STATES OF MATTER 2.1 Gaseous State 2.2 Solid State ATOMIC STRUCTURE SOLUTIONS CHEMICAL ENERGETICS AND THERMODYNAMICS CHEMICAL EQUILIBRIUM REDOX REACTIONS AND ELECTROCHEMISTRY RATES OF CHEMICAL REACTIONS AND CHEMICAL KINETICS SURFACES AND CATALYSIS SIGNIFICANT FIGURES ® (i) ii) (jv) w) wi) (wit) All non-zero digits as well zeros between the non-zero digits are significant. Zeros to the left of the first non-zero digit are not significant. If a number ends in zeros but these zeros are to the right of the decimal point, then these zeros are sig- nificant. Ifa number ends in zero but these zero are not to the right of a decimal point, these zeros may or may not be significant. Itdependshow thenumberisexpressed in the exponential form e.g. 10500 may be written as 1.05 x 10*, 1.050 x 10? or 1.0500 x 10*. These have 3, ‘4and 5 significant figures respectively. The result of addition or subtraction should be reported to the same number of decimal places as that of the term with least number of decimal places. ‘The result of multiplication or division should be reported to the same number of significant figures as, possessed by the least precise term. Ifa calculation involves a number of stepe, the result should contain the same number of significant figures as that of the least precise number, other than the exact numbers. MATTER Anything that occupies space, possesses mass and the presence of which can be felt by any one or mors of our five is called matter. It exists in three states i.e. Solids, Liquids and Gases. ‘The fourth state of matter is known as plasma state and is found only at a high temperature. (CHEMICAL CLASSIFICATION OF MATTER Matter may be heterogeneous (non-uniform com- position) or homogeneous (uniform composition). Homogeneous matter compris fsolutions (non-fixed com- position) and pure substances or (fixed composition). Pure substances are further classified as (@) Elements (cannot be decomposed and (ii) Com- pounds (can be decomposed by chemical reactions). Amixture is formed by mixing two or more elements or compounds in any proportion, It may be homogeneous or heterogeneous. PURIFICATION TECHNIQUES ‘The different methods used are as follows @ Filtration ' (di) Distillation : It may be (@) Simple distillation (6) Fractional distillation (ii) Extraction Gv) Fractional crystallization (0) Gravity separation (v) Magnetic separation (ii) Sublimation (viii) Chromatography. CHEMICAL EQUATION It is a brief representation of a chemical change in terms of symbol and formulae of reactants and products. LAWS OF CHEMICAL COMBINATION * Law of Conservation of Mass. Total mass of reac- tants = Totel mass of products * Law of definite proportions (Joseph Proust 1799) Achemical compound always contains same elements combined together in the same proportion by mass. + Law of multiple proportions (John Dalton) combine with fixed mass of the other bear a simple whole number ratio, ‘* Law of Reciprocal proportions (Richter 1792) ‘When two elements combine separately with fixed mass of the third element, then the ratio of the masses in which they do so is either same or some whole COMPREHENSIVE OBJECTIVE CHEMISTRY number maltiple of the ratio in which they combine with each other ‘+ Gay Lusssi’s law of combining volumes ‘When gases react they do oin terms of volumes which bear simple whole number ratio to one another and to the volumes of the products, if these are also gases, provided all volumes are measured under similar conditions of temperature and pressure, ATOMS, MOLECULES AND THEIR MASSES 1. Atom, Atom is the smallest particle of the element, which may or may not have independent existence. 2 Molecules. Molecule is the smallest particle of the substance (element or compound) which is capable of independent existence. A molecule may be homoatomic (made of same kind of atoms) cr heteroatomie (made of different kinds of atoms). 3. Atomic Mass. Itis the average relative mass of an ‘atom of the element as compared to the mass of C-12 isotope whose mass is taken as 12 units. 4, Molecular Mass. It is average relative mass of a molecule of a substance (element or compound) as compared to the mass of C-12 isotope whose mass is taken as 12 atomicmass unite, 5. Formula Mass. This term is used for ionic substances ‘because ionic compounds do not exist as true malectles. 6. Equivalent Mass (E) It is the number of parts by mass of the element or compound which combines or displaces directly or indirectly 1 part by mass of hydrogen, or 8 parts by ‘mass of oxygen or 36.5 parts by mass of chlorine. ‘The equivalent mass of different substances can be calculated as summarized below : Substances Equivalent Mass i ‘Romie Mase | 5 Blemente Aormie Mase | Molecilar Mi | ele Basictys of acid | Bases | sate oxidant. | Reduetant | SBaaieliy isthe number ofreplacaBle Hatoms parmaleale | of cia. | "Acidity i the number of OH ~ ions furnished by each melecule ofthe base MOLE CONCEPT Itis a basic SI unithused to know number of chemical particles. [tis a mole roel ‘Avogadro number (Na) of particles or 6.02 x 10% paificles irrespective of their nature. ‘1 Mole = Na Pallticles = 6.02 x 10” Particles Calculation of number of moles (n) SOLUTION ponents. In binary solu proportion is called SOLI proportion is called SOL’ (a) When mass (w) ofa substance is given Miss (w @n(atoms) = He (#) (molecules) + M8 @) When volume (») bfa gaseous substance is given _ volume (») *" 92.4 pr 22,400 ml (c) When number — (N) are given ne i (NK) ‘Avogadrp's Number (No) It is a homogeneops niixture of two or more com- mn, the component with smaller "E and the component with Iargor NT. Methods of expressing concentration of solu- fon Wax 100 + Mase Percentage = W2%100 Wy 1000 * Molarity (M) = @onx Van Wa x 1000 + Normality ®) = CEM x Vom Wa x 1000 * Moality (=) = Ginx Wala + Mole fraction of * solute, xp = OTB * Mole fraction of For binary sclution)y 4x5 =1. solvent x, = A TAYE x 10° Wat We + Parte per millida (ppm) = LIMITING REAGENT (LR) reaction and determines t called limiting reagent. The reactant which} completely consumed during the ‘amount of the products formed is ‘ATOMS, MOLECULES AND CHEMICAL ARITHMETIC 5| EMPIRICAL AND MOLECULAR FORMULA. ‘+ Empiricalformula. Itisthe simplestformulaof the ‘compound which gives the atomic ratio of various cloments present in one molecule of the compound. + Molecularformula. Itis actual formula of the com- pound which gives the number of atoms of various elements present in one molecule of the compound. * Relationship between Molecular formula and Empirical formula ‘Molecular Formula = (Empirical formula)n where ne 1,2,3, can becalculatad as n ‘+ Allotropy. The preperty by virtue of which an element existe in more than one erystalline forme which differ in their physical properties, Such forms are called allotropes, ‘+ Atmotysia. ‘This term is coined with separation of gases on the boss of the difference in their rates of dilfsice/fuion. + Atomic Mass Unit (amu). It is quantity of mass equal to.12th part of the mass of C-12 atom. ‘Mathematically, 1 amu = 1/N, g, Na =6.02x 10 ‘+ Avogram, It is reciprocal of Avogadre number. It is elso referred to as 2 amu or 1 Dalton or 1 Aston. ‘+ Barn. The nudear cross section is meansured in bar's bam = 10 m? ‘© Demal unit (D). itis a unit of expressing concentration ‘of solution. One demal unit refers to one mole of s solute Present in 1000 ml of solution at 0°C. + Dulongand Petiterule. This isan empirical rule which states that the product ofatomic mass and specific heat of ‘the element is approx. equal to 6.4 Specific heat x Atomic mass ~ G4 ‘+ Isomorphous Substances. ‘The substances which have ‘same composition of atoms and similar erystal structures are called Isomorphs of each other. This simple gemeralisa- tion is called Mitecherlich's Law. Some examples are : (ORMnO4, KC1O4 (Hf) Zn804 THzO and MgSO, 7H,0. ‘Ttmaybe noted that valenciesof elements forming isomor- phous compounds is same. For example, valencies of Cr, ‘Se and 5 in KyCrO4, KySeO, and KS0, issame. + LeschmidtNumber. Themumber of molectlesin Lem? of gas at S.7.P. Itsnumerical value is 2.687 x 107°, ‘+ Vapour Density (VD). It is the ratio of the mass of certain volume of a gas or vapour to the mast of same vue of hydrogen under similar conditions. _- Mass of V litres of gus VO. © Ffnee of V Htes of ig Mae similar conditions VD. isrelated to molecular mass es MOLECULAR MASS = 2x VAPOUR DENSITY 4 The term atom was introduced by Ostwald while molecule was introduced by Avogadro, © Slunite. The ST units of some common physical quan- tities ore ‘SOME MISCELLANEOUS IMPORTANT POINTS Length «metre (), Mass = kilogram (ig), Tae = second), Temperature = Kerlin (0, Blectric current = empere (W), Amount of substance = moles (ml), Luminous intensity © candela (Ca), Volume = m®, Density = kg m”®, Velocity « ms~', Force = Newton (N), Pressure = Pascal (Pa) or Nua“ ?, Work or Energy =Joules. ‘© Subsidiary units, These arecither multiplesor frections ofthe base unitaey. (@) Dect © 10°, conti = 107%, mil = 1073, micro = 1076, pico = 10712, famta= 107 15, y ane = 10" Fermi «10° ¥ em 210° m (6) Deka = 10°! hecta = 10°, Kilo = 10%, megs = 10°, ‘ga = 10%, tera = 10", peta = 10'*, exa = 10". ‘+ Some useful conversion factors. 1A = 107! Yom = 10°® m1 pm 107! a, 1 tre = 10° ma 1 dn, 1 atm = 760 mm or torr = 101.825 Pa or Nu”, Lbar= 10° Nix“ * = 10" Pa, 1 calorie = 4.184 J, 1 electron velt(@¥) = 1.6022% 10°19 J, Some important formulae relating molarity and other modes of concentration + Molarity and Normality : Molarity x GMMaotute =Normality xGEMnotate + Melarity and Moss ‘% (rames) x10 dootaton Motarity = te satrity ond yin) 3 * Dilution formula: MsVa =MaVa oF NiVi = NaV Forman Miso. Wwe bain dee bane th molarity My and My and volumes; and Vz aremixed, Molarity (0 of resilting solution ia given by M(V, + Va) = MVi + MaVs, + Motarity Qf) and Mole fraction of elute Op) ee 1000 ek (Mp - Ma) Xp +Ma ++ Molatty (m) and Mole fraction of solute =y 1000 Xe. O-xa)MA" 4 6 0. Choose the correct answer from the four alternatives given in each of the fol Which of the following tance ? (a) picometer @lightyear (mirror (@radin. The compounds with identiesl erystal structures and analogous chemical formulae are called (a) allotropes (@)isotones (c) isomers (a isomnorphs. ‘The correctly reported difference of 18.9426 and 8.06 will have significant figures equal to (a) three (four (o)fire (@aix On dividing 0.86 by 16.564, the actual answer it 0.0519198. The correctly reported answer will be (a) 005 0.081 (0.0515 (a)0.08191. Which of the following is correct ? not a unit of length/dis (@)1L=1am* @1L=104am* (9 10L=14m" @)L=1m". The nuclear crose section is measuredin ‘barn’. Abarn stands for (a) 107 4m? @) 107m? (c) 107m? (q)10-* mi, ‘A mixture that can be separated by sublimation is, (a) MgCl; + Nac (®) HgClz + NaCl (c) AgCl + Nach (d) BaCly + Nach. A mixture of sand and napthalene ca. be separated by. (a) Fractional crystallisation (6) Chromatography (cMgavity separotion _(d) Sublimation. Which of the following pairs of compound illustrate Jaw of multiple proportion ? (@) KOH, Cs0H (#20, D20 {@)ethane, benzene (ci, KBr. Which law of chemical combination is illustrated by the following data ? 0.5 of lime stone on heating gave 0.28 g of calcium oxide and 112 ml of CO, at §.T.P. 4. 13. 14. 15. (a) Law of definite proportiobs (©) Gay Lussaeslaw (Law of easervation of mss (@)Law of multiple p - ‘Which of the following bedt explains thelaw of ccnser- vation of mass? (a) No change in mass ved when 2.0 of Mgis ‘heated in vacuum | (©) 12 ¢ofcarbon when burnin exe ofxygen cenmunes only 3.2 g oft to form 44 g of carbon dioxide (©)12 g of carbon when hedted in limited supply of air produces only 20 g ofcatbon jnoncxide (d) A sample of tir on heating does not show any change in mass but volume ine ‘Two samples of sodium dhloride are produced when sodium combines separptely with two isotopes of chlorine Cl and *"Cl.|Which law is illustrated ky the above facts ? @) Law of multiple ps () Law of reciprocal ()Lawof constant volumes} figures illustrate (@) Law of multiple propor (@) Law of conenrvation of} (Law of reciprocal pro (@ Law of combining volum ‘The balancing of chem (@) Law of combining vol Whichof the following sef illustrates law of reciprocal proportions ? (@PCh, Hel, Her (©) Pls, F202, P20 (©) PH, P03, H2S- (@) PH, Pz0p, Hz0. ATOMS, MOLECULES AND CHEMICAL ARITHNETIC 16. 17. a. ‘Which of the following illustrates the law of conserva- tion of mass ? (@) Mixing of 10 ¢ of sulphur and 2 ¢ of sand doesnot show a change in mas (0) The macs of platinum wire before and after heating remains constant (6) 2.2 gof propane and 8 ¢ of oxygen produces 102 x of ‘gaseous mixture (@)2.8 ¢0fCO and 1.5 g ofoxygen gave only 2.24 L of €0; at STP. "The mole percent of oxygen present in gaseous mix. ture containing 14.0 g nitrogen and 32 g oxygen is (@)50 333 ©668 40. ‘The number of moles of oxygen present in one litre of air if its volume content is 21% at ST.P. is (a9.375 x10" wos (0.068 (20.06, For a reaction, A + 2B —> C. ‘The amount of C formed by atarting the reaction with 5 moles of A and 8 moles of B is @5 md (©)16 mot A compound contains 10-2 of phosphorus. If atomic ‘mass of phosphorus is 31, the molecular mass of the compound having one phosphorus atom per molecule @) emo (4 mol. @ar (@aixi0* (si x10! (@aixi0® 12.gof Mg (at. mass 24)react with dilute mineral acid to produce maximum hydrogen equal to (20.5 mol (6) 1.6 mol ss (@)1gaton. ‘What is not correct about 1 mol of ethyne ? (@)Teontains 2 g-atom ofhydrogen (yt contains 1.2% 10? C atoms (oy Tt weighsis 26 g (dt contains 24 g ofcarbon. Which of the following does not have same percentage of carbon as in ethene ? (@)2-Butene (®) Cydohexane (©)Cylohexene (@ 2-Methyl but-2-ene. ‘The correct arrangement of the following in order of increasing ma: 25. 26, 27, 28. 1. nitrogen molecule I. oxygen atom IIL. Lavogram IV. 1% 10"! g-atom of copper. @I>1>m>V @l 21L ‘Gly gases respectively. They will contsin equal num- (>maLbut<30L | (d)exeuy 2.2L. Jet of minjectoe tf 40. Which of the following dobs not occupy a volume of (4) the mass ofall the gases is same 448 Lat 3.7 (the mols of the eee epanore (02 moiety by an grey (c) temperature and pressure ofall the flusks are same 326 Cervera (4) 800 mg of He. (d) temperature, pressure, and masses are same in the 41, Which pair of species will have equal volumes at ake STP? : e 'S0) and 0.5ynol of NO; 34. 161,80, ioioes as ALES ESO Noe 11,80, + 21,0 +210" + 60,2. (©) 6.02% 10 50, moloules qd 1 mole ofl carton ‘Then total number of ions in 20 ml of 0.1 M HS0, (0 6.02 10 0, moleofles ahd 0.1 msle of Ny 502 solitinae . (@ 1 gm-molecle of C04 and of Ca )s0x10 @rssiaxio! 42, Whichof the fillowingpairs contain species with same (361210 (4) 0.1 mol otions volume under similar Fonditions ? 35, Which of the following does not have same number of {a) 9.1% 10" molecules #fCOp ; 9.1 x 10 He atoms atoms? 23 3 (@) 1g ofoxygen and 1 gm-atom of oxynen Woanay” saves stipe ht x i Heaton, (8) 1 gofoxygen and 1gm of ezone (09.1% 10" He atoms ;p.1 x 10 NaCl unite (©) 1g ofhydrogen and 1 gm-atem of hydrogen (@) 9.1x 10 sodium chipride units ;9.1 x 10° 0 atoms. (@) 4 gmofcaleium and 0.1gm-atom of calcium, What is not correct regarding 22 g of COz? 36. 5.6 g of potassium hydroxide contains same number (0 ft occupies always 11 L of volume of K* ions as are present in ‘ (@) 188 got potassium caronaie (21 orrepends to mp of tacben dite (6) 1.38 ¢ of potassium carbonate (Itcontains one gm-atdm of oxygen (6) $00 ml of 0.2.M solution of KOH ( Teeontains 0g of cafbon, (2)74.5 g of potassium chloride. 44, The ratio of the massfs of 20 L of certain gas to the 87. Two flasks of equal capacity contain argon and mass of 20 L of hydr}gen cas at 20°C and 10 atm chlorine gases respectively at room temperature. What is true about them ? (a) Both contain same number of atoms (8) Cl atoms are half of the Ar atoms (©) Caters ere double thenumber of Ar atoms (@) Chlorine molecules are double the number of argon molecules, Which of the following will cocupy a volume of 1.12 L atS.T.P.? (@) 05 mol of nitrogen gas (€)3.01x 10" nitrogen atoms (c) 3.01 x 10” Nz molecules (1 got Hy mas. ‘The volume occupied by 1 mole of helium gas at 76 cm of Hg and 27° Cis pressure is 20, Then (The gasis lighter th (2) 6.02 x 10% mo ‘NE (4 gothetium (0/40 g of calcium oxide (@) 121 gofiodine. C-12 and C-14 isotopps are found as 98% and 2% respectively in any sdmple. Then, the number C-14 atoms in 12 gof the ‘will be ATOMS, MOLECULES AND CHEMICAL ARITHMETIC {a)1.5 moles atoms (6) 1092 x 10 atoms (0) 2.06% 102" atoms (2) 2¢ atom. 47. Which of the following will not have a mass of 10g? (@) 0.1 mol of CaCO (6) 15110" Ca?* ions (0.016 mel of CO: (d) 7525 x 10 Bratom. 48, IfNq is Avogadro number, then number of O atoms in one gram-equivalent of oxygen is om wt woMs woamy ‘The correct relationship between molecular mass (M) and vapour density (VD) is (a VD= 2M ® vem? v= @M~vp)'4. 2 50, If specific heat of certai approximate atomic mass (a) 100 @10 (20 (&) 200. 51, . In thereaction HyPO, + NaOH — NaligPO, +120 the equivalent mass of sulphuric acid is (98, was (©) 196 yas. 52, Vapour density of 2 metal chloride is 77. The equivalent mass is 3. Its atomic mass would be element is 0.064. Its 3 w6 9 aie. 53, The equivalent mass of H;PO, in the reaction given below is (HgP04 + NaOH —+ NalgP 0, + Hg0) caves 98 (1326 (a) 40, 54, The molecular formula of certain compound is M,Oy. If 18.88 g of the compound contains 10 g of M, the atomie mens of M is approximately (40g ) 54g are Ww) i2g. 55. Ifmolecules mass of KMn04 is M and it is converted to K,Mn0, in chemical reaction. The equivalent mass of KMnO, will be @M os 7. a1. fe Ms (on. 200 ml of given sample of hydrogen peroxide solution on analysis is found to contain 136 mg of Hy02. The molarity of the solution is (a) NiS0 Ooms oom (20M, The molar concentration of CI ions in a solution obtoined by mixing 300 ml of 0.3 M NaCl and 200 ml of 0.4 MBaCl, is (a) 09M LEM (025M (05M, Which one of the following solution of sulphuric acid will exactly neutralise 25 ml of 0.2 M NaOH solution ? (a) 12.8 mi of 0.1 M solution (©) 25 ml 00.2 M solution (e) 25 ml of0.1M solution (d) 50 ml of 0.2 M solution. Themass of 60% HC! required for the neutralisation of 10 Lof 0.1 M KOH is e088 @r192 10% @r9s. 30 ml of an acid is neutralized by 15 ml, of 0.2 N alkali. The strength of the acid is @)01N @o2Nn oan (oan. How much cone. HNOs, (ep. gravity 1.42 and contain. ing 70% HINOs), is needed to make a litre of normal solution? (a) 63 mil © tom (028ml (42m. £NHg+y 0, >2NO+wH0. Ifxis 2, then y, z, w are respectively )5,4,6 25.2.3, 93,24 2.4.6 Which of the following satisfy the requirements of chemical equation ? (@) 2au-+9H2—+ 2AuHiy ()H,8 +2Br—2HBr + 8 (6) BNHg + 8Cly —s GNHAC1 + No (e) 2NHg + 31, —+ NH . Nig + 3H. For the reaction given below, MnO," + ¢,02> + H*—> Mn" + CO2 + Heo, 10 n. n. ‘The correct coefficients of the reactants for the balanced reaction are : Mnog' G,0, «HY @ 2 5 1B 6) 16 5 2 © 6 16 2 @® 2 1 5. 100 kg of iron ore (Fez0,) containing 20% impuriti ‘on reduction with CO give iron equal to @ 112k 80 kg (©) 160 kg (a) 56 kg. ‘NO reacts with Op to form NO2. When 10 g of NO; is, formed during the reaction, the mass of Op consumed is (a) 172g 60g 343g sg. 27 gof aluminium will react completely with (a) 8g of exygen (8) 24 gofoxvgen (€) 16 gof oxygen (c) 48 g of oxygen. If given sample of potassium chlorate is 80% pure, then 48 g of oxygen would be produced from (a) 98.0 gofthe sample (6) 153.12 gof sample (ec) 245 gefsample (a) 1225 gofsample. 276 g of silver carbonate on being strongly heated yields a residue weighing 2168 309 i628 (248g, Assuming that petrol is iso-octane (CyHyq) and density 0.8 ¢ ml” ', 1.425 litre of petrol on complete ‘combustion will consume oxygen (a) 50% e125 (09325 mol (50 mal. The conversion of oxyzento ozoneoccurs ta the extent of 15% only. The mass of ozone that can be prepared from 67.2 L of oxygen at N.TP. will be (a) 44g 96g 60g (a) 84g, 2.4 g of pure Mg (at. mass 24) is dropped in 100 ml of 1 MHC. Which of the following statement is wrong? (ay1.12 Lot hydrogen is produced at S.7.P. (6)0.01 mol of magnesium is leftbehind (©) 0.1 mol of CI” ions are formed in solution (@) FCA is the limiting reagent. (@ 10% 28 288 be. 74. 16g of Sp was burnt completely to form SO, which is with BaClzolution. The: is (@)0.25 mol (04 mol 75. 0.6 mol of barium chlori (0.1 mol 04 mol 76. Tha number of moles of moles of Ky Hg, by (04 required to produce 0.4 jon with HgClz is 7. ratio of 1: 35.5 by mass. required to react with 2 wre one The number of moles of| ‘quired to be decomposed (@)3 met (©0.16me ‘The minimum amount of 6.35 g of Cu ions from (ese oe If atomic mass of molecular mass of water: (06.25 (e202 ‘The number of neatrons| proximately be (a) 4.216 x 1079 (€)9.682 x 107° Ifthe charge on electron is (on 0.1 mol of copper IT io (2F (o1F 78. 1. ATOMS, MOLECULES AND CHEMICAL ARITHMETIC 11 ‘83. On reduction with hydrogen 3.6 g of anoxide of metal leaves 3.2 g of metallic residue. If the atomic mass of metal is 64, the formula of metal oxide is, MDs (m0 (MO (M02, Among the pairs of compounds given below which pair contains compounds with different percentage of carbon ? Level II ‘Themole fraction of solute in 1 molal aqueous solution is (@oo176 ws (©0085 (oss. Choose the correct answer from the four alternatives given in each of the following questions : 1, 10g ofa sample of silver which is contaminated with tilver sulphide produced 11.2 ml. of hydrogen aul- phide at S.T.P. by treatment with excess of hydrochloric acid. The mass of silver sulphide in the sample is (Ag= 108 : S = 32) (1g (6) 124 mg (95 x107? mol (a) 62mg. 2 Dehydration of sucrose, C1gH,20;1 by concentrated HySO, gives purest form of carben. The amount of cearbon which can be obtained from 34.2 g of sucrose is (44g (6) 12 g-atom (09.2 ¢-atoms (d) 16.4 gatom. 3. 142 gof chlorine represents (@)' Two g-molecules of chlorine (0)4 molesof chlorine atoms (©) Two moles of C1 atoms (a) Both a, 6, 4. 20 g of a tri-atomic gaseous element was found to ‘occupy a volume of 448 ml at 76 cm of Hg and 273 K ‘The mass of its atom is (@)80amu (6) 583x10° g (938g (4) 553 amu. 5. 5 1Lof0.1 M solution of sodium carbonate contains (0) 53 g of NayCOy (6) 5 x10" milli moles of NagCOs, (0) 106 g of Naco;, (4) Both a, 6 1% 10. aL 12. 13. Total number of g-molecules of SO,Clz in 13.5 g of sulphuryl chloride is (@o1 wo (03 (os. ‘Total number of protons in 10 g of caldum carbonate is (a) 18707 10% (6) 2.0478 x 10 (01x10 (a) 4.0956 x 10%. 2 gof oxygen contains number of atoms equal to that in °. (2) 05 got hydrogen ()4 gof sulphur © Tg ofitrogen (23 got sodium. The number of moles of sodium exide present in 620 gofit are (@) 1mol 10 mel © 18ma (@ 100 ma. 8.5 g of CO gas at 0°C and 760 mm pressure contains molecules equal to (a) 35 x6.02x 10" (8) 28 x6.02% 10 (©) 7525 10" (a) 1.25% 10%, ‘Thenumberof moles of helium that occupies 2.24 litre at 0°C and 1 atm pressure is (@10 oa ow 1x10", ‘Thenymber of molecules in 8.96 Lof agas at 0*C and 1 ‘atm pressure is approximately (@) 60210" (0) 12.08 x 107% (©) 18.06x 10% (4) 24.08 x 10, In the reaction, 2Nay8103 + Iy—+ Nag84Og +2Nel, 12 COMPREHENSIVE OBJECTIVE CHEMISTRY| 4 15. i. 18. 19. 1. ‘the equivalent mas ‘of NapSz03 (Mol. mass = M)ix mM om wt M mM o% we. ‘The number of atomsis largest in (@) Tigofehtorine ©) 48 gefmognesium (©) 127 g of iodine (d) 4g of hydrogen, ‘The number of valence electrons in 4.2 g ofnitzide ion ‘is (Ng is Avogadro number) @24Ny )42Ny 16N, (32%. KCr07 + xH,S0, +ySOz > K,S0, + Cry(S04), + 220 ‘The valucs of, y, = are (1.3.1 O44 €)3,2,3 (d) 2,1, 2. ‘The hydrogen phosphate of certain metal has the formula MHPO,, The formula of metal chloride is (Mer Mer, © MrCly Mel, ‘The number of moles of oxygen in one litre of air (21% oxygen by volume) at S.T-P. would be © (2) 0.186 mel (@)0.21 mal 2.10 mot (2) 0.0088 mal. ‘The number of atoms in 4.25 g of NH, is approx. G@) 1x10" 45 10% te 2x108 (a 6x 10%, 0.01 mol of iodoform (CHI,) reacts with Ag powder to produce a gas whose volume at N-T.P. is () 224 ml @) 112 mt 396 mi (2) 168 mi. The number of moles of NaCl in 3 L of 3 M NaCl solution is, @1 ws os ws. Which one of the following has largest number of molecules ? (a) 84g ofwater (©) 46g ofmethana, (8) 28 gof carbon dioxide (a) 34 got NOs. Volume of certain gas at N.T-P. is 1.12x 10" ec. The number of molecules that it has is 26. 34. 32. (@) 3.0110" 3.01 x10! (©)30.1%10% Ge gor 10%, ‘The maximum amount on mixing equal volui }BuSO, thot can precipitate of 05M BaCl, and 1 BM H)$0, solutions is | (05m 01m (©) 1.8 mol (20 mal. When 40 g of iron is byfnt at high tomporature to ferric oxide at constant pressure 29.28 ki of heat is ‘evolved. The amount of hdat evolved for producing one mole of ferric oxide is (a) 819.84 kJ (8) 40.99 kd (e) 81.96 kT (d) 409.9 kJ. ‘One mole of GOp con (@) 2 gratoms of C02 (@) 18.1 10% molecules of} (€)6.02x 10 O atoms (@) 8.02% 10 C atoms. ‘The mass of AgNOs (Molj mass 170) present in 100 mi of its 0.25 M solution is (a) 4.256 (0/4258 ite @hitg. ‘The molality of a solutign containing 5 g of sodium hydroxide in 250 g of solption (@)05m 1.0m (20m (ch 0.1m. Anorganic compound with C= 40% and H = 6.7% will have the empirical form@la @CH (®) CH,O (© CyHO (e) CaH4 0». If 100 mil of 1 N-H,SO, 4s mixed with 100 ml of 1 M NaOH solution. The resting solution will be a) highly acidic () pectral (e)bighly basic (2) slightly acidic ‘The normality of 0.3 M fhosphorus acid (HPO) is, (a1 wos (03 os On rendering anhydroup, the sample of hexabydrate calcium (If) chloride Jos (029.4% of ts mass (c)46.5% of its mass (8) 49.3% of its mass (c)48.9% of its mass, ATOMS, MOLECULES AND CHEMICAL ARITHMETIC 13 38. Which of the following seriesof compounds have same ‘mass percentage of carbon ? (@) C02,C0 (0) C4, CoH, CoH (0) CyHy Ogle, Clg (@) HCHO, CHyCOOH, CgH120g. 82.5 g of zinc (at. mass 65) will completely react with dilute sulphuric acid to gi (@) 1 mol of Hy (8) V2 mol of Hy (oF roto (obin 2 md tO nde, 2240 ml of Hz at N.T-P. will be given by reaction of (@) 46 gofNe with excess of water (0) 65 gof Zn with 5 gof HCL (©) 6.35 g of copper with excess of dil, HCL (@)all the above methods, 36, ‘The volume occupied by 0.04g-molecule ofnitricoride at 0°C snd 760 torr is (@) 896 ml 1000 mi ©) BA litre (dh unpredictabie, 37. What is not correct about 1 mol of methane ? (a) It contains 6.02 x 10% atoms of kydrogen (Oy contains 12 g of carbon (©) Keontains 6.02 x 10% motecul (a) W ocenpies 22.61 at STP. ‘The molarity of the solution containing 2.8% mass- volume solution of KOH is M oF Me @% M ot @iM, 90, avelumes of 80, react with 1 volume of oxygen under similar conditions. The equivalent mass of SO, is (a) 64 ()32 C6 (d) unpredictable. 40. Assuming complete dissociation of Hy$0q a8 (41,804 +2110 —+ 21140* + 80} ‘The number of H30* ions in 10 ml of 0.6 N H,SO, ia 41. 47. (@)3.01« 10%! ) 15 x 102 (6.02% 102 (@) 301x10% 33.6 L of water vapour at S.7.P. are condensed to liquid state. The volume occupied by it is ap- proximately @1L © mi 2m (e127 mi. In the reaction NaOH + AKOH)y —+ NaAl0; + H,0, ‘the equivalent mass of Al(OH); is om @ 28 (esa (2) unpredictable ‘The volume of 0.5 M HyS0, required for complete neutralisation of 10 mi of 0.6 M NaOH is: @ 20m © 10m (5.0m1 75 mt ‘Tho molar concentration of Na* ion in mixture of 100 1m] of 2 M NaCl and 200 ml of 8 M Na,SO, is @4serM @ss7M @2emM @istM. ‘The mole fraction of water in a solution containing 117 sodium chloride and 900 g of water is (@ 0.0682 0088 (0.9615 (d) 1.000. 10 ml of 0.2.N HCl is added to 10 ml of 0.1 M H2S0, and 10 m! of 0.4 N Ba(OH)g solution. If Ba(OH), is completely ionised, the resulting solution will be G@) neutral ©) strongly acidic ()alkaline (4 slighty actie. ‘The density of chlorine relative to air is @s 0246 4 (unpredictable. ‘The mass of ammonium dichromate that contains 42 atoms of oxygen is @ 1.512% 10% amu ©) 1512x210" g © 252ame (1252 mg. 27 g of Al will completely react with (@) 3 mol ofoxygen (©) 8 gatoms ofoxygen (©)8 geequivalents ofoxveen (3 etomsef oxygen. ‘The HS required to precipitate all the Ca ions as cadmium sulphide from 500 ml of 0.2 M solution of codmium chloride is aa You have either reached a page that is unavailable for viewing or reached your viewing limit for this book. ‘ATOMS, MOLECULES AND CHEMICAL ARITHMETIC 15} n. In a reaction, 3Fe + 4H,0 —> Feg0, + 4H, the equivalent mass of iron (atomic mass = 56)is ar ws (112 wa. During synthesis of ammonia from its constituent elements, the volume of the product relative to the total volume of the reacting species is (@onefourth (©) threefourth @onehalf @) one-eighth. ‘Which of the folowing contains largest numberof toms?” (1 goon ©) 1g of stone (1 gofowgon @ 1g othaliom. {50 ml of samples of distilled water, ordinary tap water and boiled tap water requires 2 ml, 26 ml and 10 ml of soap solution respectively to form permanent lather. The ratio of permanent hardness to temporary hardness in tap water is 1". @) 7M (16M osm 5M. 80 gof acompound on analysis gave 24 gof O, 4 g of H and 32 g of O. The empirical formula of the com- pound is, (a) GH, ® cH,0, (0) CyH202 (a) CH:0. ‘Two oxides of a metal contain 50% and 40% of metal (M) respectively. If the formula of first oxide is MO, the formala of 2nd oxide will be (a) MOz ()Mg03 M0 (@) My0s, 4 g of hydrogen is ignited with 4 g of cxygen, the amount of water formed is @25g @osg 458 ts A solution contsining 0.1 mol of a metal chloride eure baka MCI, requires 500 mol of 0.8 M AgNOy solution for faa ar. complete precipitation. The value of xis 300 ml of 3.0 M NaCl is added to 200 mi of 4.0 M ot oe BaCl; solution. The concentration of CI” ions in the aa oe resulting solution is ANSW QUESTION BANK (Level 1) L@ 2@ 38 40 5. @) 0 10) 2@ © me me Pac) 1 @ wo 18 60 © Be we 20. a. @ 2 2.6 1 © 2 @) R~ Me 2. 0) 2. 0) a. & a. ® 2 © 30 wb eo 38. © a 0 2. © 0. @ a@ 2&0 &@ 4 6) 5 6) 6 ® a 4. © 20 80 Le 2 @ 5. 0) 54 55. 5. @) 2) 6h 6 0. @) a. @ a. @) 03. @ 4 () 65.6 6. 6) 69. @ m0. © n © 2. Ro %0 % @ n. ©) © 2. ® 0. © 0 &@ 80) 4.) 5. @. QUESTION BANK (Level 11) 16 2@ 2 2® 46 5 @ 20 Le 2o 6 mo ne 2 18. @ uo 6 @ © © 18. 0) 20. 6) 2. ©) 2 ©) 23. ® uo © %*6 me 2.) 28. 0) 3 & a1. @) 2 ® 16 eae ie 10. nn 2 13. 37. 45. 53. a. 9. 01 ‘OBJECTIVE CHEMISTRY] @ |39. &) 4. @) © 47. ® 48. (@) ® 55. (a) 56. ® 88. (@) 64 © 10. n. @ 2. 33.) 4.) 8B. @) 36. (@) © &@ 4 ©) 44. @) © 6.) 51. 0) 52 ©) 87. @) 6B). GB) £0. (©) 6. © 6) 8 @) 68. @) BO 16) © 7%. ©) (@) radian ie a unit of ionising energy that is absorbed. @ isomorphs have same crystal structure. (@) The difference between (18.2426 - 8,06) is 10.2826. Figure 8.06 hes least number of decimal places vis. 2, therefore, reported difference 10.2826 has four significant ‘figures. (2)0.98 « 16,564 =0.05101. The least precies tarm involved thas two significant figures viz, 0.86 :. reported answer should be 0.051. (@) 1.1 = 1000 em = 1000 (0.1 den)? = 1 dm’, © Lbarn = 10°28 2 (®) The compound HgClz sublimes whereas NaCl does not. (@) Out of sand and napthalene since napthalene can sub- lime therefore the mixture can be separated by sublime- tion, (© In ethane and benzene, the elements are carbon and hydrogen. c oH ethane = 2G Benzene 6 6 ‘The mass ofcarbon combining with fixed mass of Hydrogen in these compounds in simple ratio of 2:6 or 1:3. (CaCO —+ COz + CaO ose 2ml 0285 atSTP ‘2 ml of COg has amass = Ae x 112 =0.22¢ “ Ba00 or 054 =0.222 +0288 (#) Only in} the mass of products isequal to that ofreactants. (@) Question based on statement. (o) In HO, H = 11.1% and 0= 889% ratio of H:0 = 1:8, 1“ 15. 16, rca 18, 1. a1. ~04,1%. Now 8 parts by mass of 59 1 spounds = 1: 0.501 =2: 1,0 In Hz0z, H=5.9% and (© will combine with H= 52 jes = 0.501 Hence ratio of H in two ‘simple ratio. (©) General principle. (2) See definition given. (©) Calg + 912 Op —>i ‘Mass of reactants = 2.2 ‘Mase of products = 10.2 to Mote etX= foo shorteummary. +3H,0 8= 102g j moles of oxygen = $2 = 1 2 Mots pacentef oxygen 4212 w 66.68, (@) Volume of oxygen in 4 litre (1000 ml) = 210 ml. ‘Moles of oxygen = 9.976 x 10°, @a+B—C Hero, 8 mol of Bisa liming reagent 8 moles of B wil (6) Ifmass of P is 10°* g)masg of compound is 100 ¢ ; 100%91 ‘fsanss of Pin 31g, mas of compound ia 10? =a1xfo4, (@) Mg + THC MgCl, + Hy me @gort mod ‘Thus, 124 of ma produces Hy = 1 of 0.5mal. (©) Bthyne is HC = CH 1.2% 10% Caton. 2-tatene (C4Hg) and 2-Methyl ATOMS, MOLECULES AND CHEMICAL ARITHMETIC 17] ‘24 (1. Mass of nitrogen molecalo =4.66x 107 g TIL. 1 Avogram = 1.56 x10-™* g TV. 1x 10°™® g atom of copper x68526.3%10" g 25. (a) 11.1 g of CaCl = 0.1 mol of CaCl and has 0.3 mol of Sons (0.1 mo! Ge? ancl 0:2 mol C1, (@) 1 mole of NegCO, . 101;0 has 0 atome = 19% Ny 0.2 mel of NayCOs . 10H0 has Oatome mons 8.02% 108 4 156% 10%, 21. (@ NEANOg; 80g of NH{NO, containe N= 28 ¢ 11108 g of NINO, contains N t 28x16" = =035%10%g or 3508. 28 (6)6.02x 10"5 C atoms =1 gm-otom of C which is present in. I mol of CO, ie, 28 g of CO. 28. (t) PHy mdlecules =®: 30. (6) In each formula unit ratioof AlS* : P~ is 1:9 ata} xa x 10% e110", SL. (2) The ratio of mass of CHy : 80g is 16: 64 ie. 1:4. Hence the ratio ofmolecules will be 1:2. The moler ratio is same ‘as themolecalar ratio. (@) It is based upon Avogedrels law. (@ Statement is based upon Avogadro's law. (8) Moles of HyS04 = 20% 0.1 10 =2 x 10° Moles of ions = 3x 2x 10° mol No. ofions = 6 x 10°! x 6.00 x 10% = 3612 1077, 85. (a) Number of © atoms in 1 g of oxygen and 1 gm-atom 6 @) of oxygen are different. Re8 23 36 (@ Brin 66 gon - $02* 10X58, 6.02 1022 ‘Same number of K* will be present in 600 ml of0.2M KOH. 37. (@ Assording to Avogadro's law, the number of Ar and lz molecules must be equal. Since each Cly molecules has 2atoms. Therefore, number ofClatomsmust be double the number of Ar atoms. 5 ' a. 48, aL. (©) 05 mol Np=05x224LatSTP.= 1121 8.010 N atoms = 15x 102! Ny molecal 15 x¢102 5 29. 6.02% 10 8.01% 10% Ny mol» 202X122 9, 6.02 x 10 1eorth ewe 22 enna = 0056L () Volume at 27°C (1 atm. pressure) > volume at OC (1 atm. pressure), .e., greater than 22.4 L. (d) Bach of the species in o, b, ¢ represents 0.1 mol and ‘occupy volume of 4.48 L at S.T.P. (d) 1gm-molecule of CO, and 71 gofchlorine (1g-melecuie) both will have same volume at N-T.P. Le.,22.4L. (@) Both 002 and He have samenumber of molecules and hence same valume as per Avogadro Law. (i *00,= 22 «08mal or @gefeibn (®) VD. of gas = 20 ‘The gas is thus, heavier than NH (VD = 17) but lighter ‘than 802 (VD = 32) and Cly (VD =35.0. @) 1 g-molectle means 1 mole ofa substance. Bx 100 6.02 x10" «0.24 7 (€)0.1 mole CaCO; = 0.1 x 100 =10 5 51 x 10 x40 602x107 0.016 mol CO?” = 0.016 x 60 = 0.96 ¢ ©) Amount of C-14is 12g n0.268 ‘Number of C-14 atoms =: 151% 108 ox? =10.08 1.525 x 10 x0 6.0210 (b) 32 g 02 contains O atoms = 2Nq 1 g eq. of Og = (8 g) contains O atoms 2NAx8 Na Bz 2 (a) According to Dulong’s and Petit’s rule's " 64 Atomic mass x specificheat~ 6.4. or At, mass = >°S> (a) In this reaction, H 80, exhibits basicity =: E=Mol.mass (@) Let formula of metal chloride is MCl Molecaler mass of MCI, = (Ex<+#x 35.5) = (x +35.50) or (85543 =2xVD. =2%70 = 140 7525 x10 Br =100¢, B 40 or xn tied s Mone mise =Bxee = 3x4 12 63, (6) Basicty of iq in the given reaction = 1 Bquivalent mone = MELIBAM op 54. (0)Lat the atomic mam be '. 4m parts by mass of M are present in (4m + 96) parts ty mace ofcompound, 4m+ 96 20 pate mac eM are proneat i ‘by mags of compound. Now M2010. 1556 or matt 55. (a) In the reaction KMnOs—+ Kz MnO, ‘The O.No. of Mn changes from + 7 in KhinO, to + 6 in Ky MnO,, therefore Eq, mass = M Weg) 1000 ‘GMM Vent 87, (a) [NaCI = [C1 ions in resulting selution 300 x0.3 = 20008 20.18M 0 pat 56. (a) Apply the formula M = 200 0.4 [BaClg} in the resulting solution =: 16M *. {C11 from BoC, = 2 x [BaCh] = 2x 0.16 032M ‘Total concentration of CI” ions = 0.32 + 0.18 = 0.50 M. 68. (a) Moles of NaOH = 25 x 02x 10 +5 x10~ Moles of p80, required «x61 «25107 which pertains to 26 ml of 0. M,80¢ sli, 0, (@)Mole of KOH= 10001 =1ma KOH + HC1— Kol + #0 Pre HCl reaird for L mol of KOH= 265 comtictreqired= 12x 008-008. 60, (a) Ny V)= Nz Vz Boxny=15x02 « Ny=45¥92s01N, (a) Moasity ot HNOy 22222225. 77714 ‘To prepare 1 Lof 1N solution, HNOs required = pay 0.06981 or 633m! =63 ml. 62, (b) The balanced equation is 4NHg + 502—> NO + 61,0. equation. ‘Thus, (a) sud (6) do npt satisfy essentials of chemical equation, i} ‘Moles of HC1 = 100 x} x 10° = 0.1 mot ATONS, MOLECULES AND CHEMICAL ARITHMETIC Motesotity 24-01 mel From the equation, HCIL. Reagent and moles of magnesium left =0.1-0.05 = 0.05, 73. (a) 60 mlof05 MCa(OH) = 0.096 mel of Ca(OH), ‘Theeqn.is Ca(OK)g +COg—+ C2005 + 0 1 mol of Ca(OH)y gives CaCOy = 100 ¢ 0,025 mel of Ca(OH)g gives CaCOs = 100x 0.025 =2.5 ¢ +}0, BaClz 14. (0) 184 +02——+ $0; —+ $03 —+ Ba804 {82 gofrulphur prodice BaSO, = 1 mol 16 g of uiphur produce BaS0, =0.5 mol. 76. (a) The equation is ‘S3BaCl + 2NaxPO4 —+ Bag(OP4); + 6NaCl ‘Limiting reagent is 0.2 mol sodium phosphate, 16. (d) IKI + HgCly —> KC1 + Help 2KI + Hela — Kael, For producing 1 mol of KzHgl,, KT required = 4 mel For producing 0.4 mol of KpHely, ‘Ki required = 4x 0.4 = 1.6 mol. ‘72. (€) This is in accordance with law of definite proportions 78. (a) 2PbINO3)—+ 2Pb0 +4NO2 + Op 2mol aug Load nitrate required for producing 48 g of Oz 2x48 248 «3 ma 79. (6) Theionie equation is HS + cu —s cus 42H" Me 655 pt 80. (6) atomie mass ofO is 16 mol, mass ef HO = 18 [atomic mass of is 100 mol. mass of HzO 8 x 100 BAM suze. ()Noutrons in 1 molecule of HO = 16 Neutronsis 18 g of water Lexcoo x16 9 1% 82, (@) Lmol of Cu* ions hascharge = that on 2 mal of electrons = 2F 21023, 0.1 mol of Cu has charge = 2x 0.1 F 20.2P. 83. (6) Mass of oxygen which gets displaced from metal oxide 204g (Now, 04 g of oxygen combines with metal =3.2 g +: GEW ofmetal = 64 ¢ rl meta] = GAM , 84 Velency of metal = Gory = pres) Hence, formula of oxide is Mz0. 84. (a) Thecompounds whoseempirical formulae are different will have diferent percentage compositions. 1900, 85, (@) Moles ofsolute = 1 ; Moles of solvent = 05.5 se 1 Mole fraction of solute = <2 = 0.0176. 1. ©) AgyS+2HC1—+ HyS+ 2Ag0 2405 24 11.2 of HyS is produced from Ags 248x112 = MBI ores. Now 0.194 got Ag =124 mg = 2324 05 10~ moe 2 (a) 342 gor CigHy201; gives C = 144 g 94.2 gof CisHyi011 give C= 144g 3 (@) M2 gof Cl = 142/11 =2 mol of Cly Itisalso equal to four mole of Cl atoms. 4. (6) Mass 22400 ml of gas =: 100 anasto ia 2 Aton masse 120383 Py 6.02 x 10% 553x107 g. 6. (d) AmuntotNog003 in 0.1 M of Leon $0.1 x6 105 = 53g Mass of ne atom = 33:3 amu = tay =5x 107, = 5 1000 munmotes = 5107, 8. 10. 12. 13, re 16. rt (©) The number of protons in one CaCO3 unit =204643x8=5) ‘Number of protons in 10 g of CaCOg _ 10x 50x 6.02 x 10% 24, Se =3.01x10%, 2xNy (0)0 atoms in2 goronygen = 44(= zeae Jsteipharabocontaine stom = 0.125 Na. (&) Mole of Nag0 in 620, (@ The number of molecules in 3.8 gof CO 2 8:5x6.02 x10 1.25 x 1022, B (b) Moles in 2.24 litra of helium at 0°C and 1 atm. (d) No, of molecules in 8.96 L of the ga =8%8 600% «10% 898 6021108 «2408.10 tSTP, 42-2 eae (a) In this recction, 28;03———+> 8,0", ‘Since one molecule of NazS203 loose 1 electron for ‘undergoing oxidation. Z “Equivalent mass = Mom, 04-2 10 No. ofatoms =2Nq Mes Bazpol ——« Noafetomne2Nq Tete 22 mols. Neoftom = 2 4 Neofatoas = Hy=4=2mol, ., No. of atoms = 4Nq Hence 4 ¢ of kydrogen has largest number of atoms, (@) 42 g of N-* ions has valence electrons Nqx42 Se = 24 Na. () KgCry07 + Hy804 + 8802 —+ KS + 6ry(804g + 1,0 Hience the correct coefficients ofHgS04, 803 and Hg0 are 1,8, 1 respectively. (®) From the formula it is clear that the valency of metal 1s. a1. 20 ‘CONPRI & (a) Gmoecne - 258,185.51. 18 @ 1 Lofeir contains daygeh =0.21L (@) Gmolecnle= Grind ~ a6 ~° Now number of moles fraxygen 45x 104 (6) 2CHIg + 6Ag—+ GAgl + CH=CH) anal mat ‘hee preted enf.01 a efiodetrm ab STP. 70804 + Hy 65¢ Lmal 2 825g $ret 85, (a) 2Na + 2H0—» 2NaQH + Hy 2200 ml Hy is givenby Na = 48 g 2200 ml Hy ia given by Now 45 g. 36. (0)0.04moles of NO occupy velume at N.TP. = 22400 x 0.04 m= 896 ml 87. (a) 1 mol of CH, contains 4 x 6.02% 10° Hatom. Hence, Tatatatement io wrong. (Oy Molarity » 28 12 «Ly, 5 (8) 2802 + 02+ 503 8 g0f Op will combine with 902 =!28 xg <92¢ » epivalnt mae 6903 = 2. 40. (a) Moles of H280, in 10 ml of 0.5 N Hy804 nt0x05xd x10 =25% 0% H,O* ione « 22.6 x10 = 5x 10°S moles = 6x10 x 6.02x 10 =3.01 x 10%, 41, (©) Mass of 33.6 L of water vapour at STP aq TE Now density of water~ 1 g mI" + Volume of 27 gofliquid water «27 ml. 42, (a) The amount of AMOH)s which reacts with 1 gm- equivalent of NaOH (Le. 40 g of NaOH) = 78 g + Equivalent mass of AIOH}s here, is = 78. 4B, (©) 2NAOH + Hy804—+ Nag90, + 24,0 05x10 05xV 2 T or Vea1xde5 ml a. a. (a) Na® ions in 100 ml of 2 M NaC) eclution. = 100x2x 10 =0.2 mol Na? ions in 200 ml of 3M No,SO, solution. =200x3 x10" x2 1.2 mol Total volume = 300 ml Moles of Na* in $00 m= 0.2412 = 1.4 moles Molar concentration of Na* ions = 24 x 1000 = 4.67 M. ©) Motes oe NaCing) = 22 me, 18 Moles of water (np) = 220 = 50 Motetacion otweter= 2 2098515 (a) Miti-equivalenta of HCl = 10x 0.2 = 2 Mill-equivaleats of H,$0, = 100.1% 2 =2 Ce normality of Hy804 =0.1%2) Milliequivalents of Ba(OH), = 10 x04 = 4 Thu, total mili-equivalents of acids (2 + 2) are equal to those of eal (4) therefore, the solution will be nevtral. ®) VD. of Clzrelative to sir $55 2.46. 1a (@) (NEQCr207 252 emu conteins oxygen atoms = 7 Amount of ammonium dichromate that contains 42atoms ofoxygen 252 x42 mae = 1512 amuor 1512 108 ama te) 4Al +302—> 2Al;03 f1 got Aireutawih Ope Si 2—21¢ 48 gof 0; = 24 gm-equivalents =8 gn-equivelents. (a) The number of moles Cd* = 500 x10 x 0.2= 0.1 mol. Now, 0.2 mol of Ca”* require HS = O.Lmelor 01x04 = 94 & (8) Motes of KCl in 260ml of 0.5 MKC 05%250 S525 «0.125 Moles Moles after diktion of 950 ml to 500 mt Moles of KO1 = moles of CI” ions « 0.0625 Number of CI" iona = 6.02 > 1024 + 0.0695 = 3.76 x 107? 8 RE 87. 50. on, en. (©) Mass of water = 250° 1 = 250 ¢ Many «3008 (© Ratio by volume of A: B: 22. Ratio ofnumber efmelacules in A, Band C =2:5:2or1:62:1 ‘Thus, 1 molecule of C contsins 1 molecule (ke, 2 atoms) of A-and 5P molecules (a, 52% 2 atoms of BD. Hlonce the formula of Cis ABs (a) H atoms in 0.4 mol of HyS =2.Np x04 = 0.8 Np. (@) NzO, has to be mol. formula because its empirical formulais NO, MgoN ie only a molecular forms. (6) The balanced equation is SNHyNIy—+ 8N2 + 91, + 6NEL (a) Mol. fraction « tee: () Velency of X is 3 and that of ¥ is 2. Hence formula is ays (@) Let mace ofiren ie 100 g and that iron rusted i= g Ae + 30; —+ 2203 24 96 Increase in mass for 224 g Fe =98 204m, 0177 26x Increase in maseforxg Fe= 9S% = 0.42% Now, 0428¢=10 or 2= 28.9%. (61) Atonas of 60 g of NgO = BNA x$P=3.4Nq (2) Atoms in 17 gof Nity =4Nq (3) Atoms in 160 em* of pyridine (CyigN) 150 0.008 x 11Nq t= 2058 Na (4) Atoms in 1 mal of Np = 2Nq = 2Nj. (4) From the formula Cag(Pz07) the valency of pyrophos- phate radical is two, Thus iron (IID pyrophosphate is FexP20ns. (8) CoH +30 4 260 + 2F720 20 ml of OpHy requires Oy = 60 ml. (©) Atomic mana cf X = 9 12 = 108 m.n. . Mol. mass = atomicity x stomic mass = 108x4=82emu. (6) Mass of wea = Molarity x Mol. masexVi, 02% 60 x 10010 gn4.8 (©) This property is known as eflorescence. (a) In 125 g (GFM) of 21003, the mumber ofO stoms = 3Nq, Tn 125 gof ZaCOp, the number of O atoms 1203. Nq= 086.0210 » 1806%10", 1g of hydrogen is disp Hence, equivalent mass of (Nae) + Hele) —> 2 n “ 7, 2. Msss of 20 formed: 78. (MCL, +: AgNOg—>: nA goforygen =38 x4 = 4.5 ¢ 1+ (NOs), ences = 4 any DIFFERENT STATES OF MATTER + Solids. The substance is called solid if its melting point and boiling point are above room temperature. Matter in solid state has definite mass, definite volume as well as definite shape. Liquids. The substance is called liquid ifits melting point is below room temperature but boiling point is above room temperature. Matter in liquid state has definite mass, definite volume but no definite shape. Gases. The substance is called gas ifits melting and boiling points are below room temperature. Matter in ‘gaseous state has definite mass butno definite volume and shape. Plasma state. It consists of gaseous mixture of electrons and positive ions. About 99% of matter in this universe is found to exist as plasma state. GASEOUS STATE Some measurable properties of the gases aong with their respective units are as under. (a)Mass. Expressed in grams (g) or in mol (n). (6) Volume (V). It is expressed in litres (L), cubic metres (m®) or em? or dim’. 1 mS ~ 10° L~ 10% dm®~ 108 om’, (©) Pressure (P). It is expressed in the units such as ‘atmospheres, millimetres (rum), centimetres (em), torr, bar, ete. The SI units of pressure are poscals (Pa) or iloposcals (kPa). Tatm = 101.3 kPa=1013Nm“= 1.013 bar = 76 cm Hg = 760 mm or torr (d) Temperature. It is expressed in Celcius scale (0) or in Kelvin scale (K) TR) = °C) +273, These are the generalisations developed from the quantitative studies of behaviour of the gases. BOYLE’S LAW (Robert Boyle, 1662) The volume of a definite mass of a gas is inversely proportional to its pressure at constant temperature. 23 + Mathematical representation Pa} or PV=Constant (or PV; =PaVs ‘The constant K depends upon : temperature, mass and nature of gas. © The law can be represented by means of plots of Vo Por PV vs Por de Pat constant temper ture such plots are called isotherms, CHARLE’SLAW (Jacques Charle’s, 1787) The volume of a given mass of @ gas al constant pressure increases or decreases by =2- ofits volumeat °C for ‘ach degree rise or fall of temperature. * Alternative statement. ‘The volume of a fixed mass of the gas at constant pressure is directly propor- tional to its temperature in Kelvin. '* Mathematical representation. ¥ Ma Va VeT or = Constant (Q) or gaze ‘The value of K depends upon, mass, pressure and nature of gas. '* Graphical interpretation. The law can be repre- sented by plotting Vus Tat constant pressure. Such plot is called isobars. PRESSURE-TEMPERATURE RELATIONSHIP (Amanton’s or Gay Lussac’s Law) ‘The pressure ofa fized massofagas.at constant volume is directly proportional to the temperature in Kelvin. ‘* Mathematical representation. At constant volume ‘and fixed number of moles P«T or T ‘Plot of P vs T at constant volume is a straight line passing through origin. Such a plot is known as isochores. Pi_Pa = Constant) or i= ips: | 24 AVOGADRO'S LAW The volume of some number of molecules or same number of gram-molecules of different gases under similar conditions is same. Mathematically, it can be put a VeN or Ven ——(atconstant T, P) Here, N is’ number molecules and n is number of gram-moles. IDEALGAS EQUATION I is mathematical expression which gives the com- bined effect of Boyle's law, Chatle's law and Avogadro's law. ‘The equation is PV=nRT Here, R is a universal gas constant. + Ris constant and independent of nature of gas. + Rrepresents work done per degree per mole. ‘+ Rhas different values in different units. Some of which are 0.082 L at mK-! mot”! ; 82.05 em’ at mK! mol! ; 8.91 JE! mol"; 1.99 eal K! mot™!; 831 x 10" ergs K-! mol”; 8.31 dm’ kPa K-! mot”? + The ratio of universal gas constant to the Avogadro number is called Boltzmann constant (k) n-E No * The equation, PV = nRT is called equation of atote for ideal gas. «+ For a gas of fixed mass (2) undergoing a change from inital stato(P,, V1, Ty) tothefnalstato(Ps, Vo, 7), the general gas equation can be written as PiVi _ Piva hh + Relationship between density and pressure aRT PVenRT or PaAEE wxRT___dRT *GMMx V" GMM * 8.1.P. Conditions. S.T-P. stands for standard con- ditions of temperature and pressure. These includes (°C or 273 Kand 1 atmosphere pressure. DALTON’S LAW OF PARTIAL PRESSURES (ohn Dalton, 1807) ‘At constant temperature, the presoure exerted by a mizture of two or more non-reacting gases enclosed in a definite volume, is equalo the sum ofthe individual pressures or partial pressures which each gas would exert if present alone in the same volume. COMPREHENSIVE OBJECTIVE CHEMISTRY + Pressure exerted aqueous tension, parameter. + For gases collected of dry gas can be aqueous tension Pay gar = Pots * At constant tempe gas in a mixtare ny Water vapours is called ch isa temperature dependent the observed pressure, _ aqueous tension. ture, the partial pressure of a fnon-reacting gases is equal to the product of totalppenvtre and it mole fraction P-Pkx, AMAGATTS LAW OF PARTIAL VOLUMES At constant temperathre and pressures, the volume of a mixture of non-reacting gdsor is equal t the sum of thelr {dividual pera volumes qudey euler conditions Vrout= Yh +¥2 + ¥o DIFFUSION AND G! PS LAW (@ Diffusion. ‘The pipcess of freeintermixingof gases irrespective of grat external agency. (i) Effusion. The es through a very fi + Volume of a gas di called rate of dif tation and without the help of aping of a gas under pressure or small pin hale. fused or effused per unit time is jion or effusion. tes of diffusion or effusion of conditions is inversely propor- tof their vapour density. na Vata Ta Vaty d;, dg = respective vapour densities My, My = reopectife milecilar mas * Ifthe two gases: ise at different m Pie Padi ~ EQUATION Tes a matbematic exjresion decived on the basie of assumption of kipetic theory. PV=imNu? m =mass of each molecule © Kinetic Energy of relations 1 # root mean square speed N= number of molecules ‘a gas is given by the following GASEOUS STATE, E)=SRT — Forone mole of gas By=ZaRT Form moles of gus. ‘As the tomperature is increased, the molecules poss- ‘easinghigher speeds increase. Therefore, the maxima, in the curve gets flattened and shifts to the right side DIFFERENT TYPES OF MOLECULAR SPEEDS 1.Most Probable Speed (a). It is the speed pos- sessed by maximum fraction of molecules at par- ticular temperature, EE Here, R= Gas constant ; M = Molecular mass ; m = Mass of molecule 2. Average Speed (©). It is average of the speeds possessed by different molecules at particular temperature. It is given by following expressions oe petetuetes 280 2/8 W = im 8. Root Mean Square Speed (1 oF tinm). It is the square root of the mean of squares of speeds of individual molecules at particular temperature. It is given by following expressions oy og + og W et yal Ve “VM Ww Va Here, P= pressure ;d = density ; V = volume. + temn 7 oF oof a gas gets doubled when absolute temperature of the gas is quadrupled. + For two different gases, the ratio of root mean ‘square speeds at same temperature is given by ‘For the same gus, the ratio of root moan equare speeds at two different temperatures is given as [te u th 4. Relationship between 0, thrms and 5 B= 0.0218 > pam 3 5 1.128 ot = 973 & theme = 0.8184 thyme 1:7 tpgg = 1 1428: 1.294 This implies that ot any temperature, Urs > ¥ > ‘COLLISION PROPERTIES 1. Collision Diameter (0). When the molecules approach each other to collide, there is a distance of closest, approach beyond which they cannot get closer. The distancebetween the centres of the molecules when they are closeat to undergo a collision, is called collision diameter (0). Collision diameter can be calculated from the follow- ing mathematical expression, Deaverage velocity = van der Waal constant m-=mass ofmolecals R= Gas constant 11 viscosity M = molecular mass. . Mean Free Path (). It is defined as the average distance trovelled by the molecule between the two ‘successive collisions. Itis mathematically given by the following expression i Bron Here, 0 = Collision diameter +N = Number of molecules per cm’, ‘+ Mean free path « T (at constant pressure) + Mean free path « 5 (at constant temperature). 3. Collision number. It is the number of collisions which a molecule registers with other molecules per second. The mathematicel expression as derived from. ‘the kinetic considerations is 1- Collision number = V2 na"S N Here, N= Number of molecules per em? B= Average velocity. 4. Collision Frequency (2). It is the total number of collisions taking place in a unit volume of the gas at particular conditions in one second. The mathematical expression for collision frequency pertaining to binary collisions is e-Letont a” N Here, 6,0 and Nhave usual meaning. * Collision frequency « T’’? (at constant P) * Collision frequency « P* (atconstant T). 26 communns|yve Quacrive cuemusrey] VAN DER WAAL EQUATION + Real gases deviate from ideal behaviour and do not obey ideal goa laws at all tomperatures and pros sures. The deviation is observed at low tempera ‘tures and high pressures. ‘+ Deviations of res) gas from ideal bebavicur is ex- pressed in terms of Compressibility factor (2), which is mathematically expressed by relation evRD. sifz = 1, ie, PY=nRT haviour Iz >Lie, PV >aRT Positive deviation, i.e, the gas is less compressible than expected from ideal behaviour. Ife e1,ie, PV 1 (@=2 et ‘A vessel containing gas at a pressure of 60 em of Hz ‘was connected to arm A of open end manometer. The atmospheric pressure was recordedas 74 cm of Hg. If mercury in arm A stands at 84.5 cm height, the mereury in arm B will stand at (a)70.5em (0) Thom (0)245em (@)88em. Which of the fallowing gives a plot of compressibility factor (Z) ve P at particular temperature for helium ? @ ® z In certain experiment 150 ml, of Hz was collected at 1 bar at 27°C. The collected hydrogen represents @ 12mg (8) 0.06 mol we (2) 06 mol. ‘Which of the following do not pertain to the postulates of kinetic theory of gases ? (@) Molecular collisions are perfectly elastic ©)Gas molecules move at raniom with ever changing speeds ©) Molecular collisions against the wall are responsible of gas pressure. (4) KB ofa gasio given by the sum of 273 and temperature in colius scale. Fourgasballoons A, B,C, D ofequal volumes contain- ing, H, NzO, CO, CO, respectively were pricked with needle andimmersed in a tank containing COz. Which of them will shrink after some time (al ()toth A, c ©) only © (@) both A and D. According to Maxwell-Boltzman distribution ofspeeds among gas molecules, what is false ? (@) The maxima in plot of 3 us speeds pertains to most probable speed 0) The pat oN repair ne with spe (© The valve ofc incremeewithriwin temperate G2) Area under the curvegivesthatotal number of molecules Which of the following relations is true for ideal gas & refers to kinetic energy of molecules) ove 2NE @ Pa p= E 2NE @P2iNe (@)bothd and. Which of the following mixtures of gases at room temperature follow Dalton’s law of partial pressure. G@) Nig, HCL (6) Ha, Op @No, op (@ Al the above. Which among the following samples of gases contains Avogadro number of atoms at S.T-P. ? @)2gof Helium (©) 11.2Lof carbon monoxide © 112 Lofsulphur dioxide @ 1 mol of phosphine. 30 ENSIVE OBJECTIVE CHEMISTRY | a1. Which of the following expressions does not cor- respond to Boyle's law ? (a) PV =constant VP =PaVy (e)botha ands or 1000 mi of a gas A at 600 torr and 500 ml of gas B at 800 torr are placed in a 2L flask. The final pressure in atmosphere is (@)2000 (© 500 @ 1000 (os. Helium atom ie twice as heavy as Hy molecule is At 298K, the average kinetic energy of helium is (a) two tims that of kydrogen (yale that of hydrogen (6 fous times that of hydrogen (@)sameas that of hydrogen. What is true about the gas that diffuses through the porous plug at 2th of the rate of diffusion of dihydrogen gas? (a) The molar mess of the gos is 72kg mel“ (Oy ts one ofthe structural isomer of GsEf12 (6) The vapour density ofthe gaa (c) Tha gow is lightor than dihydrogen ‘The gases A and B have molecular masses 64 and 100 respectively. ITA effuses out of porous plug at a rate of 16 mL}, the rate of effusion of other gas under similar conditions will be (@)12 mL s ents! (24 mbs (@10mLe! ‘The temperature of ideal gas can be increased by (a) decreasing the volume and pressure but keepingthe amount conatant (@)increasing the pressure but keeping thevolume and ‘amount constant (©) decreasing the amount but keeping the volume and pressure constant (a) any of 6 oF c operation. ‘The melar volume of CO, is maximum at (@stP (8) 127°C ond 1 atin pressure (©)0°C and 2 atm pressure (@) 227°C and 2 etm pressure. 72. 37. 39, 41. ‘The bottle of liquid ia is cooled before opening the seal so as to Io (@) vapour pressure (8) mrface tension (viscosity (Gl extent of H-bonding, ‘Two gases X and Ylare at same temperature and pressure. The {temperature of Xis below unity while that of Y is abdve unity. Thus, (a) X can be liquefied by compression but not Y (6) ¥ ean be liquefied by compression but not X Air contains 79% Ng sare is 750 mm of (0) 175 mm of Hg land 21% Oy by volume. If pres- , the partial pressure of oxygen (©) 157.5mmof Hy (0 32) mm of Hg (@) 250 mm of He. A gaseous mixture dontaining 2 g H,, 8 g He, 22 g C02 and 8 g Op is edclostd in a vessel. The gas with highest partial pres @He Oi (0.003 0, An ideal gas cannot be liquified because (@) its critical temperature is always above 0°C (6) its molecules are refatively small in size (it solidifies before bacoming liquid (@ forces operstingbereen its molecules are negligible. The density ofa gus t.27°C and one atm isd. Pressure ture will the densityfbe 0.75 d (a) 20°C @ 130°¢ (400K (@) 900K. (©) same in all the (© greatest in solid: @ greater in liquid Under which of the (@) 16.atm, 200K (©) Late, 273 K (8) 0.5 atm, 500 K (@) 15 atm, 500K. or a fixed number of gas moles, gps inerenses with the rise in temp. GASKOUS STATE 41. 51. 52. 31 (a) increase in average molecular speed (©) increased rate of callisions amongst molecules (@) increase in molecular attraction (@) decrease in mean free path: A gas of volume 100 cc is kept in vessel at pressure 10' P maintained at temperature 24°C. If now the pressure is changed to 10° Pat the same temperature, the volume of the gas becomes (@)10c (8) 100 ce lee (10° ec. Equal masses of methane and hydrogen are mixed in an empty container at 25°C. The fraction of total pressure exerted by hydrogen is wi ws 4 a wh. Five gram of each of the following gases at 87°C and 750 mm pressure are taken, which of them will have least volume ? HF @Ha ube om ‘The critical temperature of water is higher than that of Oz because H,O molecule has (a) fewer electrons than Oz (t) two covalent bonds (V-shape (@ dipole moment >0. Om of gas weighs 2 g at 300 K and 1 atm sure. If the pressure is made 0.75 atm, and Temperature i ‘is brought down to 250 K, the gas will occupy a volume of @2L @inn 222L om, In aclosed room of 1000 m*, a perfume bottle is opened up. The whole room after some time develops smell of perfume. This is due to which property of gases (a) viscosity @) density (© diffusion @ none of these ‘The density of Neon is highest at (@) STP 0c, 20m, (©) 278°C, 1 etm (278, 2 ate, ‘The number of moles of Hy in 0.224 L of hydrogen at STP (assuming ideal gas behaviour) is, @. oa oor (oon. 55. 57. 5a. ‘Which of the following statement is false ? (@) The product PV for fixed amount of gas is independent of temperature. (@) Mclecules of different gases havesame KE at a given given temperature. (©The gas equation is not valid at high pressure and low temperature, (@) The gas constant per molecule is known as Boltzmann Boltzmann constant. ‘The dimensions of pressure are same as that of (0) Energy (6) Energy perunit volume (© Force per unit area (d) Force per unit volume. ‘The densities of two gases are in the ratio of 1 : 16. 1d i 1:16. ‘The molecular velocities of two gases at same temperature are u and ug, Their masses sre m, and ‘my respectively, which of the following expression is correct mm 7. ate (0) myuy = mau © myam mae, (mpg? = maul, ‘The ratio of tho root mean equare velocity of Hi at 50K and that of Oy at 800 K is (a4 we 1 or @z ‘X ml of H; gas effusos through the hole in a container in 5 seconds. The time taken for the effusion of same volume of gas specified below under identical condi- tions is (c) 10 see ;He (©) 25 s0e;CO (8) 20 sec; U2 (d) 85 se; C02. ‘The average kinetic energy per molecule of ideal gas at 25°C in SI. units ia (oar? es werrx10 3 6.x (a) 716 x10 5, Which of the following law lead us to arvive at the conclusion that 1 g-molecule of each gas at 8.7.P. occupies 2 volume of 22.4 L? (a) Dalton’s law (8) Law of combining volumes © Avogadro's lew (eh Boyle's law. G1. The compressibility factor for ideal gas is (15 @10 (20 ee. 62, A sample of gas occupies 100 ml at 27°C and 740 mm pressure, When its volume is changed to 80 ml at 740 mm, the temperature of the gas would be wee @) 240° (39 (soc. 63. Real gases deviate from ideal behaviour because, the melecules (@)arecolourless (b)attract each other (contain covalent bonds (d) show Brownien movement. If volume of two molos of ideal gas at 540 Kis 44.8 L, then its pressure will be @1ata (©) 204m sam @4om, 65. 1.12 10°" ccof Oy atS.T.P, contains molecules equal to (@)3.10% 10" )3.01x10 (©3.01x10" (01x10, ‘The volue of universal gas constant depends upon (a)temperature othe gos (8) volameof the gas (Cawmber of moles ofgas (d) none ofthese. 87. At 26°C and 730 mm pressure, 980 ml of dry oxygen was collected. If the temperature is constant what vohtme will oxygen occupy at 760 mm pressure (@)365 mt © 449 mt (0369 mt (4) 621 mi. A mixtare of three gases A, B, C has a totel pressure of 10 atm and total number of moles equal to 10. If partial pressure of A and B are 3.0 and 1.0 atm respectively and C has mol. masi in grams of C present in the mixture is, @)4255 ws wr 8. ‘The r-m.s, velocity of hydrogen is YT times the rm.s, velocity of nitrogen. If T is the temperature of the gas (0) Tig Try OT > Ty OT, 2241, Vn <224L (Vn = 2241 (Vy = MBL n. 7. 7%. 16. cia Al 100°C and 1 atm, if denkity of iquid water is 0 gem" * and that of water vapourfis 0.0006 gem” °. The volume cccupied by water molefules|in 2 L of steam at that temperature is (@6an* oben? (©06 em ()}0.06 em’, Which ofthe following ghs has highest value of (one 02 cy Om. ‘The van der Waal equatjon for 0.6 mol of real gaa is ofr] Ss oa ©) Pega avn (Pes aay or (em Which among the following has rate of effusion lees than the moist air He (©) Dry air (ONE (qd) Heavy hydrogen. ‘The mass of CO that cfn be mixed with 70 g of dry nitrogen so that both hdve same partial pressure is @0¢ wR. nde (28x70. ‘The conditions at wifich a sample of ideal gas exhibit apresoure of [5 atm and also concentra- tion of 1.5 mol "is (@)-208¢ (@) Letan, 260 woe (@81P. coniitions, Which among the follbwing gases is most readily soluble in water under| conditions ? @ Nig (©) Carbon dioxide ons mp. A sample of soda erysiai weighing 1.287 gon reaction with excess of hydrocBlorie acid gave 100.8 cm® of COp at ST.P, The sampled mainly (@) monohydrate sodium carbonate (@)heptahyarate sodium (© anbydroas modivm earfonate GASEOUS STATE 33 79. Which among the following contains Avogadro num- 81. ber of molecules ? () 38 of water vapours (8) 49 g of hydrogen sulphate (17 gofammenia (c 4.8 g of carbon dioxide. ‘Open end manometer was connected to gas chamber. ‘The Hg level stood 15 mm higher in the open end as compared to the end connected to gas chamber. If the atmospheric pressure is 101.3 kPa. The gas pressure inkPa is 1033 e013 (e)943 (a) 115.8. Each of the four species, hydrogen sulphide (1), protium oxide (11), hydrogen chloride (I1l) and chlorine (IV) at S.7.P. have been put into four separate vessel each of one litre capacity. ‘The largest number of molecales are present in wt on (eM {(€) Allwill have same no.of molecales. ‘What is wrong about the 60 L vessel that contains dioxygen snd dihydrogen in the molar ratio of 1: 1, at apressure 101.8 kPa? (a) Hy molecules move faster than oxygen (©) Bach gas has same number of molecules (€) 102 ia removed from the mixture, the preeeure wil fll 10253 kPa (@ Onthe averege, the molecular impacts of yon the walls per unit time are larger that those of oxygen. Which of the following has maximum number of molecule (@2.7 gofNHy (6) 1LofSO, at STP. (0) 2LofCh at STP. (4) 04 mel of HS. By what factor does water expand when converted into vapour at 100°C and 1 atm pressure. The density of liquid water at 100°C and 1 atmis 0.96 gem". (ais 46) 2000 (©1690 (d) 500. ‘The value of critical volumes of four gases A, B, C, D are 0.025 1, 0.312L, 0.246 L and 0.492 Lrespectively. ‘The gas with larger value of van der Waal constant’ A oR ep (d) unpretictable. 7. Which of the following gas has rate of diffusion 0.88 times that of phosphine ? (@NOg won, 10 Leach of hydrogen and oxygen gases were mixed at 150°C and 1 atm pressure, An electrical spark is. struck in themixture. The total volume of the gaseous mixture after the process (under original conditions) would be wasn 75L A gas undergoing expansion through porous plug ex- hibit neither heating nor cooling if its temperature is equal to (a) Boyle's temperature (6) inversion temperature oe (critical temperature, The behaviour of real gas is generally depicted by ploting which of the following parameter vs pres- sure (a) critial volume (@) density (©) Tat’ Troal (@ Vat’ Vit Pick up correct answer among the following about Boyle's temperature (@) The ratio PPT approaches unity (©) The ratio RT/PV approaches unity (c) The ratio PV/RT approaches unity (d) The rotio FVIRT approaches zero, ‘Two gases X and ¥ have densities, diy) = Sdy) and molecular masses, Me) = 0.5 My. Then, the ratio of their pressures, i¢., pe: ry would be 1 wt oa wae. Which offollowing is correct plot ofthe volume of fixed amount of ideal gas as a function of temperature (at constant pressure) ®) o wo WOT, @wL (@)20L. 1 4 v v TH) re 34 COMPREHENSIVE OBJECTIVE CHEMISTRY] © (a THK) Tk) ‘Three different gases X, Y, Z of molecular masses 32, 64, 71 were enclosed in a vessel at constant tompera- ture till equilibrium is reached. Which ofthe following statement is/are true? (6) Gas ¥ will be at the top of the vessel (c) Gas X will be at the bottom and Z willbe at the top (d) Gases will form homogeneous mixture: Atcertain place at 20°C the relative humidity of air was, found to be 65%. If aqueous tension at 20°C is 17.6 mm. of Hg, the partial pressure of water vapours in air is (@)175mm (@) 11375 m (15.47 mm (@) 20mm. A bottle is heated with a mouth open from 27°C to 127°C, the fraction of air originally present in the bottle that is expelled is (a50% (33% ‘The temperature of 1 g of helium that is confined to 2 L flask at a pressure of 2.05 atm is ©) 25% (a) 40%. Choose the correct answer from the four alternatives given in each of the [ol An ideal gas is at pressure P and temperature T, in a bor which has been placed in another evacuated large container. The inner box is pricked so that the gas, inside it starts escaping out. What is correct ? (a)the temperature falls (the temperature rises (6) the temperature remains the same (aunpredictable. For ideal gases isotherm refers to the (a) gases at same temperatures 100. (@) 200K (@) 200°C (10K (@ joorc 5.6 Lof an unknown gas ipquires 12.5caloriestoraise its temperature by 10°Clat chnstant volume. The ¢), and atomicity of gas are Jespectively (a) 2cal, 2 (b) Teal, 2 (©) Weal, 1 (d) 15 cal, 3. A rubber balloon pble to all isotopic forms of tytiogen ned it fery hoes eet pacd x tankof pure hydrogen. some times, the balloon will tostrinkinaze |e) expand C@rensinassch | ovink thao si. ‘certain gets out ew erat veses and B. Ahasa circular orificg while Bhas a square orifice of length equal to the raqius of the orifice of vessel A. Thereof rateofAufon othe gr Bom wooed A to that from vessel Bis wr wt ean ap:2 Which of the following sfatement's is/are true? I. The ratio of average 5 with temperature II. The mean free path in sure at particular to 111. Rx Avogram = Bolt 9d to rms ofthe gas changes Avogadro's law. Ln n,m (©) geses at same pressure (© gases having same heat}eapatities (@) plot of P vs V ateo A thermometer reads OPC ahd barometer reads 700 torratthe top of mountafn, while the temperature and pressure at the bottom pf mountain is 27°C and 750 torr respectively. The ratio of density of air atthe top (d,) and that at the bottbm (d;) is (ay 1e2 (@) 10251 (2:1 (212. GASEOUS STATE 35 4. The volume occupied by 10.0 g of oxygen at 0°C and 2 atm pressure will be spproximately (20% (4481 (ash @ish. 5. For X g of ideal gas V and Vo are the volumes at ¢°C and 0°C respectively at 1 atm pressure so that V/Va =(1+u). Here, ais (et (<0 @>1 @=0. & Aproosure of 0.101325 bar when exprescedin atmos- pheres represents (0.01 atm ©) tatm (0. stm @ 10am. 11, Three gasee X, ¥, Z are enclosed in a container and have partial pressures as p1, pz, p3 respectively at ‘Tkelvin. The total pressure at T kelvin will be (a) always equal to (py +72 +3) Owitertey (d= (p1 + 2 +3) only if the gases do not react chemically at T. A.sample of gas contains Ny molecules and the total kkineticenergy at - 123°C as E, ergs Another sample of gas at 27°C has total kinetic energy as 2; ergs. Assuming gases to be ideal, the number of gas ‘mwlecules in the 2nd sample will be Ni @N oF om, wan, Among the gases given below, the highest value of mean free path is for, (Ny om, (6) 0, won, 10. If two gases have molecular masses Ma, Mp at temperature T, and Ty such that, T,Mp=Ts Ma. then which property has the same magnitude for two ase (a)density () pressure (ORE. per molecule @rms, 11. The state in which 99% matter of the universe exists is (@)Plasma () Salia (Gaseous 4 Liquid, 12. ‘The temperature above which the gas cannot oxist as | liguid howsoever large the pressure may be, is 18. 4“. 16. a. (a) Absolute zero (8) 0° Caleine (¢) Critical temperature (@) Inversion temperature, ‘The density of a gas is given by the expression (aynP PRT © PMRT wom, ‘Tho comproseibility factor for ideal gas at particular temperature (a) equal to z0r0 <0 (6) equal to 2 (d) can be > Lor <1. Partition Gas A| Gas B | ——____ 250mm) 250 mm| removed at same | Pressure =P temperature In the above experiment, the value of P is (a) 250 mm (©) 500 mm () 300 mm (4) 400 mm. When ideal gas undergoes unstrained expansion, no cooling occurs because the molecules (a) are above inversion temperatire (©) exert no attractive force on each other (c) work equal to loss in kinetic energy (cd) callide without loss of energy. Rate of diffusion of a gas is (a) directly proportional to its density (6) directly proportional to its molecular mass (c) directly preportional to the equare root ofits molecular (d) inversely proportional tothe square rost ofits molecular mass, The average velocity of an ideal gas molecules at 27°Cis 0.3 msec" |. The average velocity at 927°C will be (a) 06 msec (8) 0.3 m see (©) 09 msec” (23.0 m see" | Ifa gas is expanded at constant temperature (a) the pressure decreases (©) the kinetic energy of molecules increases (c) the kinetic energy of molecules decreases (d) the number of molecules of the gas increases. In van der Waal's equation of state the term which accounts for intermolecular forces is @wv—o) oP savy RT 36 at. The values of van der Weal’s constant ‘a’ for 0, Np, NHy, CHy are respectively 1.360, 1.390, 4.170 and 2.253 L? atm mol” * respectively. The gas which can be liquefied casily i (@) 0» (Ne (NHa (CH, ‘The density of neon will be highest at (STP (6) 0°C and 2 atm (© 278°C, Latm (d) 278°C, 2otm. ‘The rate of diffusion of methane et certain tempera- ture is 2 times that of gas X. The molecular mass of X (640 (o) 2.0 oso (aso. ‘An ideal gas at 1 atm is heated from 0°C to 873 K. If its initial volume is 5.46 L, the final volume would be 746 L (b) 18651, san (aou. ‘Which one of the following gases will contain largeat, number of molecules under similar conditions ? 86g 0fNe (@) 32g ofoxygen (©) 12¢0f helium (a) 32 g of methane. ‘Which of the followingrepresents mathematical state- ment of Boyle's law ? (c) VI= constant at constant T (8) PV-= constant at constant T (©) PV = constant at constant T and n (d) PV! = constant (y being Cp/Cy). ‘The behaviour of a real gas approaches ideal be- haviour at (6) low temperature, low pressure (high temperature, high pressure (o)low temperature, high pressure (@) high temperature, low pressure, ‘The critical temperature of water is higher than that of oxygen because the water molecule has (c) two covalent bonds @ a linear shape (01a dipole monent>0 ()a loser number of clectroas than oxygen. ‘The average kinetic energy per molecule of ideal gas is equal to oss @osRt rset (sR, 32, 33. @)2mv mo ome? Od mt. ‘The mixture of three gqses X (density 0.90), Y density 0,178) and Z (density} 0.42) is enclosed in a closed ‘vessel at constant temperatare. When equilibrium is established, the (©) gnsesX, Y, Z will conshitute homogeneous mixture ‘throughout the (@) gas ¥ will bo at the . ‘Which of the following is| not the postulate of the Kinetic theory of (@) gas molecules rein apermanent state of random motion (©) pressure of gas is duelto molecular impacts on the walls © the molecales are: have speedsof 2ms"!, 4 ms”, 6 ms" respectively. The root mean square 22428) oT va ss = o(F) o%. Which of the following moleciles has the lowest average speed at 273 @co wor 00, (calls ‘At what ratio of the partial presouree, poy : PN, Will themassas of two gasds, Oy and Ns containedin vessel be equal ores (oa ua (oars. AO. dm! flask contqine gas A (Mol. mass X) and ‘Ldmé flask contains qgas B(Mol. mass 2X). Ifdensity of gas Ais 3 gdm” 4nd that of B is 1.5 gdm™®, the ratio of the pressure fa : Pp is GASEOUS STATE 37 (wast 3:1 (2:1 1:2 38, 56 g of Nzis mixed with 44 g of CO, ina container in which the total pressure at 20°C was recorded as P ‘atm. The partial pressure of Ny in the container is @Paotm ©) PB atm (P2atm (Cf) 2°03 stem, ‘39, Which of the following will not change ther.m.s. of the gas molecules? (@) Changein temperature (6) Changein veluine at constant T (€)Change in pressure at constant T (@ Changein number of gas molecules, 40. By how many folds chould the temperature of the gas be increased in order to increase the r.m.s. speed of ‘gas molecule in a dosed container from 5 x 10* an s”* to10x10'ems! (0.5 times (8) 04 tines (4 times (cd) 40 times. 41. The value of Avogadro constant is (6.02% 107* mor! (6) 02x 10 gm"! (€)6.02 10% mot”! (2) 602x107 kg"! 48, The units of van der Waal's constant 6 are (@molL! ©) mol 'L ©malL (@) mol” "1 43. A gaseous mixture contains oxygen and nitrogen in the ratio of 1 : 8 by mass. The ratio of their respective number of molecules (No, : Nx,) is (ise ut 7:64 (ise, 44, The temperature at which the rms. velocity of CO becomes equal to that of nitrogen at 21°C is @ 273K @) 871K @1B71c (2) 400K 45. Real gases deviate from ideal behaviour because their molecules (a) cause repulsive interactions on one another (©)cause attractive interaction on one another (rare polyatomic : (2 donot undergo elastic collisions. 46. Two gases X and Y have their molecular speeds in ratio of 3 : 1 at certain temperature. The ratio of their ‘molecalar masses Mz: M, is a. oL. (@)1:3 (3:1 19 (yo: Critical temperature of the gas refers to the tempera- ture (a) st which it occupies a velume of 2.4 1 (b) ot which if gets liquefied at 1 atmosphere (below which it alwoye exists as liquid (@) above which italways eristeas gas. ‘The ratio of universal gas constant to Avogadro num- ber (R :Nq)is commonly known as (@) Planck's emstant (@) Boltemann constant (©) critics constant (a) no apocisicname is ansosiatod. ‘IfVo is the volume of X gof an ideal gas at 273 K then its volume at 20°C will be (@) 20Vo vox 2d BaVy 20 oR cova Which of the following expression does not give root mean square velocity of a gas at particular tempera- ture (@ erry? (wy @Pvamy'* (@) @PraL)* (asrv.ag), “One gram-molecule of o gas at S.T'P. occupies 22.4 LL.” This fact was derived by using (a) Dalten’s theory (©) Dalton's law of partial pressure (©) Avogadro's low (@ Boylsolaw. (One litre of a cortain gas weighs 2 at 300 Kand 1 atm proseure. If pressure is reduced to 0.75 atm, then at which temperature 1litre of the gas will weigh equal to 1 gm @ 50K (&)500K © @0K (900K. 50 cm® of a certain gas A (mol. mass = 64) diffuses ‘through a porous plag in same time as 40 cm of gas 1B (mol. mass = M) under similar conditions. The value of Mis (@) 200 @ 160 (4096 (e100, Which of the following does not pertain to thevalue of, ‘universal ges constant 38 a7. 61. (a) 831K"! mol" (6)0082.L atm K! mol! (@)0.082 Lata” ' Kine (@) 82.05 atm em? mol”! The volume of a gas of fixed mass at 0° Celcius is (a) <0 @)>0 (=o (@ 0. Hence inter-partisle forces in water sir. Other gases have VD. less than moist air(V.D.9— 14). ee 15, @) 10¢0fN2 = 22 = 2.5 moles, Mass of 2.5 moles of CO. a9, (Vy AUT Ve, ms » (0)Ve= ap = "300 «0.75 * =25xB= 1g. 60. (a) This is as per definition of diffusion. mh. ©) P¥seRr a. @ pe, 1521 pos K 62 (0) 22.4 L of Hydrogen = 1 mol = 0.224 L of Hydrogen ab Usx00et 0.01 ma. #1295 -278 = - 50.80. 59, (a) The product PV depende on tempersture and number 7%. (a) Rlfy molecules are able to form H-bonds with water of mole. nolectlen é 1B. (4)NagC0g x HO + 2HC1-—+ 6% Cutty! 4Oy= NaCl + COy +(e +1) HzO 56.) Apply Grahom's aw. 224 Lof €Og S.1P. is produced trom coda rystl 60. ()KE parniclecale ie given by SED, 287 x 22400, 2NA os 768 60. (c)Avogwcr's Iw help us to derive this conclusion 61. (6)Thecompressibility factor for ideal gas = 1. Now 26 = 106 + 18 x or x = 285= 106 49 ¥, 2, Co Amply Fa Henes the sample is Nags . 108,0. na. (c)Partial pressure of C= 10-4 =6 atm. Number of moles of C = 6 moles (Because 10 moles have pressure = 10 atm) Mass of C =6 x2= 12. oR tg TNa%Miy 1 un Tn,x28 Tyee (biz < Lor PV oM @<5M (@ Cannot be predicted. The enthalpy change on mixing water and ethanol at room temperature is ()Zr0 Positive © Negative (@ Depends on proportion of alechol ic. the solution, ‘Two liquids A and B form an idesl solution. At 300 K the vapour pressure of a solution of 1 mole of A and x moles of B is 550 mm, If the vapour pressures of pure A and B are 400 mm and 600 mm respectively, then is @1 we wos @. ‘The number of moles of sodium hydroxide present in 2.6 Lof 0.5 M aqueous solution is (a) 1.25 05 os @s. ‘The molality of 2% (wh) aqueous solution of sodium chloride is (0)0.17m im @)0.7m @034m. 10, The molarity of decinormal aqueous solution of sodium hydroxide is ms @Mno ome (@Mnoo. 11, ‘Two beakers contain 1 L and 260 ml of pure water respectively. What is same in them? (@)number of meles (© number of molecules (molarity (@) mass. 12, According to Raoult’s law, the relative lowering of vapour pressure ofa solution of non-volatile solute is (a) equal to mole fraction of solvent (equal to mole fraction of solute (© directly proportional to mole traction of solute (@ equal to normality of the solution. 18. Which aqueous solution will have lowest vapour pres- sure at room temperature ? (01M Nec (@)0.1M glucose 01M urea (01 Mcach, 14, Athigher altitudes, water boils at temperature < 100°C because () temperature at higher altitudesis low (®)atmospheric pressure is low (othe proportion of heavy water increases (@ atmospheric pressure becomes more. 16. An azeotropic solution of two liquids has boiling point lower than ether when it (@) shows a negative devistion from Raoult’s]aw (®) shows a positive deviation from Raoult’s law (©)shows nodeviation from Reoult’s law (@is seturated. A liquid is in oquilibrium with its vapour at its beiling point. On the average, the molecules ofthe two phases will have (a)same magnitude of interparticle forces (&)same potential energy (©)same total energy (@)oamne free energy. 17. Ehullioscopic constant depends upon (o)nature of solvent (@)nature of solute aa You have either reached a page that is unavailable for viewing or reached your viewing limit for this book. aa You have either reached a page that is unavailable for viewing or reached your viewing limit for this book. aa You have either reached a page that is unavailable for viewing or reached your viewing limit for this book. aa You have either reached a page that is unavailable for viewing or reached your viewing limit for this book. aa You have either reached a page that is unavailable for viewing or reached your viewing limit for this book. aa You have either reached a page that is unavailable for viewing or reached your viewing limit for this book. aa You have either reached a page that is unavailable for viewing or reached your viewing limit for this book. SOLUTIONS a1. a. (@) 18.6 (134 wos (136. ‘The molarity ofa solution obtained by dissolving 0.01 moles of NaCl in 500 ml of solution is (0.01 610.005 M. (002M @o.0m. 5.85 g of NaCl is dissolved in 500 ml of water. The molarity is @or 102 os aoa. In the phonomenon of osmosis, the semipermeable membrane allows the passage of (a) solute particles () solvent molecules only (©) both solute and solvent (a) none. If 6.85 g of NaC! aro dissolved in 90 g of water, the mole fraction of NaCl would be (a0. 001 oz (d)0.0196. Equivalent mass of crystalline oxalic aci (@) 90 ayes. 8a (ays. 4.0 of caustic sodais dissolved in 100 cm of solution. ‘The normality of solution is wt (oa 05 ao ‘The osmotic pressure of 5% (mass-volume) solution of cane sugar at 150°C (mol. mass sugar = 342) is. (a) 4 atin @)B.0T atm (€)3.58 atm (a) 2.45 atm. The molarity of the solution containing 7.1 ¢ of 'Na,SO, in 100 ml of aqueous solution is, @2m 05M om (d) 0.05 M. ‘The volume of 0.1 M HaSO, required to neutralise completely 40 ml of 0.2 M NaOH solution is (@) 10 mi (8/40 mt 20m (a) 80 mL ‘The normal boiling point of waters 973 K (at 760 mm). ‘Vapour pressure of water at 298 Kis 23 mm. Ifenthalpy of vaporisation is 40.656 kJ mol’, the boiling point of ‘water at 23 mm prossure will be (250K (6)298K (516K (125K. 49. 95 ‘The concentration units independent of temperature would be (@) Normality (6) Mass-volume percent (c) Molality (@) Molarity. According to Raoult’s law, the relative lowering of ‘vapour pressure of solution is equal to the (@) Molo fraction of solute (6) Moleo of aolute (6) Mole fraction ofselvent. (4) Moles of solvent. 51. 52. 87. of vapours pressure of first solution is (a) equal tothat of second (6) greater than the second (c)lessthan thesecond (2) doubleof the second. Calculate the normality of 10 ‘volume’ HzO, solution. @1IN oN ()90.3N @)0.0903N. i Inotonic solutions have same (a)Molar concentration _(b) Molality (Normality (d) None ofthese. ‘Which of the fallowing methods is used for measuring the osmotic pressure of the solution ? (@) Ostwald method (©) Berkley Hartley method (e)Solray method (@) Haber's method 2N HCI will have the same molar concentration as (O5N M80, ©) LON Hy804 (12 NHy804 ()4N HyS04 Vapour pressure of CCly at 25°C is 143 mm Hg. 05 gm of non-volatile solute (mol. mass 65) is dissolved in 100 ml CCl. Find the vapour pressure of the solution. (Density of CCl, = 1.58 g/em’). (@) 141.93 am (6) 94.39 mm (19984 mm (2) 148.99 mm. ‘What is the molarity of HaSO, solution that has a density of 1.84 gm/ce at 35°C and contains 98% by weight ? 4.18 alam, 184M 18M. ‘The relationship between osmetic pressure at 273 K when 10 g glucose (P}), 10 gurea (P,) and 10g sucrose (Ps) are dissolved in 260 ml of water is (0) Py>P2> Ps @)Py> Pi > Pa (Pp >Py>Py (Pp>Py> Py. aa You have either reached a page that is unavailable for viewing or reached your viewing limit for this book. aa You have either reached a page that is unavailable for viewing or reached your viewing limit for this book. aa You have either reached a page that is unavailable for viewing or reached your viewing limit for this book. SOLUTIONS 99 m1, 2. On cooking the dilute aqueous solution of sucrose below its freezing point what will be observed (ice wil start separating out (®)sugar will rystallise oat {©)sotution will freeze as such with same composition (a) water and augar in frozen part will be present in the stoichiometric ratio of 1:1. At certain temperature, the osmotic pressure of an aqueous solution ofurea was found to be 405 mm. How ‘many times the solution should be diluted in order to exhibit the osmotic pressure of 81 mm at the same temperature ? 113. The mass of solute ‘A’ (mol. mass = 75 g mol”) that should beadded to 180 of pure water in order tolower its vapour pressure to 4/5th of its original value is @ 1808 0g ©1508 wee. 114. Which of the following statements about the molal elevation constant, K; is true ? G@)It depends upon the nature of solute (6) Its units are K kg mot”? © Ita value depends upen cone. of the solution Its value is independont of the nature of solvent. — oe e QUESTION BANK (Level 1 © Ze) Be) 4) Ba) Bo) Te) 8a) a@) 10. (b) IL@) 12.(6) 13.) 14. (6) 16. (6) 16. (@) 17. @) 18.() 19.6) 20. (c) ‘21. (a) 22. (6) 23. (d) 24. (6) 25. (c) 26. (b) 27. (a) 28.(d) 28. (d) 30. (¢) ‘81. (6) 82, (6) 87. (d), 58. (b) 59. (2) 60. (6) 61. (6) 62, (0) 63. (c) G4. (c) 65. (b) 66. (a) 67.(c) 68, (a) 69, (d) 70. (d) ‘Th. (@) 72, (b) ‘97. (a) 98. (b) 99. (¢) 100, (6). QUESTION BANK (Level II) Le 2.6) Be) 4@ 5d) 6@) (a) 8.(b) (@) 10. (a) 11.(@) 12. (6) 18. (e) ‘14. (e) 15.(c) 16. (6) 25.(a) 26. (a) 27.) 28. (c) ‘29. (a) 30. (0) ‘31. (6) (32.(d) 49. (a) 50. (a) 51.@) (52.(a) 53. (b) 54. (6) 55. (a) ‘56. (c) B70) ‘58. (d) 59. (d) 60. (c) 61. (c) 62. (d) 63.(b) 64. (c) 65, (d) 66. (6) 67. (c) 68.(c) 69. (a) 70. (6) 71. (6) 72. (6) aa You have either reached a page that is unavailable for viewing or reached your viewing limit for this book. aa You have either reached a page that is unavailable for viewing or reached your viewing limit for this book. aa You have either reached a page that is unavailable for viewing or reached your viewing limit for this book. SOLUTIONS 103 ‘90, (c) Azeotropic solutions keeps on distilling at constant composition and cannot be separated into constituents liquide by simple distilaion. or int-% 982, (a) Assuming complete dissociation of particles the solute particles concentrations as 0.001 M MgCl = 0.003 M 0.001 M NaCl= 0.002 M 0.001 M urea = 0.001 M 0.01 M NaCl = 0.01 M ‘Thus correct increasing order of boiling prints is 0.001 M ‘urea < 0,001 NaCl < 0.001 MgCl; < 001 M NaCl 5. (d) Molarity and normality depend upon volume of the solution. Volume of the solution changes with change in temperature. 6. (d) Factual question. 7. (a) Like urea, glucose and sucrose are also molecular solutes which do not underge association or dissociation in solution and hence preduce same ne. of particles in the solution. (=CRT. (a) Factual question. 10, (a) ()Normality may be equal toor greater than melarity. (6) Normality of a solution decreases with increase in temperature. 11, (@) Sodium chloride and sodium sulphate are ionic com- pounds, They dissociate in solution producing more no. of particles. 13. (¢) Mole fraction of solvent = 1-0.2=08 Moles of selvent = 08 x5 = 4 14. (@)NxEq, mass =M xMol. mass M>xMol mass _ 2.3 x98 °' Ba, mass 48 °F Normality 16, 19. @a- a= 20 ~)=20 -0.52) = 0.96. () Kor the application of Raoult’s law, the solution should bedilute andthe solute should not undergo any association oF dissociation, (©) The relative lowering of vapour pressure will be least for glucose. The other solutes being ionic will ionise to {nerease particle concentration in solution, (a) Thisis as per statement of Raoult’s aw. ()Sucha solution may exhibit positive or negative devi tions depending upon the nature of A and B. () Factual question. (6) In cyelohexancl-ethanol the solute-sclvent interactions will be weaker than eolute-solute and sslvent-solvent in- ‘teractions, Ty = (100 + 0.256) (©) Electrolytes dissociate in solution producing higher cone. of particles and hence produce higher colligative effet. (@)No. of equivalents of NOH NxV 1250 =" 1000 225040 1000 (6) 1% olution of NaCI has maximum concentration of particls in the selution. (a) Since both are non-electrviytes and have same molar cove,, beth selutions would have same osmotic pressure, (a) Strength of solation i 10% oF 100 gfe. ‘The volume of solution containing 180 gof the solute 1000 = 002 180 = 1800 ml or 1.8L Mass of NaOI slg saa) 02 (ppm (trie) = 22 10° = 400 (a)Bq,maanot the aid =222 100 1010 mof01 N station cntaina = 100 10g late 10.100 _ 100 ml of0.1N sotution contain = 22730 1g aa You have either reached a page that is unavailable for viewing or reached your viewing limit for this book. aa You have either reached a page that is unavailable for viewing or reached your viewing limit for this book. aa You have either reached a page that is unavailable for viewing or reached your viewing limit for this book. ‘THERMODYNAMICS It is the branch of science which deals with the quan- titative relationship between heat and other forms of ener- gies. Thermodynamics does help us to predict the feasibility of a chemical process and also the extent to which it occurs underparticular set ofconditions. However, thermodynamics cannot help us to know the speed or mechanism of the ‘chemical process. Thermodynamics is primarily based on three fundamental law: Tet Law of Thermodynamics Deals with equivalence of different forms of energies. 2nd Law of Thermodynamics Deals with direction of the chemical change. 3rd Law of Thermodynamics Deals with evaluation of entropy. ‘SOME IMPORTANT TERMS System. It is the part of universe selected for ex- perimentation. Surroundings. The part of universe other than the system is referred to as surroundings. Surroundings and system are separated by real or imaginary boundaries. ‘Types of System System capable of exchanging mass as well as energy with currondingg is ealled open aystem. ‘System which can exchange energy but not mass with surroundings is called closed eystem, System which can neither exchange mass nor energy with surroundings is called isolated system. State of System. The conditions of existence of the system when its macroscopic properties have definite values refer to the state ofthe system. State functions. The thermodynamic parameters which depend on the initial and final state of the system but independent of the path are called siate functions. Thermodynamic process. The operation which brings about the chango in the stato of the system. Various thermodynamic process are : (@ Isothermal process.The process occuring at cons- tant temperature (AT = 0, AE = 0). (ii) lsochoric process :The process occuring at con- stant volume (AV = 0). (Gi) Isobarie process. The process occuring at constant pressure (AP = 0) (iv) Adiabatic process. The process in which no heat enters or leaves the system (fq = 0). Cyclic process. A process in which, the system ater undergoing a series of changes returns to the original state, AByee = 0; Hye = 0 Reversible process. A process that follows the r versible path, i.e, a process occuring in infinite num- her of steps maintaining equilibrium conditions at each step. In a process fellowing the reversible path, the opposingforce is infinitessimally smaller than the driving force. Irreversible process. A process in which the changes are producedrapidly so that system nolonger remains in equilibrium conditions. All, netural processes a irreversible. MODES OF TRANSFERENCE OF ENERGY Extensive Properties. Properties of the system which depend upon the quantity of matter contained in it. Examples are : mass, volume, energy, heat- capacity, enthalpy entropy free energy, ete. Intensive Properties. Properties of the system which are independent of the quantity of matter con- tained in it. Examples are : pressure, temperature, density, apecific heat, viscosity, surface tension ote. 107 ‘Various modes of transference of energy between system. and surroundings are : @ Heat (q). This mode of transference occurs if there is a difference of temperature between system and surroundings. Itis a random form of energy. Itis path dependent parameter. aa You have either reached a page that is unavailable for viewing or reached your viewing limit for this book. aa You have either reached a page that is unavailable for viewing or reached your viewing limit for this book. aa You have either reached a page that is unavailable for viewing or reached your viewing limit for this book. (CHEMICAL ENERGETICS AND THERMODYNAMICS STANDARD FREE ENERGY OF FORMATION (4G"s) ‘The free energy change taking place during the for- mation of one mole of the compound from its constituting clemente at the standard state. + AG*;of free elements at their standard state is taken to be zero. + Standard free energy change (4G* is related to the standard free energies of reactants and products as, AG" = E AG"; (Products) ~ 2 0G"; Reactants) ‘AG? is related to AH® and AS® of reaction as ese | aG°=aH°- Tas + AG? is also related to the equilibrium constant (K) as ‘AG =— 2.303 RT log K. THIRD LAW OF THERMODYNAMICS ‘The entropy of a substance at absolute zero (zero kelvin) is taken to be zero. i | The importance of law lies in calculating absolute ‘entropies of pure substances at different temperatures. This is possible by knowing the values of heat capacities (C,) at number of temperatures between 0K and TK. 7. T ss- J, cpdint=2.000 | cpa tee. ° The plot G, vs log T gives the value of integral. ‘p * It should be noted that this law is true only for the substances which exist in perfect crystalline form at 0K. However, if there are imperfections at 0 K, then entropy will be larger than zero. © Glassy solids even at 0 K has entropy greater than zero. ‘Solids having mixtures of isotopes do not have zero entropy at 0K. For example, entropy of solid chlorine is not zero at 0K. ‘© Crystalsof CO, N20, NO, HzO, ete. donot haveperfect order even at 0 K thus their entropy is not equal to aa You have either reached a page that is unavailable for viewing or reached your viewing limit for this book. aa You have either reached a page that is unavailable for viewing or reached your viewing limit for this book. aa You have either reached a page that is unavailable for viewing or reached your viewing limit for this book. (CHEMICAL ENERGETICS AND THERMODYNAMICS 48. 49, Bi. Given that, HO) —> Hs0(@) ; AH = + 43.7 ka H,0(6) —> H,0() ; AH = + 6.05 kg ‘The value of AH, attimation OF ie is (04975 15 mot (0)31.65 4 mot"? (©) 43.7 ke mot (d)~43.67 kJ mol"*, Given that, 2 Sy(s) + 60 —> 4S03(@); AH =~ 1580 kJ the enthalpy of combustion of sulphur is (6) ~ 1590 5 mot”? (0)-8180 4 mot (0) + 1590.45 mol”? (d)~798 i mot” Which of the following is an extensive property ? (o) Temperature (oy Pressure (@Mass/Volume ratio d) Energy. Which ofthe following represents the largest amount of energy? (@) 1 Bleetron vat (18g (@ 1 Calorie (@ 1Joute. Which of the following is correct for an adiabatic process ? @aB=¢ (ya1=0 oqztw @rav=0, 48° and SH? for combustion of methane are 186 JK-? and — 74.8 kJ mot”? respectively. The value of ‘AB® for the process would be (a) unpredictable (0) (14.8 - R)JE* Taras (dy (096R- 748), For which of the following procoes AS is > 0 {) MgCO3is) —+ MgOl) + Cx) AQ) Le) (ONa"@)+ Clg) —> NaCl) (6) Hy) — #2065). A sysiem which can exchange energy but not matter with the surrounding is classified as (6) closed system (6) microfinedaystem, {© open systern (cd) hoterogencous system. Given the reaction COs) + Hale) —» CO(g) + HOG) ; AH = 40 kd. ‘The AH is specifically called (c) Heat of formation of CO (©) Heat of combustion © Hew ofreaction (@ Heat of hydrogenation of € =O bond ICHT (eq) + OFM(aq)—+ H,0() ; AB heat liberated for complete neutralisation of 1 gm- molecule of HpS0, in aqueous solution will be @)-97.0KI (0) 2K Kg xe (a -xAvs. What is correct about heat of combustion ? (a) It is +ve in some cates while - ve in others. (b) tis applicable to gaseous substances only. in always— ve. ‘The net energy change in a reversible, cyclic process: is (a)32RT (b) zero (@aways>0 (a) always <0. Which of the following expressions represents Kirchhoff equation ? (a) Kx Ar BRT (0) Benn = SEE tag 2 alla ~AFy oR (@) AH — AB = nk. The mixing of gases is generally accompanied by (a) decrease in entropy (b) decrease in free energy (e) change in hoat content (c) increas in free enersy. For a reaction, 2CgHg(l) + 1502(¢)—~» 12COxg) + 6HzO() the value of gp) — acy) will be (at 25°C) @)-748%5 ) +3.701 +743kI (2) unpredictable as vahues of AH is not given. XY, Xp and Yo are diatomic molecules if AH x, ‘AHy_y and AHy_y are in the ratio of 2: 1: 2 and enthalpy of formation of XY fram Xz and Ya is ~ 100k mol” !. The value of Hy -x in kJ mol” is (a) 200 @) 400 (e)250 (a) 500. Onelitre-atmosphereis approximately equal to 1019 wsaus (asi d (a) 19.25. Ba” (aq) + 80,2" (ag) —+ BaSO,(s) :AH =~ 22.4 kJ. ‘The heat change represented by above equation is, called aa You have either reached a page that is unavailable for viewing or reached your viewing limit for this book. aa You have either reached a page that is unavailable for viewing or reached your viewing limit for this book. aa You have either reached a page that is unavailable for viewing or reached your viewing limit for this book. (CHEMICAL ENERGETICS AND THERMODYNAMICS 9 a 4 Choose the correct answer from the four alternatives given in each of the following questions: ‘The heat of formation of Cl” \aq) can be represented by the equation (@) Ogi) — 201 tap Lowe) +e" ore) oh cige) re 40g —9 te) (a) HCV) + ag —+ HT (aq) + CT (ae). The AHtomisation Of Hy) is 436 kJ mol? while AHy of HOG) is~ 241.81 kJ mol”. The ratio of the energy yield for the combustion of hydrogen atoms to steam to the yield for combustion of equal mass of hydrogen molecules to steam is (@)1.80 @)2.80 ©0585 (a) unpredictable because AHo «0 is not given. For the process 2F(g) — Fa(g), the sign of AH and AS respectively are (@)+,- s+ ©-- @-+ The free energy change occuring during the irre- versible process is (a) negative ©) positive (€) dependent on whether the process is exothermic or end- thermic (a) negative only ifthe process is exothermic. Given that AHsomb, of cyclopropane as ~ 4000 kJ mol” }. The amount of cyclopropane that needs to be ‘burnt in oxygen for producing 2 x 10° kJ of heat is (a) 20 kg 2A kg jue (a) 210me. In certain chemical process both AH and AS have values greater than zero. Under what conditions, the reaction would not be spontaneous LaH>Tas ILTaS-aH>0 TIL. aH = Tas. WV.aG<0. 10. 1. 12. in 1m (om,1V (@) only I Which of the following processes proceed with in- crease of entropy ? 1 solid changing into quid IL expansion of gas Il. dissolution of solid in iquid 1V. polymerisation. (@)Oniyt LMM onm omy. ‘The heat of reaction depends upon (c) temperature of the reaction ) physical states of reactants and products © botha and’ (q) path of the reaction and temperature. ‘Which statement is not applicable to thermochemical equations ? I They represent chemical change as well as heat change TL They give the idea about mechanism of the process TI. They depict ihe spontaneity ofthe process 1V, Bxathermic or endothermic nature of reaction. (n,1v omiv emt @u,m. Which of the following process shows negative values ofS? 1 HQ0—> H20%) Tl 30y(¢) 420340) TIL H;0)—+ Hye) _1V. Hard boiling of egg. nn (©) Only tH om @) Nene, Inwhich of the following process entropy increases ? L Rusting of iron IL Vaporisation of Camphor TIL Crystallisation of sugar from syrup IV, Atomisation of @i0 100 Liv @ Only IV. ‘The heat evolved during the combustion of 46 ¢ ethanol in a bomb calorimeter was determined to be 670.48 kcal mol” at 25°C. The value of AE of the reaction at the same temperature is aa You have either reached a page that is unavailable for viewing or reached your viewing limit for this book. aa You have either reached a page that is unavailable for viewing or reached your viewing limit for this book. aa You have either reached a page that is unavailable for viewing or reached your viewing limit for this book. CHEMICAL ENERGETICS AND THERMODYNAMICS 87. a. ()isobarie procest (2) isobaric as well as isothermal. ‘The value of AH? for the hypothetical reaction, AgB(s) — 2A(s) + 3Bp(e) is + 7.3 kd The value of AE* for the reaction is (a)> 73%) O< 73d @=73%5 + (@unpredictable. Which of the following io not required to calculate the value of AH' of C)Hytg) ? (a) alo C02 (aH yor B00) (6) AH timation of diamond (2) AF" of CHG). For which of the following process will AH be + ve? (a) Hole) + Ig) —> 2HIyy) (6) C+ 02—» UOg (d)803+ 10 —+ 03 Enthalpy of combustion of liquid benzene is ~ 3264.6 kJ mol”. The heat produced by burning 3.9 g of benzene is 00 +b0,—4003 (o BSS as (2048 is 288. aes, (4) 3264.6 bs. 39 Which of the following relation is incorrect ? (a) AS = AH «TG as «HE AG (-4G==Wonexp. (Wey ==P av. For an endothermic spontaneous reaction which two parameters are greater than zero. (@) 4G, ait (0)48, 46 (oa, a8 ()AG,aE. Which of the following can give the value of AH ? (a) aG-+ Tas (aE + Pav (e) bath (a) and () (AB AnkT. ‘An isothermal process proceeds with the absorption of heat q bat does not involve any change in volume. The value of AP, however, is greater than zero. Then g represents (a) ak, ag (08+ Pav (aya8 + Vor. @ gram of ethanol was-subjected to combustion in bomb calorimeter producing x Joules of heat then, @) ABcomh, =~ 285 mol? (6) aE comb, *~ 84 Smet? n. J mt (AE comb, (@) Alleoms, Under certain conditions, the value of AG” for a hypothetical reaction, X + Y= Z is greater than zero, then (@) The reaction has tendency to procecd towards Z (©) The reaction has attained equilibrium (©)Ineroasein temperature increases the yield of product 2 (dX and ¥ predominate in the reaction mixture. For a hypethetical reaction, 2A(g) + SBlg)—> AgBale) the value of AH?=~z kJ. Which of the following ‘expressions can help in calculation of AS° ? (a) S°qyp4 488% 28, We TG ace x-aG os wae For which of the following, the standard enthalpy of formation is 0.0 kJ? (a) Brzig) @) Bred ole) (a Br, ‘Standard free energy of which of the following is zero (a) O36) ig) 02) (d) both Op and 03, AB, Ap and By are gaseous diatomic molecules. If and AH’yof AB from Ay and By is ~ 100 kJ mel”), then AH, of A@) is (a) 200% mol”? (6) 100 a mot * (€)50 KS mot” + (@) 300 kJ mot” Which of the following chemical processes proceeds towards increase in entropy ? (a) Cats) +} 02+ CaO) (0) Cg) + Oxe)—> CO2te) LO NgQ) + O24) —> 2NO) (d) CaCO sls) —+ Ca0(a) + COE), In which of the following procses, the total entropy change of the process is greater than zero (a) Reversible (@) Reversible and exothermic. aa You have either reached a page that is unavailable for viewing or reached your viewing limit for this book. aa You have either reached a page that is unavailable for viewing or reached your viewing limit for this book. aa You have either reached a page that is unavailable for viewing or reached your viewing limit for this book. CHEMICAL ENERGETICS AND THERMODYNAMICS 27 ps 10. oo) ()CS~C*C) CCN 108.0 ‘QUESTION BANK (Level I) ie ie, t@ oo so km eae mer ne 8 4e We meee 16 Be) 20M) kw) RO) ®t) 20) he) 26) 8) HH) — BL «BRD me | KO BO 8H) BRB a) ae =e oy 4 eke ao ome She Sk BRB) BBD BT.(c) 58. (a) 59. (6) 80. (a) 61.(c) 62. (c) 68. (a) 64. (a) 65.(d) 66. (c) 67. (6) 68. (c) 69. (a) 70. (d) ‘TL. (b) 72. (b) 13. (c) ‘74. (d) 5. (b) 76. (c) 77.6) 78. (b) 79. (a) 80. (c) ae | ees m0 wrk) wk) cau) one) mL) Be) me me) %60) 100.6) 10h.) 10) 108.) 1K) 1050) 108,@) 0) 108.0) 100) 110.0 HINT: Bu COT) QUESTION BANK Level D | (@) AH ombuston of carbon is same as AHyof CO2. (d) Feoezing of water involves decrease in randomness. (®) TS refers to the random form of energy, which is non- railable for conversion into work. (©) AG? = 2.303 RT log K or - 2.908 R log (Ky) = 2.808 x 8.314 x 298(- 9.80 . Ae ed = 55.9 ks. (®) Bndothermie procats can be spontanceus only ifAC <0 and Steal > 0. (a) For one mole of H20(@) ; For2 mal of HaO(), Ax = 2x (-0.044) x 10003 = -88J. (a) $is zero only at absolute zero. (a) Entropy eriterion of spontansity is AS raat 0. (d)Inadiabetic process, noheat entersor leavesthesystem. (©) Multiply equation (i) by (2), eqn. (i) by 2, eqn. (it) by 4 and add to get the final equation. 11. (6)Factual question based on definition of state function. 18, (303i. — Ogle; ‘AB reaction = AH, (CoH) ~ 84H, = 85 ~ 9(230) = 606 kJ mol”! benzene. 18. (©) Thisis in accordance with Hess's Iw. 16. (€)For spontaneity AGr, p< 0. 17. (©) 4Hp refers to formation of 1 mol of H20(0) ‘Heat given out for 2 m¢ kd Heat given out for 1 mal = 3 kd. 19. (©Dissolution of NHClis endothermic: NHCls) + aq —+ NEG (eq) + CF (aq): AH= + v0. Other processes, being combustion processes are exothermic (qd) By definition, aS = 5% . Since 80 for adiabatic reversible process. Hence a8 =. aa You have either reached a page that is unavailable for viewing or reached your viewing limit for this book. aa You have either reached a page that is unavailable for viewing or reached your viewing limit for this book. aa You have either reached a page that is unavailable for viewing or reached your viewing limit for this book. aa You have either reached a page that is unavailable for viewing or reached your viewing limit for this book. aa You have either reached a page that is unavailable for viewing or reached your viewing limit for this book. aa You have either reached a page that is unavailable for viewing or reached your viewing limit for this book. aa You have either reached a page that is unavailable for viewing or reached your viewing limit for this book. aa You have either reached a page that is unavailable for viewing or reached your viewing limit for this book. aa You have either reached a page that is unavailable for viewing or reached your viewing limit for this book. aa You have either reached a page that is unavailable for viewing or reached your viewing limit for this book. aa You have either reached a page that is unavailable for viewing or reached your viewing limit for this book. aa You have either reached a page that is unavailable for viewing or reached your viewing limit for this book. aa You have either reached a page that is unavailable for viewing or reached your viewing limit for this book. aa You have either reached a page that is unavailable for viewing or reached your viewing limit for this book. aa You have either reached a page that is unavailable for viewing or reached your viewing limit for this book. CHEMICAL EQUILIBRIUM 7. 98. L 4 ‘The solubility of AgCl is maximum in (@) Pure water (@)0.1 MNHs (01 MNacl (a)0.1M CeCly, Which of the following is true for the saturated solu- tion of sparingly soluble PbI, ? rar) wR © Kp oie XaY(e) the amount of Xs¥ at equilibrium is affected by (@) temperature and pressure () temperature only (0 pressure only (@) temperature, pressure and catalyst A buffer solution cannot be prepared from a mixture © @) sodium acetate and acetic acid in water (©) sodium acetate end hydrochloric aid in water (© ammonia and ammonium chloride in water @) aramonia and sodium hydroxide in water. The concentration of (H“] and concentration of (O71 ofa 0.1 M aqueous solution of 2% ionisod weak acid is fionic product of water = 1x 10-4) (@) 0.02 x10"? Mana 5x 10" 1x10" Mand 3x10" (24107 Mand 5x10? (2) 9107? Mand 4107. For the reversible reaction, Nog) +3Hy@) == 2NHy@) at 500°C, the value of Ky is 1.44 x 10°5 when partial pressure is measured in atmospheres. The corresponding value of K, with concentration in mole litre” ia (@) 1.44 1075/(0.082 «50072 ©) 144 x 10°58.914 7737? (9144 107 5/¢0.082 773)" (@) 144 x 1075/0,082 x73) ®, ‘When two reactants, A and B are mixed to give products C and D, the reaction quotient, Q, at the initial stages of the reaction G)is ero () decroases with time (is independent oftime (a) increases with time, aa You have either reached a page that is unavailable for viewing or reached your viewing limit for this book. aa You have either reached a page that is unavailable for viewing or reached your viewing limit for this book. aa You have either reached a page that is unavailable for viewing or reached your viewing limit for this book. CHEMICAL EQUILIBRIUM 151 HINTS/SOLUTIONS QUESTION BANK (Level I) L@ PCis(g) = PClg(g) + Clog) Cone. 0.52 0.512 0.252 2 ii 01 02) (02 [S02!"(O2} _ (05/2)? 0.25/2) iltbrium cone. a 92) (92 = SOal 102) (05/2) (025/) _ o 195, Pes (%) (a8) (30) rT fy os - PCIsICH _ 0,020.02) 16. @) Kast 21x10 TPOIsl an, "1 5x10 2 17, (@)aG?=-RTIn K=-2.303 RT log K 2 a Ky= 2 When K = 1, log K= 0. POO, 18. (d) Le-chatelier’s principle. ‘ 19. (@)Forendothermicresctions, the valueofK increases with 8 (SOxXe)+ > Ore) = SOste) Ky increase in temperature. 2 « 10-82 = 10-22 28d) Ky = 1,0"? = 0-8? = 10 SOs) <= SOze)+ 2028) “Kh 21. (c) pl 5, POH = 14— 5 0H 49°85 280ai¢) 2 280xg) + Onte) qi (0H = 1078 = 107 * 22. (c) Kgp =82, 8 = Vp = VI0" = 107? mol 7? ee 2107100 g1- "= 10617 4. (0 Forexothermic reactions the value of Kdecreaseswith 28. (6) Total volume = 300ml, increase in temperature while for endothermic reactions 1 vappsels the value of { increases with increase in temperature. ‘mmoles of HCL» 300. 55+ 100% a99 5. (©) The valle of K is characteristic of the reaction at a 5 en " particular temperature TEDL egg LORIN BTEC] 4 (© Catalyst doos not disturb the equilibrium state saeaeeeemr dene % (@ Kpis constant at the particular temperature (q) It i the conjugate base of weakest acid HCIO. Acid & (y= Ke (RT)™. Here An =0. strength of oxy-acids ofehlorineis inthe order 9, (a) Here, number of moles of products is less than the AO 5 Ba SACs = BON number of mole of reactants 25. (©) Acid having least pia value isthe strongest acd 10, (@) Here the reactioa is exothermic and hence increase in 26. () AlCIs is electron deficient. It cannot donate a pair of temperature would shit the equilibrium in backward elec trons. ‘dieeetion: 27. (d) HCOO™ cannot donate a proton. It can only accept a ban Dad proton 12 (d) Doubling the volume would reduée the pressure and A G2) Bwincreanes with incense in teriperature, shift the equilibrium in that direction where number of 9, (¢) a= VKa/O ark, = Co? ‘moles is more so that effective decrease in pressure is VES NERC ces = Ce: ‘undone to some extent 30. (a) His the conjugate baseofH which isthe weakest acid 18, ‘@)K depends upon temperature. ‘among Ho, HCI, HNO3 and CH;04. 18. (6) 2303@) = 280z(¢) + O2@) ut Atatart : 2 No. of moles 1 0 0 Bh HOOy COs ee Aca Conjugate base No.ofmoles 05 05 045 aa You have either reached a page that is unavailable for viewing or reached your viewing limit for this book. aa You have either reached a page that is unavailable for viewing or reached your viewing limit for this book. aa You have either reached a page that is unavailable for viewing or reached your viewing limit for this book. CHEMICAL EQUILIBRIUM 1B au (d) The value of K depends on temperature only (Far a particular reaction). @ 28034) <= 280x8) + Oxi) Initial cone. Amol“! 0 ° ium O4molL"? 06molL"! 0.2 mol L~! soni 2 0210 x. SHPO 06703) . 46, {SOs} (o.sy (0) 0 0t conetan temperature and preseere fe) 28g) <> 2Hye) +826) Bautibrimene 2 oa 98 nat 4 Hal? IS21 0.10.4) 016. isi? 0.5? - 2x 1794 AG? = - 2.803 RT log K log K= +0.5959 or K=39. (@) Sodium borate is a salt of strong base and weak acid. (d Since K,, and K,, are equal solution would be neatral. 1 PH =3 InKw + PK, ~ pK (a) LigNaglAtF@(s) <= Li" + BN" + ZAIRE Bs Ky = 1a" 7 1a" 18 tat? |? = 9 (30)9 ae)? = ores SOcIt ‘al Arrhenius acid due to—SOgH and Lewis hase due to NH group. NH Ia] (a) pit pty tog fat Increasing the conc. of salt would increase the pH. id) Has = Se 2H" On ailditio of dil. HC, (2 ] increases and shifts the above equilibrium in backward direction and hence (S?-} decreases. (a) p= pK, + ox S8t when (Slt = [Acid pl = Ky (Doubling the volume would result in less moles pe litre. In order to undo the effet aecording to LeChatelier’s 36. a7. 38. 40. a 42. Bs 45. at 48. 155 principle, the equilibrium shifts in the direction where number of moles is more, (©) pH = ~ log {Hj0° | = ~log x when cone. is increased 10 times BH =~ log 10x = ~ flog 10+ log xi = loge (b)Inereasein pressure shifts the equilibrium in the direc tion where number of motes of gaseous species isles. (b)Itisam exothermic reaction AH = ~ ve It is a spontaneous reaction AG = ~ve No. of moles of gaseous species is ine (0) Kep = Lag? CT | = Ag? © A+ Bo w+ D Initial «= 220 (nol Ly Atequii: 10 20° 02 Od rium 2 2 (CPD) ©2759, IIB? ayer @) BHT = Hel, ~~ 2 © 0 Initial cone. (Suppose) 1 05. G5. ..Banilibrium cone Hallie) 05x05 _ gos ont (c) Temperature would inerease due to shift of equilibrium in the forward direction. (a) Addition of product at equilibrium shifts the reaction in backward direction fe) PoCly = Pb*+2cr sb Bp = (PH (Cr PP = tayo? =? £= Kyo (a)Sulphidehaving ower value ofK,, is precipitated frst. 21 = Hye) + hw 2-2e ow «Moles at equilibrium ‘Total no, of moles =2-20+a+a=2. (6) Apply Le-Chatelier’s principle (a) N20ae) = 2NOxe) (1-0) "04 Motesateqtrium Talon oftmole= 08 04-32 (0 py +g SAL wp, gS hg = OA + wo 2 20” 3020 Inia moles aa You have either reached a page that is unavailable for viewing or reached your viewing limit for this book. aa You have either reached a page that is unavailable for viewing or reached your viewing limit for this book. aa You have either reached a page that is unavailable for viewing or reached your viewing limit for this book. REDOX REACTIONS AND ELECTROCHEMISTRY 159 Am=Aq— be, b being constant which depends on temperature and nature of solvent. ‘Ase approaches zero ; Am approaches Ax, ie, molar conductivity at infinite dilution, The values of equivalent or molar conductivity at infinitely dilute solution are called limiting equivalent or molar conductivity and are represented by Ag; oF Ain * ‘The values of Az, for strong electrolytes can be deter- mined graphically by extrapolating the plot of Are Us. Ye. Whereas the same for weak electrolyte cannot be determined graphically. KOHLRAUSCH’S LAW Itstatesthat at infinite dilution, when the dissociation ofelectrolyte is complete, cach ion makes a definite cantribu- tion towards the molar conductivity of electrolyte, irrespective ofthe nature ofthe other ion with which it is associated. ‘+ The molar conductivity of an electrolyte at infinite dilution can be expressed as the sum of the contribu- tions from its individual ions. It 27 and 22 represent the ionic conductivities at infinite dilution. Then Rete AT where 1, and ¥. represent the number of + ve and ~ ve ions furnished by each formula unit of the electrolyte. ‘+ Kohlrausch law can help to calculate : ( degree of ionisation of weak electrolyte (a) An An (i the value of Nj for weak electrolytes and iii) ionie mobilities. ELECTROCHEMICAL CELL It is the device in which the decrease of free energy during the indirect redox reaction is made to appear as electrical energy. ‘TYPES OF ELECTRODES ‘The various typos of electrodes which are frequently used in the electrochemical cells are described as follows 1. Metal-Metal ion electrode. It includes a metal strip dipped in the solution of its own cations. Some examples are Zn /Zn™, Cu/Cu™, Ag/Ag’, ete. 2. Amalgam electrode. It is similar to metal-metal ion type electrode, but here, metal is replaced by its ‘amalgam with Hg. This is done to improve the activity of metal. Zn -Hg/Zn* is a common example. 3. Gas electrode. It involves the inert metal foil like Ptimmersed in the solution capable of furnishing ions of gas. Pt, Hy/H" and Pt,Clz /CT are common ex- amples of gas electrodes. 4. Metal-Metal insoluble Salt-Salt anion. This type ofelectrode include metal in contact with itssparingly soluble salt and aqueous solution of some salt contain- ‘ng same anion. Some examples are : ‘Hg - HgyCl{s) - KCI (aq) Calomel electrode, ‘Pb — PbSO4{(s) ~ KySO, (aq) and Ag-AgCl(s) KCI (aq). 5. Redoxelectrode. These clectrodesinclude aplatinum wire dipped ina solution of mixture of the two salts of, the same motal but with different oxidation states. ‘The common example is Pt, Fe™*/Fe™, 6. Standard Hydrogen Electrode (SHE). * The standard electrode potential of SHE is taken to be * The SHE can act as cathode @H* + 2° —+ He) as well as anode (Hz —+ 2H" + 2e") electrode. ‘+ Pt. foils used in SHE becauseit is very good absorber of Hy gas. + The pif of acid solution in SHE is zero. CONCEPT OF ELECTRODE POTENTIAL The tendency of an electrode in a half cell tolose or to goin electrons when it is in equilibrium with the solution of its ions is called half cell electrode potential (E). ‘+ The reduction potential of the electrode at standard state conditions (1 molar conc. of the ions, 298 K and J atm. pressure) in called standard reduetion Potential (E"), +E is an intensive property like temperature or molarvolume. This means that E is same for half cll, reaction whether it is sepresented as 2X +267 3 Xp oF Xt +07 4%, + E* values are not thermodynamic functions and may not beadded directly. However, they can be calculated from free energy which is extensive property. For example, Felt se 5 Felt By = 0.77; AG? = 10.77) F Fe + 20° Fe Ey" =~ 0.44V ; AG2? = 2-044) F Add Fo®* + 3e°"—+ Fe; E*# Ey? + Ea? : aa You have either reached a page that is unavailable for viewing or reached your viewing limit for this book. aa You have either reached a page that is unavailable for viewing or reached your viewing limit for this book. aa You have either reached a page that is unavailable for viewing or reached your viewing limit for this book. REDOX REACTIONS AND ELECTROCHEMISTRY 163 16. W. 18. a. ‘Oxidation number of carbon in diamond is, G4 yea (oo (+2. Which of the following redox reaction is feasible ? (a) Za(s) + 2Ag* (aq)—> Zn™"(aq)+ 2Ag{s) (0) Zn™"(aq) + 2Ag(s) —s 2Ag"(aq) + Za(s) (©) Za(s) + 2Ag(6) —+ 2h4"(aq) + Zn"(ag) (d) Zn (aq)-+ Ag"(aq)— Ag(s) +Zn(s). ‘The species undergoing reduction in the following reaction is Yrs 98,0 + ocr — Cr+ + 30r+ 60H Cr 6) HzO wc @er, In the following equation : C103 + 6H" +X—+ CT + 3H20. Xis wo ee 0, ase. ‘The oxidation state of phosphorus in barium hypo- phosphite [Ba(H,PO2) is +a (+t ‘The correct oxidation number of phosphorus in mag- nesium pyrophosphate [MgsP207) O42 1 (eso ea (3 (a) +5. In the reaction ; Cl, + 60H” —+ SCI” + C105 + 3H20, chlorine is (@) reduced oxidised (©) oxidised as well as reduced (@) neither oxidised nor reduced. Inthereaction ; Iz + 503 + HgO—+ 2H1103 + 60z, the substance undergoing reduction is (@ Iodine ©) Orane (©) Water (4) Iz as well as HO. Conversion of CyaHy2011 (sugar) —+ COs involves (a) oxidation of sugar () reduction of sugar a1. (© oridation ofoxygen (ed) both oxidation of sugar and reduction of oxygen. 2Mn0j + 6H’ + 5803+ 5SOF + 3H20+ X In the above equation X stands for (a) Ma? 20a? fe Mn0f (a) MnO). In the equation Crg0F + Fe + Ht —> Or** + Fe +90. the coefficients of Fe” sind H’ are respectively @6,7 (06,14 5,7 (a514 ‘KMnO, oxidises C,03~ to CO, andeach two molecules of KMn0, gain 5¢" during the process. The number of moles of KMn0, required to oxidise 126 g ofoxalic acid (ig0204 - 2420) is (oa woe os 10. ‘The number of moles of KMnO, required to oxidise one mole of ferrous oxalate in acidic medium is @os (04 @o2 (ay.67, The oxidation states of most electronegative element in each of the species BaOz end Na,SO, are respec- Wr? (a-2,-2 Which ofthe following reaction is not a redox reaction ? (a) 802 +2H8-—+ 2H,0+8 (b) Na +0g—+ NagOz (©) NagO + HyS04—+ NagS0q + Hg0 (d) AK C103 — SKCIO, + KCI. What is correct about Fe,(Fe(CN)gls ? (a) All the iron atoms have same oxidation number (0) ON. of iron atoms outside the square brackets is + 3 +2 (ON. ofiron atoms outside aquare brackets is (d) ON. of ron atoms inside square bracksta is + 3, Oxidation number of sulphur in Hy80s is (a)+2. +4 (ose +6. aa You have either reached a page that is unavailable for viewing or reached your viewing limit for this book. aa You have either reached a page that is unavailable for viewing or reached your viewing limit for this book. aa You have either reached a page that is unavailable for viewing or reached your viewing limit for this book. aa You have either reached a page that is unavailable for viewing or reached your viewing limit for this book. aa You have either reached a page that is unavailable for viewing or reached your viewing limit for this book. aa You have either reached a page that is unavailable for viewing or reached your viewing limit for this book. aa You have either reached a page that is unavailable for viewing or reached your viewing limit for this book. REDOX REACTIONS AND ELECTROCHEMISTRY 57. (©) Nernst equation for single electrode potential is EE Nog aye Beet Top ox (a) The efficiency of HzO; fl collin 70%,“ ‘A gas X at ono atm. pressure is bubbled through the solution containing mixture of 1M Y and 1M Z at 25°C. Ifreduction potential of X < ¥ < Z, then (a) will exidise X and not Z (BY will oxidise Z-and not X (OY will oxidise Z and X. (@)Y will reduce both ¥ ana Z. ‘Thermodynamic efficiency of cell is given by KE (@aH/G one =nEP : = (ane. E*yityi ©— 0.25 V; BYqu%/ay = 1.60 V. The value of F°qu for the coll Ni | Ni? (1M) Au®* (1 MD | Au is (@)125¥ 15 LTB @aov. ‘The electrode potential of hydrogen electrode at pH 10 is (@os1¥ 0.00 (0.58 (a)0.089V. Red hot carbon removes oxygen from XO and YO but not from ZO. ¥ also removes oxygen from XO. The above information is sufficient to provide the order of activity among the metals X, ¥, Z as @X>¥>z @z>¥>x (OY>X>% (hE>X>r. A cell is constituted by coupling the two electrodes ‘Sn/Sn™ and Ag/ Ag’, if E°s,/ga, E°ag'/ag and Boe are — 0.14 V, 0.80 V and 0.94 V respectively. The correct representation of cell is, (a) Sa/8a?* (1 MDI Ag (0.1 MAR. (b) Ag/Aal (MD Sa? 1 MAS (©) Sa/Sn2* (LNDH Ag’ CMV AR (a) Sn S02 (0.1 ND Ag™ CO Mog. ‘The measured olectrods potential for the reaction Pb” + 26° —+ Pb(s) does not change if (a) concentration of Pt* ions is increased in the sclution (@) size of ead strip is increased 61. 171 (@ temperature ofthe system is raised (cd) volume of solution is doubled by adding water. Given that E°4)*/al =~ 1.87 V ; E’Mg*/Mg = - 2.34 V5 B'cu""/Gu = 0.84 Vand By" = + 0.53 V, predict 1s to which ona is the bost reducing agent ? @al ly (Mg @Cu. Ifthe solution of copper sulphate in which eopper rod is immersed is diluted 10 times. The electrode poten- tial will (a)increaseby0.030V (6) decrease by 0.030 V (c)inerease by 0.059V —_(d) decrease by 0.059 V. Given four half cell reactions WAC 8 =-020V wor —c%; 25 (D+ op je = L258 (yD +22" —4 D**; B= 0.65 V Which combination would give a cell with largest cell voltage? (a) Amd B (Bande (Cand D (@)Dand a. Iewhich of the following cell, the value of Bay 20 but E%cen = 0. (a) Cu/Cu?* (1 MD Ag MVAg (6) Pt, Ha/H? (025 M) EH" (0.025 MY, Pt (0) Ag/Ag" (0.01 BD 1-Ag? (0.1 MYAg (4 Both in (6) and (c). Zine is generally coated over iron to prevent its cor- rosion because (a) zincis cheaper metal (©) zinc gives a good luster to iron OE p21 95> Byte EB ¢n 29? > Ep, pe ‘When a solution of sodium hydroxide is added to acetic acid solution, the conductivity oftheresulting solution (a)increases (6) remains unchanged (decreases (a) bocomes zor. Given E° values for some reactions as 1p +203 OF; E°= 054 V MnO," + 8H* + 5e° —> Mn* + 41,0 ; B= 52 aa You have either reached a page that is unavailable for viewing or reached your viewing limit for this book. aa You have either reached a page that is unavailable for viewing or reached your viewing limit for this book. aa You have either reached a page that is unavailable for viewing or reached your viewing limit for this book. ‘REDOX REACTIONS AND ELECTROCHEMISTRY 115 47 (a) Au + 36° —> Au 197 g of Auis obtained by charge = 8 96500 C 96500 x 1.284 197 1.294 g of Au is obtained by charge Now, 3x ¢ « 2% 98500 1.234 48, (d)Coll constant is not given. g B REE S 88 oi. (a) Resistance « = (c)Thestatement refers to Faraday’ second law of eletro- ysis Jt ay a volt impce “amp C* amp * amp (o)Reduction Means gaining of electrons. (© Cathodic reaction involves reduction. (a) X has least reduction potential and hence maximum tendency toaxiise. (@) Reaction in hydrogen slctrode aH 207 Hy 0.0595, Lat? “a PS ay ean be greater than E: if cone, of H*ions is increased. (d) Large negative reduction potential means large positive ‘oxidation potential for X° —> X + e”. Hence X” can be readily oxidived. (@)E isrelated to tomperature as (a Samp?-c0e = =ohm. EsE*+ (c)H* cone. should be 1M... pH should be zero. (a) Thereaction ly + 2B" —+2I" + Brgisnon-spontaneous because reduction potential of Brg/Br” is larger than that of e/T. (@) Beall = Ered (cathode) + B°gxi, (anode) 041 + 0.76 = 035 V. (@) EMF = EMF" + 2.058 (0.1) 42.088 jog SON? EMP + 9-088 6 o.0n8 63.) Mg™/Mg couple has reduction potential lesa than Hydrogen- electrode. Hence Mg can reduce Hons to Hy. 4, (a) The highest cell voltage would be obtained from the ‘electrode with highest oxidation potential and highest reduction potential beestse Regt = Baxi, + Ered 65. (d) Frye of zy, w areless than y. Hence allof them can reduce y* ions. (a) Choice itself isthe answer. (d) AgNOg + gO ———> Ag + H* + NOs" + 503 (PH gradually decreases due to increasein H' conc. Blectricity Licl+ Ho ———> i+ 08 + Lg + Hy *. PHL gradually inorenses dus to increase in OH ion 68. (a) Apply Nernst equation, to the resction Ph+Sn** > Ph?*+Sn 150) yy Sa) ee ge = antileg (03). or Be (sn) ey 69. (0) Cu + 22” _+ Cu ;E* + 0.94V 20.8 22 Iog a2 or HB =0.34- (00296) (9) = 0.34 0.266 oorv. 0. (a) cu? +e —9 Cat ; By 20.16 1x0.15 xF =-0.15F AG) =-1x050xF=-05F Cu? +267 — Cu; Bs =? BGs =-2xES xP =-283F ‘Also, AG's = AG", + AG") =- 0.15 F- 0.5 F =-0.65F Now-20P 2-065? o Pye eom5¥. 1. (@pHy=3 OFT, =1x107'M; pHy=8 2 = 1x10? Now. Beat = 295% jog X10. po59x2 0.118. re x10" aa You have either reached a page that is unavailable for viewing or reached your viewing limit for this book. aa You have either reached a page that is unavailable for viewing or reached your viewing limit for this book. aa You have either reached a page that is unavailable for viewing or reached your viewing limit for this book. REDOX REACTIONS AND ELECTROCHEMISTRY 179 36. (@) For tho reaction, 2H + 267 > Hy Bees 2082 pg ar? <0 298 ng a0"9?=-050. ‘87. (b) Choice is the answer itself. Reactivity depends on the tendency to displsce the other. 58. (c) Since reduction potential of Ag*/Ag is greater than Sn. Tees vr dredemt acts cbc ‘B9, (b) E° value will not depend upon the size of the strip as trem foe ake bey, 60, (he est vt agent i ne wwe En i owe. Thusigisewrredt choke, 61. (b)1f solution is diluted to 10 times, molar cone. of Cu is trond cne enh feng les ‘Nemst equation for Cu" + 2e" > Cu ie = + 2559 tog cu) ‘by decreasing the conc. of Cu** by 1/10, E value will decrease by 2888, appr by O08. ()BandCcabintin wil gveceliaage©2 0 which Smiter ee giee son (oer mnestratins clo Bog = 0 (2 fine wl ander oidion in preference rn (@) CHGCOOH ne wea At NSO 14600 cases oman of C{COONa wilh rag Slate. Hencecnductiniy wl nerese BEB B n. “ (a) Larger the value of Ezy larger in tendency for reduc- tion and consequently atronger willbe the oxidant Similar- 1y, smaller the value of Erg larger the tendency for ‘oxidation and consequently stronger will be the reductant. (@)In the given reaction X is gotting aridised to X"*. Hence for ‘his reaction to occur, the oxidation potential of X should, ‘be greater than hydrogen electrode (ie. 0). (@) Add the two values of electrode potential to get the desired value. (a) Bloctrical work =nFE" + For Daniel call n= 2 2 Work = 2 x 956002 1.10 2212.8x 109 J = 212.3 8s. () In XYZ; the sum of ON. ofall the atomsis zero. () ea ~ Bx, =0.67 or Ex, =-0.67 +028 =-0.39V. Now -039=0+ 2952 tog Ht or -log tt = 238, 6.6, @) M04? Ma0,- tia an oxidation process and occurs at anode. ‘The ON. of Mn increases from + 6 to +7. () Factual based on relationship. (@) Choice itseifis an answer. 8) AG* of required reaction = 2 x (- 21.52) = aa You have either reached a page that is unavailable for viewing or reached your viewing limit for this book. aa You have either reached a page that is unavailable for viewing or reached your viewing limit for this book. aa You have either reached a page that is unavailable for viewing or reached your viewing limit for this book. RATES OF CHEMICAL RRACTIONS AND CHEMICAL KINETICS 4 7. Choose the correct answer from the four alternatives given in each of the following questions : ‘The rate of reacticn, 2Nj0;—+ 4NOz + Op, can be expressed in terms of NOa a @-ANOe ‘auNOst 1 ‘Which of the following is true for the reaction No + 3H —> 2NHy ) XD MH 22), 1 a8 de @ a dig) AN aay _ al 25S, (ay Meg ‘The units of rate of reaction are (o) mat ytemot (ome? (a) mo ts For the reaction, A + B—+ C + D, the rate of the reaction may be expressed asr = - 241 The negative ‘sign in the above expression indicates that (@) rate ofthe reaction’ (b) cone. ofA is decreasing with time (c) cone. of A is increasing with time (@) none of these. ‘Which of the following reactions is the fastest ? (a) Busting ofiron () Burning of sulphur (©) Transition of monoclinic sulphur to rhombic sulphur (d) Precipitation of barium sulphate by mixing solutions of sodium sulphate and barium chloride. negative ‘The minimum amount of energy that the reacting molecules must possess at the time of collisions in order to produce effective collision is called (a) Activationenergy (8) Threshold energy (€) Internal energy (d) Free energy. A reaction has rate law expression as rate =k (Al? (BY? ‘The order of the reaction is 10. 1. 12. 13. 14. 15. @. 2 wosn @-v2. For a chemical reaction A> B, it is found that the rate of reaction doubles when the concentration of A is increased four times. The order of the reaction is (4 2 or wu. ‘The unit of rate constant for a third order reaction is (a)mol 1s"? (@)L mors? (mol? L-? =? @) 1? mol? ‘The dimensions of rate constant for a first order reaction involve (a) Time and concentration (@) Time only ()Concentration only (@Neither time nor concentration. ‘Which of the following simple reactions isa trimolecular reaction ? (@)Ng+02.—32NO (8) Hg + Ig 2H ()2NO+0,—>2NOg (a) NH4NOp—> Np + 21430. For a hypothetical reaction, 2A + B—» Products, the rate constant, his oqualto5.6 x 10” °Lmol”!s~!-The order of the reaction is, (@)One Taree Two (@) Fractional. One litre of 2M acetic acid is mixed with one litre of 3M ethyl alcohol to form ester. (CH,COOH + C2H,OH —+ CHjCOOC;H, + H,0 The decrease in the initial rate if each solution is diluted by an equal volume of water would be (a)4 times (#)2 times (0.5 times (2) 0.5 times. ‘The rate constant of a reaction depends on (a)cone of reactants @) cone. of products time (@ temperature. ‘An exothermic reaction, X—+ Y, has an activation energy 30 kJ mol”, If energy change (AE) during the aa You have either reached a page that is unavailable for viewing or reached your viewing limit for this book. aa You have either reached a page that is unavailable for viewing or reached your viewing limit for this book. aa You have either reached a page that is unavailable for viewing or reached your viewing limit for this book. RATES OF CHEMICAL REACTIONS AND CHEMICAL KINETICS 187 a1. (9404107? mel L (2) 8.0.x 10°? mol L? . The reaction, 2A + B —» Products, fellows the mechanism A= hy Ag+B— Products (Slow) ‘The order of the reaction is 2 oa 1 3 @1}. For which of the following reactions, the per cent increase in the rate of the reaction is maximum when concentration of the reactants is doubled ? (a)Zero order reaction _(b) Fist order reaction (c)Second orderreaction _(d) Third order reaction. For a reversible reaction, the activation energies for the forward and the backward reactionsare 60 kJ and 20 kJ respectively. If energy of reactants is 20 kJ the energy of products would be (a) 40S () 20k coeoKs ayeoxs. For the reaction, 24+ B+ C— Products the rate law equation is r= AlAl (BI¥? Ic}? ‘The rate ofthe reaction increases with increase in the concentration of (@)A only (6) Bonly (oC only (Aor, Which of the following statements is not correct for the reaction 4A+B—>2C+2D (a) The rateof disappearance of Bis one-fourth of therate of disappearance of A (ye rate of appenrance of Cis one-half the rate of Aiaapponrance of B. (c)The rate of formation of D is one-half the rate of disappearance of A. (dy The rates of formation of CandD are equal In the accompanied diagram, indicating Ez, Ep and Ep as the energios of ronctants, activated complex and products respectively which of the following is correct ? 67. E> Progress of reaction. ——> (a) Forward reacticn is slow. (0) Backward reaction is elow. (@) Reaction is exothermic. (d) Reaction is endothermic. ‘The activation energy for the forward reaction A+B—3C+D-20kd 30k. The activation energy for the reverse reaction, (8) 50 kal (e105 (-10ks, ‘The following mechanism is proposed for a resction, 2A+B+D+E A+B==C (Rast) A+C—+E+D (Slow) ‘The order of the reaction is @1 2 os ais. In several experiments on the kinetics of the reaction A+ B—> products, the rate is doubled on doubling the initial concentration of B keeping that of A fixed. ‘On doubling the initial concentration of both Aand B, the rate increased by a factor of 8. The rate equation is @)r=ktAl"B) (6) r= RIANBL @r=HAIB? (a) r= RIA? (BP. On increasing the temperature, the rate of reaction increases mainly because (@) the activation energy ofthe reaction increases @)alargefraction ofthe molecules attain ergy equal tor greater than the threshold energy (©) concentration ofthe reacting moleculesincreases (@) collision frequency increases. aa You have either reached a page that is unavailable for viewing or reached your viewing limit for this book. aa You have either reached a page that is unavailable for viewing or reached your viewing limit for this book. aa You have either reached a page that is unavailable for viewing or reached your viewing limit for this book. aa You have either reached a page that is unavailable for viewing or reached your viewing limit for this book. aa You have either reached a page that is unavailable for viewing or reached your viewing limit for this book. aa You have either reached a page that is unavailable for viewing or reached your viewing limit for this book. aa You have either reached a page that is unavailable for viewing or reached your viewing limit for this book. [ett ai nite an a we QUESTION BANK (Level 1) 16 2) aid) 40) 5d) 6.6) 70a) Bid) a@ 10.6) IL} 12.6) 13.) 14. 16.(c) 16.(d) 17.) 18.6) 19.) 20.(a) ad) 22.(0) 23.(d) 24.(€) 25. (d) 26.4) 27. (6) 28.(d) 29.) 30. (e) 31.0) a2) 33.6) 4. id) 38. (6) 36.) 31.6) 38. (d) 89.(d) 40.0) aL) 42.(0) 43.10) 44.00) 45.0) 46. () 41.(€) 48.) 49.(d) 150. c) 51.(a) 52.(d) 53.6) 54.0) 55.(c) 56. (6) 52.00) 58.) 159.0) 60.(c) a1. (a) 62.(d) 63.(a) 64.6) 65.(e) 66. (c) 67.(¢) 68.(a) 69. (6) 10.) nd Re 73.(a) 74.00) 78.12) 76.(@) Te) 0.(d) "79.(c) 0. (6). QUESTION BANK (Level I) L© 2a) 3.10) 40 5.0) 60) 7.0) 800) 2.0) 10.8) 1e@ 12.0) 13.6) 14.@) 18.(€) 16. (@) 12.0@) 18. (a) 19.) 20.4) aa 20 © 24.) 25.6) 26. (c) 27.1) 28,(c) 29.0) 30.(0) 31.16) 32.06) 93. (6) 34d) 35. (a) 96. (6) 37.06) 38. (6) 99.(d) 40.6) AL (a) 42.(d) 43.(¢) 4d) 45.06) 46.(0) A. (d) 48.(€) 49.(¢) 50.) 51.) 520) 53.6) 54.10) 55.(6) 56. (a) 52.0) 56. id) 50. 6) 60.06) 1.) 62.0) 63. (4) oa 65.(c) 66. cd) 67.(¢) 68.) 09. (d) 10.0) © 72.6) 73.) 74.6) 75.(a) 16.(@) na 18.0) 79.(c) 80. (a) BL.) 82.) 88.) 84.0) 85.(0, 5. (d)Itisan ionic reaction. 12. (6) By observing the dimensions of we come to know itis 7. (@) Order = 3/2 (= 1/2)=1. second order reaction. 3 ate CH,CODH CHO When each cone, is reduced to half, the rate of reaction would become ! in 0 beaRay- 10. hit onder rent has denon emia? ‘ae ete, -20= 30-Eayy) 11, (atl berernsin arsine racin By =O aa You have either reached a page that is unavailable for viewing or reached your viewing limit for this book. aa You have either reached a page that is unavailable for viewing or reached your viewing limit for this book. aa You have either reached a page that is unavailable for viewing or reached your viewing limit for this book. 199 RATES OF CHEMICAL REACTIONS AND CHEMICAL KINETICS lar 48. (6) AB = Bayy Bqyy) = 30-50 =~ 20K. (6) Because its apparent molarity is more than three, (c) From the dimensions of rate constant itis clear that the reaction sa zero order reaetion. It would proceed at constant rato of 10°? mol L~! «1. In 10 minutes (600 seconds) the amount of A reacted would be = 10°? x600mol L” = 6 mol 7? No.of moles of A left unreacted = 10-6 = 4 males. 62. (c) Higher the rder ofthe reaction, sharper is the decrease in the rate of the reaction with progress of the reaction (due to fall im conc), BS 54, (d) This is an ionic reaction v2 55.) ry = hea? Hy ras hese)? cape 2 a 56, (a) Here, water is present in excess, s0 its cone. does not affect the rate of the reaction as the reaction progresses, (@k(rateconstand) increases with inereasoin temperature whereas K (equilibrium constant) increases for endother- ‘mic reactions and decreases for exothermic reactions. ay Vag cy 59. (b) ry = Cay" (by 2 = k(2ay" (by rq=k(4ay" (40) iti) From (®)and Gi) 292" ot on *E fay" From Gi) and i) Bacay" am A But n=l 4=(4) 4)" or m=O. 0, ba Ae EW/RT oe pa ncPo/ RT 61. 67, 70. n. 2. 3. “4 1. 76. 2. 81 (&) Fora first order reaction f1 9 is independent of initial A _ ay? ahaa ~diAl =a Integrating from {Alp to (Al and (alo ~ tal (2 Conc. of both tho reactants sfter dilution would be reduced to one-half ofthe initial conc, (By observing the dimensions ofk, we come to know that the given reaction is of second order. re Rial 21 « 1072 ca? =4x107? mol Ls? (c) For third order reaction & has units L? mol”? s (&) Rate = HAIB) ‘When concentrations of both A and B become half, the rate ofreaction would become one-fourth of the initial rate. 1 0.693 0893 _ 6.99 10-2 min? “ria 7 100 = 698%10 t= 2908 tog Wo _ __2903 __ j,9 100 A 1Al 393x105 ™ 10 = 88 min. (©) Starting with 4 mol L”¥, the cone. was reduced te half in 1 heur whereas starting with 2 mol > "the cone. was reduced to half in 0.5 hour. Thus, t1-2 hero is directly proportional to the initil cone. of the reactants. Hence, order ofthe reactionis zero. (dy Rate woutd increase (2)!/? or times. Go Rate = RIA? 2 Rate 200 poy mote ta? oa? (6 A zero order eaton ake pleat constant rate an Seca ofA Goran wns atonasant ate ha, 4a @ 13S} “(3 ‘The rate vs. [Al graph would have slope equal to. aa You have either reached a page that is unavailable for viewing or reached your viewing limit for this book. aa You have either reached a page that is unavailable for viewing or reached your viewing limit for this book. aa You have either reached a page that is unavailable for viewing or reached your viewing limit for this book. ‘SURFACES AND CATALYSIS 203 QUESTION BANK Choose the correct answer from the four alternatives given in each of the following questions : 1. In adsorption of methane on charcoal, charcoal is, (a) Adsorbate ©) Adsorbent (©) Absorbate (@) Absorbent. 2. Size of colloidal particles ranges between (@) Lnm to 100 nm (©) 1m te 1000 nm (©) 10mm to 1000nm (@) 100 nm to 1000 am. ‘8. Which of the following is a homogeneous system ? (a) Muddy water (©) Vanishing cream (©) Cod liver oil (2) Sugar solution. 4. ‘Tyndall effect in colloidal solution is due to (a) Scattering of light (®) Reflection of light (c) Absorption of ight (d) Presence of electrically charged particles. 5. Butter is a colloid containing (@) Fat dispersedin water (8) Fat dispersed in oil (c) Water dispersed in fat (d) None of these. 6. Which of the following will have the least coagulating power for arsenious sulphide sol ? (a) Na* (Mg war (cet, 7. Which of the following is not true for physisorption ? (a) It is reversible (@) 1 occurs in the form of multimolecular layers (o)It needs activation eneray (@)It increases with increase in pressure. 8, Colloidal particles in soap sol carry (a) Negativecharge (©) Positive charge (ONo charge (a Bither positive or negative charge 9. Which of the following is not used for purification of colloidal solutions ? (@ Dialysis (oRitteration (2) Ultra-centrifugation (@ Blectrotialysss. 10. A colloidal solution of a solid as dispersed phase and liquid as dispersion medium is known as @Ga @ sa () Salid foam (@) Emulsion. 11, Which of the following electrolytes will have maxi- mum flocculation value for Fe(OH) sol ? G@) Nac @) Nags (NH )aPO4 (d) Ky804 12. Protecting power of a lyophilic sol is expressed in terms of (@)Coagulationvalue (6) Gold number ©@Bethottheabore —__() None af those. 13. Hydrophillicsols are stable duc to (@) Small size of the particle (©) Large size ofthe particle (©) Charge on the particle @ Attractive interaction betwoen colloidal partides and the dispersion medium. 14, For the process of adsorption, AH is (a) Positive (0) Negative ero (@) May be positive or negative. 15. Bredig’s arc method for the preparation of metallic sols involves (a) Dispersion (©) Condensation (©) Dispersion as well as condenss (d) Neither dispersion nor condensation. 16. Gold number is minimum for G@) Starch ©) Gelatin, ©Gum arsbie (@ Albumin 17. Fogis anexample of @ Foam, () Aerosol Ga (@ Emulsion, 18. Lyophilic sols have ..... between disperse phase and dispersion. (@) strong attractive interaction aa You have either reached a page that is unavailable for viewing or reached your viewing limit for this book. aa You have either reached a page that is unavailable for viewing or reached your viewing limit for this book. aa You have either reached a page that is unavailable for viewing or reached your viewing limit for this book. SURFACES AND CATALYSIS 207 74. Smoke is an example of (@) variable valency of gold (a) Gas dispersed in liquid _(b) Gas dispersed in solid (©) different concentration of gold particles (6) Solid dispersed in gas (d) Solid dispersed in solid. (© different type ofimpurities 75. Surface tension of lyophilicsols is (d) different diameters of colloidal particles. (a) Lower than water (0) More than water 79. Which of the following is mosteffective in coagulating (©) Equal to water (d) None of these. ‘a farric bytroxids aol? 76. According to Langmuir adsorption isotherm, the «Bel (®)KNOs amount of gas adsorbed at very high pressures (©) Ky804 (@) KglFa(CN)e). (q) reaches aconstant limiting value 80, Which of the following statements is not correct ? (©) goeson increasing with pressure (@) Physical adsorption is due to van der Waal's forces (© goes n decroasing with pressure (©) Chemical adsorption decreases at high temperature and (ch imereases fire and decreases later with pressure lew pevooure 77. Acolloidal system in which gas bubbles are dispersed ©) Physical adsorption is reversible in liquid is known as @ Adsorption eneray fr a phyical adsorption is generally (@) Foam 801 ‘greater than thet of chemical adsorption. (©) Aerosol (@) Emulsion, 78. The colour of the colloidal particles of gold obtained by different methods differ because of QUESTION BANK ‘Choose the correct answer from the four alternatives given in each of the following questions : 1. Inadsorption ofaceticacid on charcoal, the acoticacid 6. Colloidal aalutions are classified on the basia of ts (6) Molecular size (©) Size of colloidal particles (a) Absorbent (8) Adsorbent, (©) Organic or inorganic (d) Surface tension. Waheathats: oAtecetinse:: 6. The best coagulant for the precipitation of Fe(OH); is 2. ‘The size of the colloidal particles ranges between ; G)NagPO4 (©) NaNOg (a) 1070 1074 om ©) 107? o10-Sem (0 NgS0 (mcr (©) 10°F 19 10"7 em (@ 10°7 0107 em, 7. Colloidal solutions are purified by all except 3. When beam of ight ie passed through « collsidl (@)Crystalisation (0) Utea iteration solution (Uta centrifugation ( Dialyss. (It passes through as such esrieeca taut acdsee ai . solution of solidas dispersed phase and (©) this reflected a gas as the dispersion medium is known as (Wis scattered @sa @oa (@) Iie completely absorbed. anced (@) S0lidfoam. 4. The coagulating power of an electrolyte for arsenius 9, Gel is a aystom of sulphide sol decreases in the order (6) Sed dlapense phaso—ligud dapenatonsbedtnan (a) Na’, AI, Ba”* (POE, 804%, cr (@) Solid disperse phase—solid dispersion medium (on Ba? Ne? (oer, s0¢",70, (© Liquid disperse phase—salid dispersion mecium (d) Liquid disperse phaoe—liquid dispersion medium, aa You have either reached a page that is unavailable for viewing or reached your viewing limit for this book. aa You have either reached a page that is unavailable for viewing or reached your viewing limit for this book. aa You have either reached a page that is unavailable for viewing or reached your viewing limit for this book. SURFACES AND CATALYSIS a1 @ ® © ‘The Rubin number which was proposed by Ostwald as an alternative to the Gold number in order to measure the protective efficiency of a lyophilic colloid may be defined as the ‘mass in milligrams of acollcid per 100¢-.of solution which just prevents the colour change of standard sol of dye ‘Congo-Ruhin from red tovielat when 0.16 geq, KClisadded toit, ‘mass in grams of acolleid per 100¢.¢.of solution which just prevents the colour change of standard sol of dye Congo- Robin from red toviolet when 0.1 MKClis added to it. mass in grams of acolleid per 100¢.<.of solution which just prevents the colour change of standard sol of dye Congo- Rubin from red toviolet when 02 MKClis added to it mass in grams of acolleid per 100.¢.of solution which just prevents the colour change of standard sol of dye Congo- Rubin from red toviolet when 1M KCl is added toit. ‘The ability of an ion to bring about coagulation of a given colloid depends on (a) ite nize () the magnitude ofits charge (6) the sign of the charge alone (@) both magnitude and sign ofits charge. 64. At CMC the surfactant molecules (@) decompose (associate (@) become completely soluble @ dinsociate 65. Which of the following kinds of catalysis can be ex- plained by the adsorption theory ? (@) Homogeneouscatalysis (6) Acid-base catalysis (@)Heterogencous catalysis (d) Enzyme catalysis. 66. In which of the following, Tyndall effect is not ob- served? (@) Suspensions (@) Emulsions (Sugar solution (@ Geld oat 87. Which one of the following will have the highest coagulating power for a ferric hydroxide sol ? (@ Naot Bach, (OK YCr04 (©) KaFO(CN)g. 68. Point out the false statement (@)Brownian mavement and Tyndall effect is shown by colleidal systems ()Gold number is a measure ofthe protective power of a Iyophiliceolcia (©)The colleidal solution of « liquid in liquid iscalled gel (@) Hardy-Schulze rule is related with coagulation. ANSWERS QUESTION BANK (Level D 2(0) 2@ 4@ be 8.(a) 10) 8) 10.6) 1L@ 12.6) 13.) 4.@ 18.6 16.6) 18. @) 19. 20.0) 2. 22.6) 23.(9 mw), 26.(0) 31.0) 28.(0) 29.(a) 30. (@) 31.0) 32) 4) 35.0) 26.48) 37.0) 8.) 99.) 40.6) 42.0) a0 4.6) 45.6) 4.0) 2.6) 48.(6) 50.) 5.@ 52.0) 53.(@) 54.) 55. () 56.(d) 58. (0) 59.0) 60.) ae@ 62.00) 63.) 64.() 66.9 61.@) 68.) 9. 7.10) 1.@ 2.) m0) 8.00) 70.10) me Bb 1. 80.(@) QUESTION BANK (Level I) L@ 20) a) +0 5.@) +@ 2@ ae 2.6) 1.06) 1.0) 12.6) 12.0), 14.0) 16.6) 18. 17.0) 18.(a) 19.00) 20. (2) 21. 22.(@) 23.6) 24.) 25.) 26. (d) 22.(¢) 28.) 29.6) 30.0) 3.6) 32(@) aa You have either reached a page that is unavailable for viewing or reached your viewing limit for this book. aa You have either reached a page that is unavailable for viewing or reached your viewing limit for this book. aa You have either reached a page that is unavailable for viewing or reached your viewing limit for this book. o e re xn 5 = 5 G CHEMICAL FAMILIES—PERIODIC PROPERTIES CHEMICAL BONDING AND MOLECULAR STRUCTURE CHEMISTRY OF NON-METALS-1 (Hydrogen, Oxygen, Nitrogen and their Compounds) CHEMISTRY OF NON-METALS-II @oron, Silicon, Phosphorus, Sulphur and Noble Gases) 1. CHEMISTRY OF LIGHTER METALS Il, ELEMENTS, THEIR OCCURRENCE AND EXTRACTION CHEMISTRY OF HEAVIER METALS CHEMISTRY OF REPRESENTATIVE ELEMENTS ‘TRANSITION METALS INCLUDING LANTHANIDES CO-ORDINATION CHEMISTRY AND ORGANO METALLICS NUCLEAR CHEMISTRY aa You have either reached a page that is unavailable for viewing or reached your viewing limit for this book. aa You have either reached a page that is unavailable for viewing or reached your viewing limit for this book. aa You have either reached a page that is unavailable for viewing or reached your viewing limit for this book. CHEMICAL FAMILIES—PERIODIC PROPERTIES 217 3. 1 10. UESTION BANK Choose the correct answer from the four alternatives given in each of the following questions : The law of trieds is applicable to (@ Lithium, beryium, boron, (6) Flusrine, chlorine, bromine (©) Chlorine, bromine, iodine (c Sodium, potassium, rebidiom. Law of octaves was proposed by (a) Lothar Meyer @) DI Mondalaev (JAR. Newlands (IW. Dobereiner. Lothar Meyer plotted a graph showing variation of () Atomic volume with increase in atomic weight (@) Atomic radii with increasein atomic weight (© Atomic volume with increase in atomic number (@ Atomic radii with inereasein atomic number. ‘The concept of telluric helix was developed by () Lothar Meyer @)AB. de Chancourtois (Newlands (d) Dobersiner. ‘As we go from left to right in period 8, the gram atomic ‘volume of the elements () Increases regularly () Decreases regularly (6 First decreases and then increases (qh Remains almost constant. ‘The number of elements in each of the long periods of periodic table is (@32 w 8 8 (36. Lanthanum isa member of (@) «Block, (©) pBlock (©)a-Block (a) f Block. ‘Which block of the pericdic table contains maximum number of metals ? (a) Block, () p-Block (©) a-Blocke (@ f Block ‘The variable valency is generally observed in case of (a)Transition elements (6) Inert guses (Normal elements (@) Metallic element's. Inert gases except helium belong to (a) Block (©) p-Block (© d-Blocke (@) fF Block. uw. 2 13. ue 15. 16. Ww. 18. 19. Aproperty which gradually increases on moving down a group in the periodic table is (a) Tenisation enerzy __(b) Electronezativity (© Electron affinity (@) Atomic size. ‘The clement having electronic configuration iKr] 4d", 4f"*, 5s?, 5p°, 5d, 6s* belongs to G) = Block ©) pBlock © a-Block (@ fBlock Which of the following sets of elements would have nearly same atomic radii? G@) Na, K,Rb, Cs (©) Na, Mg, Al, © Fe, Co,Ni, Cu (@F, C1, Br Which ofthe followingelements would have thelowest first ionization energy (TE}) ? «Mg RD Li ea, ‘The magnitude of electron affinity depends on (@) Atomic size (©) Nuclear charge (©) Blectrenic configuration (d) All the above. ‘The ionization energy of nitrogen is more than that of oxygen because (@) Nitrogen has half.filled porbitals ©) Nitrogen atom is smaller in size than oxygen atom (© Witrogen contains less number of electrons a) Nitrogen isless electronerative, ‘The most electronegative element of the third period @F wP Br @a. In the long form of periodic table, elements are arranged according to (@) increasing atomie number () decreasing atomic number (© increasing atomic mass (d) decreasing atomic mare, In the Mendelesy’s periodic table, elements are arranged in the ascending order of their G@) number ofneutrons (6) stomie number ©) atomic mast (@ stomic voinme aa You have either reached a page that is unavailable for viewing or reached your viewing limit for this book. aa You have either reached a page that is unavailable for viewing or reached your viewing limit for this book. aa You have either reached a page that is unavailable for viewing or reached your viewing limit for this book. 221 CHEMICAL FAMILIES—PERIODIC PROPERTIES 82. Among the elements given below, the one with highest electropesitivity is” (aca wes fcr (aa. 88. The radii of F, F”, O and O* are in the order (a) 0 > F>0>F ()0* > F->F>0 WF >0%>Fs0 (h0* >O>F >F. 84. Which of the following species has the highestioniza- tion potential ? @Lt (b) Mg? oar (@Ne. 85. Which of the order for ionization energy is correct? @Be>B>C>NrO — @)B F>0% > NO ()N* > 0% > F > Na® (d) The correct order cannot be predicted. 89. Which one of the following elements has chemical properties most like those of sulphur ? @N w@P wa (d) Se. 90. Which one of the following groups contains metals, non-motals ne well as metalloids ? @IA IA, @IVA vu. 91. The outermost electronic configuration of the most dlectronegative clement is an? np? ne? np net np (dyno? pS. 100. 101. ‘The correct order for the electronegativities of N, O, F and Pis @F>N>P>0 @F>0>P>N @F>O>N>P (N>0>P>P. ‘The first ionization energies in eV/atom of magnesium and aluminium are respectively given by (764,598 (0) 184, 754 ©5.08,7.64 (2) 5.08, 6.98. ‘The second ionization energies of Cl, Ar, K and Ca are inthe order ()K>Cl>Ar>Ca (OK >Ar>Cl>Ca () Ar>Cl>K> Ca (@Ar> Ol> Ca>K Amongst the following clemonts (whose electronic configurs are given below), the one having the highest ionization energy is (@)tAr}as! (8) tan) as tar) sa 4s! (a) tas sal! 43? From which of the following epoci it is the easiest to remove an electron ? @o wor oo wor. ‘The maximum tendency to form the gaseous uniposi- tive ion is for the clement with configuration (a) 102, 24%, 2p8, 26? gascous state (12, 20%, ap8, 202, ap” ‘The element having maximum density belongs to (a)e-block (8) pobtock (©) d-block (@) fblock. ‘The element having highest melting point belongs to (a)s-bleck () p-tlock, (©) d-bleck (@ froioek. ‘The number of elements in third, fourth and fifth period are respectively (@)8, 8,18 (8) 18, 18, 32 (8, 18,32 (8, 18, 18. In which of the following processes, the least energy is released? (@)NG@)+e°—4N- @) OF G+ F @ 0) +70 ® SQ +5" @). aa You have either reached a page that is unavailable for viewing or reached your viewing limit for this book. aa You have either reached a page that is unavailable for viewing or reached your viewing limit for this book. aa You have either reached a page that is unavailable for viewing or reached your viewing limit for this book. CHEMICAL FAMILIES—PERIODIC PROPERTIES 225 (a) 12 24? ap () te! 20% 998 (e) 12? 262 998 a0! (d) 10? 26? 975 a2, 4. Tonic radii of @ Ti < Mn @® “ore Mor, @K'>cr (ay P™ > Pm, 55. Inwhich of the following processes, maximum energy is released? (a) O(g) + &° —+ Og) 0) 0" @)+e°—+ 0" @ OSH+ET SH) MSW +48" ®. 56. In a particular period, the element having largest atomic volume belongs to (@)Holegen family ) Alcali metals (Noble gases (@) Carbon family. 57. Which of the following halogens never shows positive oxidation state? (@)Flucrine (©) Chlorine (Bromine (@) Iodine. 58. Which of the following configurations is assodated ‘with the biggest jump between first and second ionise- tion energies ? (a)3s%,26! (0) 187, 287, 208, 30! Go 1s?, 28%, 298, 6, ap, as’ 59. Which of the following elements has its outermost energy level complete ? (@)Ne Ar (Kr @AM have their outermest shell complete. 60. Atomic volumes of C, N, O and F are in the order @C>N>F>0 @)C>N>0>F @F>0>N>C @N>C>0>¥. 61. Which of the following has least tendency to form unipositive ions in gaseous state? (at oa Br FR. 62, The electron affinity of K* (g) on is numerically equel to (@)Blectron affinity of Cl stom (@)Electronegativity of atom (o)Tonization energy of Ar atom (@Ioniration energy of Katom. 63, Amongst the following elements (whose electronic configurations are given below) the one having the highest electron affinity is (oy1s?, 25? 7. 1. (a) (Wo) 26%, ap? (6) (As 404, ap te) te) 242, 2p (2) Nek 307, 98, Which of the following is the strongest oxidizing agent? oF, Oz ©By Oh Which of the following statements is incorrect ? (@) Radium is the frst manrmade clement (8) Moat of the neta (©) Alkali metals are the lightest metals @) Alkaline earth metals are good reducing agents The clement of the fourth period of the periodic table ‘having maximum number of unpaired electrons (a) agFe aor ©) acu @ ase. Which carbonate does not decompose on strong heat- ing? (a) NagCO3, (®) BaCOg, @)cxC0g (d) All will decompose. Which of the following has the highe point ? @Li Na OK (@)Rb. Which of the following is the strongest reducing agent in aqueous solution ? a) Na Oli OK Cs. The electronegativity of the following elements in- creases in the order @)F>Cl>0>8, @S8>Cl>0>P @F>0>cl>s (@Cl>F>0>8. Which of the following isoelectronic species would require least energy for the removal of electron ? ay ca cr ox ar. In graph of atomic volume versus atomic weight, the elements corresponding to peaks in the curve belong to (a) Group-1 (@) Group-4 Jemants are man-made melting (©) Group-1s (©) Grovp-14 Which of the following processes requires maximum. amount of energy ? aa You have either reached a page that is unavailable for viewing or reached your viewing limit for this book. aa You have either reached a page that is unavailable for viewing or reached your viewing limit for this book. aa You have either reached a page that is unavailable for viewing or reached your viewing limit for this book. CHEMICAL, FAMILIES—PERIODIC PROPERTIES 20, 21, g a. a7. 39. 40. 43. ()A ond B belong to 2nd period whereas C and D belong. to third period. In general LE. increases along the period. and decreases down the group. Therefore, thecorrect order is B>A>D>C, (©) P having stable electronic configuration has the lowest BA (@)Thermal stability ofearbonates ofallalineearth metals increases from top to bottom. (@) The solubility of sulphates of group-2 elements decreases on going down the group from top to bottom. (@) Solwbility of hydroxides of alkaline earth metals in- creases as one moves from top to bettom in the group. (a) One NaCl has the highest m.pt. because its ions are small and hence force of attraction is strongest. LiCl is predominantly covalent compound. (6) Carbon belongs to group-14. (© FIR Fee Fic oF +E LE) =EAP. (©) Oxidising behaviour incresses from left to right in a petiod. (©) Carbonates of alluli metals are stable to heat. (©) Basic character of hydroxides increases from top to bottom in a group. (©) The most electronegative belongs to halogens which have ns*, np® outermost electronic confizuration. (a) Mg’, No* and F” are isoelectronic and their sizes are in the order Mg”* < Na* 0) Resonating structures + Resonatingstructures ofthespecies should have same atomic arrangement and same number of unpaired electrons. + Resonating forms are hypothetical and do not have physical reality. + Resonance hybrid is more stable than any of the rovonating form, + Neutral resonating forms are relatively more stable than those which have separate charges. + More stable resonating structure has relatively greater stability towards hybrid structure. + Resonating forms having same number of bonds are called ISOVALENT resonating structures. While those with different number of bonds are called HETEROVALENT resonating forms. + Resonance Energy. Itis the difference in the energy content of most stable resonating form and that of the resonance hybrid. Higher the resonance energy, greater the stability of resonance hybrid. + Bond order inresonance hybrid can be calculated by ‘using the formula given below : 233 ‘Number of bonds between two atoms . in all resonating structures Bond order = Nonsber of rsouating structure METALLIC BOND Metallic erystal consists of three dimensionally closely packed assembly of metal kernelsimmersed inthesea of mobile valence electrons. The average force of attraction betwoon metal Kernels end the mobile valence electronsis termed as metallic bond. HYDROGEN BOND It io olectrostatic force of attraction existing botweon ‘covalently bounded H atom of one molecule and the ‘lectronegative atom of the other molecule. This bond comes into existence when H atom is directly bonded to high electronegative atom such as N, O or F. + The bond is represented by dotted line. “HF ---H—F-.-H-F--- * The strength of H-bond lies between 12 kJ mol”! to 41.55 mol, + For more effective H-bonding, the H atom should be linked to the atom with small size and high clectronegativity. + Hbonding occuring betweon molecules of same or dif- ferent substances is termed as INTERMOLECULAR H-bonding. + H-bonding occuring within the same molecule ia termed as INTRAMOLECULAR H-bonding. o-nitro Phenol, salieylic acid are some examples. + The intermolecular H-bonds causes, association of molecules ; increase of melting and boiling points ; solubility of molecular substances like alcohol, sugar, honey in water, + H-bond is weaker than ionic, covalent, dative and metallic bonds + H-bond is stronger than van der Waal’ forces. DIPOLE-DIPOLEFORCES ‘The attractive forces among the molecules with p > 0. They arise because of the attractive forces between the opposite ends of the permanent dipoles. ‘VAN DER WAAL FORCES These are the attractive forces among the nox-polar molecules (molecules with 1 = 0) in solid or liquid state. They arise due to attractive interactions between instantaneous temporary escillating dipoles and induced dipoles. aa You have either reached a page that is unavailable for viewing or reached your viewing limit for this book. aa You have either reached a page that is unavailable for viewing or reached your viewing limit for this book. aa You have either reached a page that is unavailable for viewing or reached your viewing limit for this book. [Fasc svone so weasoua oma 237 ‘85. Which of the following liquid is miscible with water? (a) C8, (©) CrHgel wccy (@ C,H. 36. Which overlap would result the strongest bond? (ass Oepep ose pp. 37. The half of the difference between the electrons in bonding and antibonding MO's: (a) molecular order (6) bond order (electronic order (4) bonding capacity 38. In which out of the following pairs, the bond dis- sociation energy of second species ie larger than the first ? (a) Bry Ty ©)Cb, Bry (0) Fy, Cy (Ch, Fe. 39. Ina homonuclear molecule, higher the bond order, larger will be (a)bond length (6) bond strength (c)paremagnetism (@ ionic character. 40, VSEPR Theory was proposed by (a) Lewis (6) Lewis and Kossel (cNybom and Giltipsie Linus Pauling. 41. In open cage-like structure of ice, which bonds are present ? (a) Covalent ané W-londe @ Covalent and co-ordinate bonds (Co-ordinate and H-bonds (dhWtondsonly 42, Which of the following species does not have resonat- ing structure ? (CO; Cole fSi0y (N03, 43. The phosphate of certain metal has a formula MPO, ‘The formula of its arsenate is (a) MAsOy ()My(As09)s, {) MASO4 (d) MyAs0s, 44, The pair of elements most likely to form ionic com- pound is (a)Al and Cy (6) Chandy (c)0y end Cly (@) CsandCly 45. Inwhich {following species Cis not sp® hybridised? (a) CHOH (6) Diamond (Propane (@) Vinylebloride, a1. Pressing of two ice cubes together results in the for- mation of a single block. The forces responsible for their union are (a) van dar Waals forces (6) intramolecular H-bonds (©)intermolecalar H-bonds (a) covalent bonds. The two atoms with similar cuter-shell configuration ‘cannot unite by (a) ionie bona (b)covalent bond (c) metallic bond (a) co-ordinate bond. Which one is not present in CO, molecule? (@) ax bond (8)a dative bond (asigma bond (ch Lowe pairs. The number of rbonds in (CN),0 = C(CN)2 are on ws os (ch unpredictable, BiCl, doos not exist, the concept which best explains the above fact is (a) resonance (6) non-availability of vacant d-orbitale (©) inert pair effect (d molecular orbital concept. A triple bond between the two atoms is constituted by (a) three sigma bonds (6) three pi bonds (e) two piand one sigma bond (eb two sigoam and one pi bond. The type of hybridisation that leads to three dimen- sional geometry of bonds around the carbon atom ? (op? (eap* (d spd, Which of the following molecule is non-polar ? (a) BFy (Ply (CHCl, (d) All ofthe above. Which of the following process involves cleavage of H-bonds? (a) Sublimation of dry ion (6) Melting of ce {c) Dissociation of Hy molecules (d) Condensation of amnmonia. (a)sp aa You have either reached a page that is unavailable for viewing or reached your viewing limit for this book. aa You have either reached a page that is unavailable for viewing or reached your viewing limit for this book. aa You have either reached a page that is unavailable for viewing or reached your viewing limit for this book. (CHEMICAL BONDING AND MOLECULAR STRUCTURE 241 114, 115, 116, ut. 118. 118. 120. “The concept which best'explains the large difference in the boiling points of water (100°C), ethanol (82°C) and dimethyl ether (65°C) is () Resonance () Fetonding (increases in number of covalent bonds from water to ethanol to either (@)increase in molecalar massfrom water toethanol toether, If MX is T shaped, then the number of Ione pair around Mis (2 wo 3 los. Which of the following overlap of orbitals is/are incor- rect ? ' O:O-@ " @O:CO- NH,’+ OH” HC] == 40° + OH” ‘Redox reactions Na ——> NaOH +H (H,0 is oxidant) H20 Ch, ——+ HCI +0; (20 is reductant) High Water Ce) —— 00+ @) ‘em Hydrolytic reactions PBrg ———+ HyPO, + HBr CagP, ——> Ca(OH)y + PH CasNz ——> Ca(OH)2+ NH CaCy ——> Ca(OH)y + CzHly CaNCN ———> CaCOg+ NHy AlC3 —— > AIOH)s +CHy SiC, ——+ Si, + 4HC1 Hydrate reactions CuSO, (9) ——> CuSO, . 510 AICI; (¢) > AICI, .6H;0 Hard and Soft waters + Hard water does not produce lather with soap readily whereas soft water does 30. © Hardness of water is due to soluble salts of calcium and megnesium. ‘© Temporary hardness is due to the presence of bicarbonates of calcium and magnesium, © Permanent hardness is due to the presence of chlorides and sulphates of calcium and magnesium, aa You have either reached a page that is unavailable for viewing or reached your viewing limit for this book. aa You have either reached a page that is unavailable for viewing or reached your viewing limit for this book. aa You have either reached a page that is unavailable for viewing or reached your viewing limit for this book. CHEMISTRY OF NON-METALS—1 (Hydrogen, Orygen, Nitrogen and their Compounds) Chemical Properties Metals Li —> Lis Ne Ca ——> Ca;Np Dinitrogen Mg —> Maa Non-metals FoMo Hy; Ni "150K, 720 atm 000K * Colourless 0. 2No Te, Compounds + Non toxic 200K * Low solubility in water AlgO3 +C ———> AIN +CO + mp.632K, bp. 77K 300K Cacz———> CaNCN+ LJ ‘itrolim Fixation of Nitrogen ‘The process of conversion of atmospheric nitrogen into its useful compounds is called fixation of nitrogen. Fixation is being brought about in following ways (a) Natural fixation. A large amount of atmospheric nitrogen is brought to the soil either by lightening discharge or by symbiotic bacteria. Lightening discharge converts dinitrogen and dliesygen of airintocxides nitrogen which ere eventualy enverted into nitric acid and nitrates inthe soil. (®) Axtificial fixation. Nitrogen can be made to take partn cimical combinations under suitable conditions to form useful compounds (refer to chemical reactuuns). AMMONIA (NH) * Ammonia can be prepared by following methods : (a) By heating ammonium salts with strong alkalies NH,Cl+NaOH — NaCl + H,0 +NH; (b) By hydrolysis of metal nitrides or calcium cyanamide MgsNo + 6,0 —> 3Mg(OH)2 + 2NHy CaNCN + 3H,O——> CaCO, +2NHy Cal, Cyenamide ‘+ Inthe laboratory itis prepared by heating a mixture of ammonium chloride and sodium hydroxide. It is collected by downward displacement of ar. It is dried by quick lime (CaO). Since NHy reacts with dehydrating agents like CaCl, P05, HyS04. They cannot be used for drying ammonia. ‘+ Iti manufactured by Haber’s process in which a mixture of Np and Hg in the ratio 1:3 is passed ‘over heated Fe/Mo at 750 K and pressure ranging between 200-900 atm. FelMo Hy + N;p——___ 2NHy "780 K, 200.900 atm aa You have either reached a page that is unavailable for viewing or reached your viewing limit for this book. aa You have either reached a page that is unavailable for viewing or reached your viewing limit for this book. aa You have either reached a page that is unavailable for viewing or reached your viewing limit for this book. 1 5 1 CHEMISTRY OF NON-METALS—1 (Hydrogen, Oxygen, Nitrogen and their Compounds) QUESTION BANK Choose the correct answer from the four alternatives given in each of the following questions: Which metal can produce dihydrogen gas by reaction with dil. HS, ? (Ag eu Fe are ‘The name hydrogen was proposed by : (@) Lavoisier (@)Rutherford (©) Henry Cavandish (a)Scheele. In which property listed below hydrogen does not resemble with alleali metals? (a) Tendency to form eation () Nature of oxide (©) Combination with halogens (@) Reducing character. According to systematic nomenclature which hydrogen ‘compound is sulphane ? (@) HF Si (SF, (a) Hs. In which of the properties listed below hydrogen does not show resemblance with halogen. I. Electro positive character IL. Electro negative character TIL, Neutral nature of H0 IV. Atomicity (a) Land tt ()T only (©) Hand IIL (@)M ana. Which of the following gas is lightest ? (a) Dinydrogen (@)etium (©)Dinitrogen (a) Diorygen According to recent views which is the correct repre- sentation of hydrated proton in aqueous solutions. @u 05" (e) Hg? (a) Ho". Which isotope of hydrogen is/are radioactive in né ture? (@) Protiam and deuterium (6) Tritium only (6) Tritium and deuterium (d) Only deuterium, 10. i. 12. 14. 15. 16. 17. In which of the following reaction dihydrogen act as oxidising agent ? (a)Cas Hp—> @)Hy+0,— (©Ha+Fa— @C0+Ha— Which type of elements form ionic hydrides ? (a) Transition elements (6) Metalloids (c)Elements with high electronegativity (a) Elements with high electropostivity. The process of adsorption of hydrogen on palladium is known as (a) Syneresis (©) Occlusion CDifusion (@) Brosion Hydrogen at the moment ofits generation (newly born hydrogen) is generally called () Protium () Nascent hydrogen (©) Atomic hydrogen (@) Heavy hydrogen, ‘The three isotopes of hydrogen differ from one another in (a) Atomie number (c)Nudear charge Which of the following species has highest bond energy? (a) Ha (Dz (6) Number of protons (@) Nuclear mass. On Ch Aluminium reacts with boiling water to liberate dihydrogen gas along with the formation of (a) aluminium oxide (6) aluminium hydroxide ()aluminium suboxide _(d) aluminium superoxide. Which hyéride is also called alanate ? (a) Lit [A Nat (Hg (CoH (@) Alls. In the laboratory preparation of hydrogen, pure zine isnot used because (@ pure zinc becomes passive due to the formatien of oxide layer. aa You have either reached a page that is unavailable for viewing or reached your viewing limit for this book. aa You have either reached a page that is unavailable for viewing or reached your viewing limit for this book. aa You have either reached a page that is unavailable for viewing or reached your viewing limit for this book. [in ScD een ee ne ol oe Bevel 70. n. 2. 7 5. 16. (a) It was developed by the scientist named Callaghan (8) twas developed first in California (It refers to calcium gone. (q) It is based on the name of the campany which developed it Ethylene and HzO; react to give (@)002, 20 @)C0, HO (©) Ethylene oxide (@) Ethylene glycol. ‘The credit of discovery of hydrogen peroxide goes to (a) Gay-Lussae () Benard (©) Thenard (@) Cavandish, ‘The structure of hydrogen peroxide molecule is (a) Linear (6) Hitt open book-ike {€) Closed book like (2) Cyetic. ‘Which of the fellowing suppresses the decomposition of Hz0,? (@) Mino (6) Finely divided metals (e) Acetanilide (d) Dust particles In the Merck's process, the reagents involved for the reparation of hydrogen peroxide are (a) BaQy, HCL (8) Nag0p, HgS04 (©) BaOp, HPO, (@)PhOp, H20. The air oxidation of which of the following organic compound produces hydrogen peroxide ? (a)2-Bthylanthracene (6) 2-Ethylanthraquinol. (©2Bthylanthraquinone —_(d) B-Naphthol Ingascous hydrogen peroxide, the dihedral angle bet- ween H-atoms is X° but in solid state it is ¥°. The values of X and Y aro respectively (a)94.8, 94.8 @)1115, 902 (©)90.2, 90.2 @i5,118. ‘What is not true about H202 ? (a)Itis colourless, syrupy liquid in pure state (b)Itis odouress and hasa Mat taste (O)Mt smells like nitric acid. (a) tie appreciably soluble ‘The bleaching properties of hydrogen peroxide is due toits (@)acidie nature ability to liberate nascent exygen alechal and ether. (o)reducingnature (d)ability to liberate nascent hydrogen. 273 77. During concentration of hydrogen peroxide, the removal of last traces of water from 99% HzO: is carried out by @) Vacuum distillation ) Placing in « vacuum desicator (© Cooling in a freezing mixture @ Slow evaporation in sun. 78. When a mixture of ammonium sulphate and 50% 1,80, is electrolysed the products formed at anode and cathode are (a) Mand 102 () WHEY 280s and Hy (Hy andNHgHS0, (dH and Hy, "79. When H;02 is added to ice cold solution of acidified potassium dichromate containing ether. The contents are shaken and ellowed to stand (a) a blue colour is obtained in ether due to formation of Cx (SOs (©) ablue colour is obtained in ether due to formation of COs, (©) Cr0g is formed which dissolves in ether to give blue cclour (d) chromyl chloride is formed. Which of the following species is reduced by HO» (@) (FetCnygt* (©) {FetC¥)gI*" in alkaline medium fo Noe Te. B1. In which reaction, hydrogen peroxide neither acts as oxidising agent nor reducing agent ? fa) NagCO3+H20, > (b) PbS + 02> (€) Crg07* +H? +Hp02 > (d)803° + Hy02> 82, The volume of 10 vol, HzOz required to liberate 590 em? of O2 at STP is (50m 5.0m) (15m (@) 100 a. ‘The volume strength of 3.57 M solution of hydrogen peroxide is (a) 30 volume (6)40 volume (© 20volume (a)25 volum 84, Old lead paintings regain their originel colour by washing with hydrogen peroxide solution, It is be- cause (a) Hydrogen peroxide oxidise the dye aa You have either reached a page that is unavailable for viewing or reached your viewing limit for this book. aa You have either reached a page that is unavailable for viewing or reached your viewing limit for this book. aa You have either reached a page that is unavailable for viewing or reached your viewing limit for this book. [cme oats pra Spe es ol ma 1 2 1. Level IT 277 Choose the correct answer from the four alternatives given in each of the following questions : Among the hydrides given below which is/are basic ? LHCL UL. NHg TIL HyS IV. PEs (a) I, TH on (omt,1v ww. Consider the following sequence and answer Q.2-6. feds |p old High pressure Yis (@) ¥205 wre (©) Feq0y/Cx03 (ni Xia (@) Producer xas (©) Carbon dioxide (Synthesis gus (4) Pare dihydrogen gas. CO; and Hyin the sequence is represented respective: lyby @2,W Ow,x OW.Z X.Y. ‘The whole sequence refers to (©) Reforming of hydrocarbons (@ Linde's process, (a) Lane's process (€) Bosct’s process ‘The process refers to the (@) Labocatory preparation of dihydrogen () Manufacture of dihydrogen () Manufacture of carbon dioxide (@ Manufacture of dinitrogen. In ordinary hydrogen, the approximate composition of ortho and parahydrogen is respectively (a) 15%, 25% (0) 25%, 76% (€)50%, 50% (99%, 1%. Among the two solvents Hz0 and Dz0, the dissolution of sodium chloride occurs a 10. cr 12 13, 14, 15. (e) equally in both (6) more in D,0 (6) more in #0 (a) Only in 40. ‘The name perhydrol is associated with (6) solution of NagO. in ether (@) dilute solution of phenol in water (© dilute solution of 203 in water (@) dilate solution of CrOs in other. Which ions are present in Nessler's reagent ? (0) Hea? ana (6) Hig? and (© gla) and KY (d) Hgly" and K* Among the following which opecies are suboxide ? 1.0303; TL.NaO; IIL PbO: IV. 60g : V. 00. (Taam (0) and mt (Manav (a) Yana v, Which ofthe fillowingreagents can oxidise iodideions to iodine ? 1.03: TL Hy09MHC1 ; @1.Iv (6) Only 1 LW (0, 10,1, ‘Which of the following pairs contains normal as well as amphoteric oxides ? TIL HNOg: IV.0g; V. Fe (a) MgO, Al03 (B) HgO, Alg03 (© MgO, HO (d) CaO, AlaOa. ‘When zeolite is treated with hard water, sodium ions of zeolite get exchanged with (@) H’, So?* (b) Ca? Mg (Ht, Ca* (d) Only Mg. ‘Which pair contains species which can react with eat other to produce dihydrogen gas ? (6) Sodium amalgam and water (@) Hydrolith and water (© Copper and water, (@) Both (a) ané (0. aa You have either reached a page that is unavailable for viewing or reached your viewing limit for this book. aa You have either reached a page that is unavailable for viewing or reached your viewing limit for this book. aa You have either reached a page that is unavailable for viewing or reached your viewing limit for this book. CHEMISTRY OF NON-MBTALS—I (Hydrogen, Oxygen, Nitrogen and their Compounds) 281 65. Of the two solvents 1,0 and C2H;OH, sodium 67. 10. chloride dissolves (a) équallyin both (©) only in 0 (€)more in 02Hs0H (@) More in Hg0. ‘The brown colour often shown by nitric acid can be removed by (@) adding alittle mognesium powder (@) bubbling air through the warm acid (©)passing ammonia through the acid @) boiling the acid extensively. Which of the following metal ion gives deep biue coloured solution with liquor ammonia? (acu wre (eag* (a) Mg. Which of the following absorbs dinitrogen gas ? (@)aluminium carbide @) calcium carbide (calcium hydroxide (@) sodium hydroxide, Which of the following substance is used to clean metal surface of utensils during tinning process ? 4. 5. (a) Potassium nitrate (© Barium nitrate Sodium nitrate decomposes above = 800°C to give (a) Na, Op (0) 02, NaNOz (©)NO2, NaNO (N02, NayO. Which of the following substnace cannot be used for drying of ammonia gas? () Lead nitrate (@ Ammonium nitrate. I. Cone. Hg804 ; Tl. CaCl; IL. PyOyo; IV. Cad. @iv (v1, 1.1 ou, m,1V (@) Only 1 and UL. Thecorrect order of acid strength among the oxides is (a)Cg07>$02>P0; (6) COp>Ny04 > $03 (@)NaqO> MgO >Alg03 _(d) Ky0 > Ca0 > MgO. Which pair of metals become passive in cone, HNOs (Ni, Mg Fe, Al (080, Ni (acu, Pe x y (C2)*——> pH, —_— Graphite Blectie are In the above sequence, X and ¥ are respectively (a) NACL (6) NH,OH. (omaon wach (a) Ha, Ha ()H20, #20 Which of the following nitrate gives off a gas or a CERO Baye mixture of gases that cannot rekindle the glowing splinter? QUESTION BANK (Lovel 1) ©) 2@ 3 4@ 5. @) ®@) © 8. @ 10. @) wo 26 MH) B® ° 18 () 1.6) A) BR) HH) Cy) 26. (¢) 2.6) B®) BL) 8) o md 3. = BK) B® =— BB — 4.) a. 42) 20 4% @ 4) a) 4. 50. () Lo BR) BB) — BLO) BHD 56. (e) 57.) 5A (6) 58 (@) 6) GLH) BR — BH) BA.) 6. (@) 66. (a) 7.0 8&8 80 Mb we Be) 2. 0) 4. 6) 7%) 1% @) 7) — HH) 80.) a. @ 82. @) 6) O85) BH) BBL) 9. ©) 90. ©) 2 6 BM Me BO BE aa You have either reached a page that is unavailable for viewing or reached your viewing limit for this book. aa You have either reached a page that is unavailable for viewing or reached your viewing limit for this book. aa You have either reached a page that is unavailable for viewing or reached your viewing limit for this book. CHEMISTRY OF NON-METALS—I (Hydrogen, Oxygen, Nitrogen and their Compounds) 285 a8: 19, 20, 21, 22, £88 £88 & Baas 49, Hat (a) 3CuO + 2NHg 98 Cu + Ny + 3H20. {6) HyS200g + HO —+ Hy80q + HgS05, (@) Allthe statements are correct. (o) Water eannet actas dehydrating agent. (6) 42,0 molecales are -ordinated to Cu” ion forming & ‘complex ion (Cu(Ft20)4)**. (a) See properties of 1209. (c) Sodium azide is made of Na” and Ng” (azide) ions. (4) 503 + Ip + Hy0 —> 2HI03 + 503. (2) Ozone in considered as alltrope of xyge8. (©) Factual question. (01N205 +H02—+HNOs + HNOs. Peroxynitec add. (o Factual question. (©) Factual question. (6) Teraporary hardness is caused by Ca(HTCO3)y and MaHCO3)2 (a) Dve to the presence of two polar O—H bonds and non planar structure the dipcle-moment of 0p is quite high. {€) In Haber’s process arumoniats syuthesised from Hy and No. (8) Hig0 oxidisos ferrocyanide to ferricyanide. {(c) Hy0p oridises PbS to PbSO, along with the liberation of 0. (€) O2* ion is isoelectronic with Fy molecule. (@) 2Na + 280 — + 2NaOH + Hy w © @ n+ 2NaOH —s NayZnOg + Hy ” © @) (os Osan of 105m paren (0 8804 BEND, +580,» o04) +204 4850+ 2NO (o2ngniogs)Cly-—tag1+N20, +10. (avPoas slate sab sini bat 0 ron todimned (0 Sense of ane (0 051 20044) — 0s. (oe ctl gation. (ih temperate nse the ost fy (b Beach pres ives te propa of Hy fom meee (oy Faduatqusion ( NigoONE + B80 —s 88%, +240 cone) 52. (b) The main reaction involved in Birckland Eyde process is Electric Nae 0p 280 53, (a) Catia + 36HNO;— 0204+ 96803 +210, onan 4. (These maa ewrepndsoeaton exchange rein Hence the cations Ca”, Mg’ inhard water are exchanged. with H’, 56. 0) Hardnig of il due to ydrogonation in the presence of catalyst. ‘ior 56. (M0 +110 +} 05,0, s sf du to ick purple ge in seid 51, (Pactual pblen 58 (Problem ted ft 50. (SiC doe rt rat with ater £0. (Zinc and Hl podoe nner hype which ete Selb brn FeCl taht een ee 81, (Fecal pele f(y Fetal praten 83, (cd) HpO + Op — HgO + 20). ry ‘va 4. Quen bse on oie rope of hydrogen peroxide. 85, Dishes contant of 0 >Calg 0H. 88. remove NOpwheheawes yells lou 67. (a) Cu" forms deep blue ecloured complex with NH (Cu + NH4OH— [Cu(NHgiq/*" +20 ‘Deop blue) 88, (b)CaCy+N;—+ CaNCN + C Heat 89. (a) NHsCl——— Nily + HCL The HC! liberated cleans * ‘the metal surface. 7. (d) NHyNOg on heating gives NyO which does not help in combustion. Other given nitrates sive O, on heating 72, 6) NaNO3—+ NaNO, +} Oz 72. (b) Hz804, ashydrous CaCl and P40yo cannot bo used for ‘drying ammonia gas bocause they react with ammonia to form (NHi4)2S04, CaCl SNHg and (NELq)gHPO 4 respectively. 78. (a) Question based on nature of oxides and periodic trends. 74. (@) Both Fo and Al borome passive in eone. HNOg due to formation of exide layers 0) Ha 18. (c (C_)”* —— Coy +26 (graphite) <20H etre are aa You have either reached a page that is unavailable for viewing or reached your viewing limit for this book. aa You have either reached a page that is unavailable for viewing or reached your viewing limit for this book. aa You have either reached a page that is unavailable for viewing or reached your viewing limit for this book. CHEMISTRY OF NON-METALS—II (Boron, Silicon, Phosphorus, Sulphur and Noble Gases) 289 SILICA ( * Silica is common name of SiO,. * Sand, quartz, tridymite and erystobalite are diff- erent polymorphic forms of silica. onide. * Agate, Jaspar, Onyn are amorphous forms of silica. | Some of the chemical properties of Si02 are * Si02is anetwork solid in which each Si atom is listed as follows : surrounded by four O atoms held by covalent bonds | + SiOz +4HF —> SiF, (2) + 24,0) ‘and each O atom is surrounded by two Si atoms. (Hydrofiuorie acid) ‘* Quartz erists as colourless transparent hexagonal GLASS Heat prisms. © S102 + CaO(s) —> CaSiO3 (6) * Coloured varieties of quartz are used as gems amethyst—purple ; jaspar—red ; opal—white. * SiQy + NagSO, ——> Na,Si0s + 802 (@) © 330; + Cag(PO4)p > 3Ca8i0j (5) + POs (8) Glass is a mixtare of silicates of different metals one of which is essentially a sodium metal. Glass is not a true solid but is in fact, a transparent or transluscent super cooled liquid. Various Types of Glasses. Ordinary glass is a mixture of sodium and calcium silicates and is produced by heating a mixture of sodium carbonate, calcium oxide and silicon dioxide (orsand) in a furnace at about 1700 K. NazCO3 + Si0p—> NaSiO3 + COz CaO + Si02—> CaSiOg ‘This type of glass is known as soda glass or soft glass which has an approximate composition NaySi0g . CaSiOg . ASi02. Coloured glasses. Addition of transition metal compounds to the glass mix gives coloured glasses. Some examples are given below in tabular form : Compound addedto| MnO, | Cu0 | Auch | CaO | CaS | CraO, | NevAlPe Blass mix, Colour of glass | Purple | Red | Ruby | Blue | Lemon | Green | Milky Yellow White Heat resistant boro-silicate glass. This type of glassis obtained by the addition of boric oxide or borax. It is marketed under the trade name pyrex glass. High refractive indez glass. tis used for making cut glass objects or optical lenses, It is made by addition of lead oxide and KyCO, to the glass mix. Crook's glass. tio a opecial kind of optical glass containing CeOz (Cerium oxide) as one of the constituent. It as capacity to cxtoffthe uo. raye which are harmful. Its mainly used in making ses. Fliné glass. It is a mixture of silicates of potassium and lead (KO . PbO . 4Si0}). Ithae a very high refractive index and shining lustre, It is used for making optical instruments, Jana glass. It is mixture of zinc and barium borosilicates and is resistant to heat, shock, ete. * Pure silica is colourless but sand is brownish or yellowish due to presence of impurities of ferric #¢ SiO2 + NoyCO,(2)—+ NazSiO5 (6) + COz (@) aa You have either reached a page that is unavailable for viewing or reached your viewing limit for this book. aa You have either reached a page that is unavailable for viewing or reached your viewing limit for this book. aa You have either reached a page that is unavailable for viewing or reached your viewing limit for this book. CHEMISTRY OF NON-METALS—II (Boren, Silicon, Phosphorus, Sulphur and Noble Gases) Properties of Hydrogen sulphide HS is a colourless gas having foul smell like that of rotten eggs. * tis. poisonous gas and produces headache when inhaled in small quantities. * It is heavior than air (VD = 17) and fairly soluble in water. Its a weak dibasic acid and forms two series of salts bisulphides (HS") and sulphides (S*). NaOH + HS» Hj0 + NaHS 2NaOH + HyS—> 24,0 + Na,S. ‘* HS acts as reducing agent. It reduces S02 toS ; KMn0, to Mn®* ; HNO toNO> and Fe** toFe”*. + HS reacts with some metal cations in their aqueous solutions to form their corresponding sulphidos as precipitate, Cu (ag) + HS —> CuS + 2H" Ca” + HS —s CaS + 2H Black Yellow Sn?" (ag) + HyS —> SoS + 2H" Ni® + HoS—+ NiS + 2H* Yellow Black Co (ag) + Hy —> CoS + 25° Zn? + HS —» ZnS + 2H Black White * HS is extensively used in the laboratory in qualitative analysis of inorganic salts. OXIDES OF SULPHUR Sulphur forms two oxides : sulphur dioxide (Sz) and sulphur trioxide (SOs). SULPHUR DIOXIDE (02) Preparation of Sulphur dioxide (@) By Burning sulphur or sulphur rich ores like iron pyrites (FeS,) 3 8540; S02 ‘Fe; + 110, —+ Fep03 + 850. (®) By the action of dilute mineral acids on suiphites NaySO3 + 2HCI——» 2NaCl + S03 + Hz0 () In the laboratory, it is prepared byxheating Cu and cone. HaS0, Cu +2H,S0,—+ CuSO, + S02 + 2H,0 Properties of Sulphur dioxide * Itisa colourless gas with pungent smell It has a bent structure with each S—O bond length 143 pm and O—S—O angle 109.5 It dissolves in water to give sulphurous acid (HSOs). Tt can act as oxidising as well as reducing agent. Ttoxidises HS to S ;Mg to MgO and MgS; K to KpSO3 and KyS20y ; Hz to Hy0 and CO to CO. + Tt reduces Oly to HCl; KMn0y to colourless Mn®, KyOry07 to green Cr ions Fe™ to Fe ions, + SO; can also act as bleaching agent. Its bleaching action is due to reduction of colouring matterin the presence of moisture to colourless reduced products. * Bleaching by S02 is temporary because reduced products regain their colour due to oxidation in aa You have either reached a page that is unavailable for viewing or reached your viewing limit for this book. aa You have either reached a page that is unavailable for viewing or reached your viewing limit for this book. aa You have either reached a page that is unavailable for viewing or reached your viewing limit for this book. CHEMISTRY OF NON-METALS—II (Boron, Silicon, Phosphorus, Sulphur and Noble Gases) HALOGEN ACIDS [HX] Halogen acids are formed by the reaction of halogens with hydrogen. Hy +X,— 2X ‘The reactivity of halogens towards this reaction is in the order Fy>Cly> Brz> Ty HCI and HF are relatively stable and can be prepared by the action of cone. HaSOa on NaCl and CaF z, But HBr and HI cannot be prepared by the action of HaS04 and NaBr or Nal because HBr and HI are oxidised by F{pSO,. HBr and Hi can be prepared by the action of HyPO, on NaBr and Nal respectively. HLF is liquid at room temperature because of strong intermolecular H-bonding. Even in gaseous state it exists in dimeric form (H-F..... H-P). Other halogen acids are gases at rooin temperature. All halogen acids are monobasic acids and have appreciable solubility in water. HX + Hj0— Hy0°+X ‘The acid strength of halogen acids decrease as H- I> H—Br > HCl >H—F H_—Fie a halogen acid which ean dissolve glass forming ultimately HSiF, ie., hydrofluorosilicie acid. Hence itis used for etching the glass. ‘Some common trends among the halogen acids is Boiling points = :H—F>H—I>H—Br>H—Cl Melting points: H—1> HF > H—Br>H—C1 Dipole moment :H—F>H—Cl>H—Br>H—I Bond length :H-I> H-Br> H-Cl> HF Bond Energy :H—F>H—Cl>H—Br>H—I Reducing Power : H—1>H—Br > H—Cl> H—F. NOBLE GASES (He, Ne, Ar, Kr, Xe, Rn) Thoso aro tho cloments belonging to group 18 of the poriodie table, ‘They are also called rare gases or aerogens ‘The noble gases occupy about 1% by volume of the atmosphere. ‘Argon is the most abundant ofall the noble gases in the atmosphere, Helium was first discovered in the chromosphere of the sun. On earth, it was obtained by heating radioactive minerals like monazite,clevite, pitchblende ete. Helium iscommercially prepared from natural gas which contains 2—7% of He. Noble gases are isolated from liquid air either by repeated fractional distillation or by adsorption and desorption on coconut charcoal at different temperatures, All the noble gases are monatomic and their C,/C, ratio is 1.67. to weak van der Walls forces of attraction but e of the noble gas increases, The 6.p. of helium ‘The mp. and b.p. of noble gases are very low di increase down the group from He to Rn as the isthe lowest, ie, 4.2 K. The polarizability, ease of liquefaction and solubility in water of these gases increase as the size of the noble gas increases from He to Rn. Neil Bartlett observed that Pts oxidises 02 to give Oz" Pt” and Xe to giveXe" Pt , since the ionization energy of Op (1180 kJ mol” }) and Xe (1170 kJ mol” } are very close. ‘The more polarizable noble gases such as Kr and Xe form compounds with strong oxidising agents such as angen and fluorine. Xenon forms three fluorides : XeFy, Xe¥, and XeFg. (On the basis of VSEPR theory, XeP, has trigonal bipyramid, Xe, has octahedral and XeF has pentagonal bipyramid geometry. A mixture of helium (80%) and oxygen (20%) is used by divers for artificial respiration. Neon gas is used in discharge lamps and sign boards since neon light can be seen from a long distance even in mist and fog, aa You have either reached a page that is unavailable for viewing or reached your viewing limit for this book. aa You have either reached a page that is unavailable for viewing or reached your viewing limit for this book. aa You have either reached a page that is unavailable for viewing or reached your viewing limit for this book. aa You have either reached a page that is unavailable for viewing or reached your viewing limit for this book. aa You have either reached a page that is unavailable for viewing or reached your viewing limit for this book. aa You have either reached a page that is unavailable for viewing or reached your viewing limit for this book. aa You have either reached a page that is unavailable for viewing or reached your viewing limit for this book. CHEMISTRY OF NON-METALS—U1 (Boron, Silicon, Phosphorus, Sulphur and Noble Gases). 305 128, 129. 130. 131. 132, 133. 134. 1 ‘Helium is found in radioactive minerals because (a) Itis radioactivein nature (6) It reacts with radioactive elements (©)Itis formed by disintegration of radioactive clements present in the mineral like menazite, ete, and remain enclosed (@) None of the given statements are correct. Iodine reacts with concentrated HNO; to form an oxo acid with formula (@) HO (®) HIOs (©) HO, (@) HO, Which of the following oxide cannot be used as reduc- ing agent ? @) 802 (6) NOp © C0g (010, ‘Boron compounds behave as Lewis acids because of their () Acidic nature (Electron deficiency (a) Ionisation propertoa. Which of following halide is most acidic ? @ BCs (©) SbCl, OBICl, @ocy One litre flaskis full ofbromine vapours. The intensity of brown colour will not decrease appreciably on ad- ding to flask some (@) Pieces of marbles () Carbon tetrachloride (Animal charcoal powder (d) Carbon disulphide, Repeated use of which of the following fertilizers would increase the acidity of soil ? () Covalent nature 135. 136. 137. 138. 139. 140. Level II (a) Urea (#) Potassium nitrate (©) Super phosphate oflime (4) Ammonium sulphate. ‘Which of the following will form new compound in air? (@)Hy0in air (#) Opin air (Nein air (@ Pginair, The purest form of coal is (a) Lignite (t) Anthracite (Bituminous (a Peat, Which of the following form of phosphorus is most reactive? (a) Red phosphorus (6) White phosphorus (Scarlet phosphorus _(d) Viet phosphorus, Solid PCs exists as (a) Pot; @PCye PC (d) PCL)" (PCIgY”. ‘Which statement about hydrogen sulphide is false ? (a)Itis a covalent compound (yt ism gas with bad smell ()It isa weak base in water (@)Itisa stronger reducing agent as compared to water. ‘What is correct about the reaction ? 3NaClO— NaClO; + 2NaCl (a)It represents disproportionation (©) The reaction is used for manufacture of halides (©) The reaction does not oceur. (@)The process involves simply increasein O..N. of chlorine. Choose the correct answer from the jour alternatives given in each of the following questions: z Lin, B—>X—> ¥+ LB, Which of the statement is true for the above sequence of reactions ? 2 (a)Zis hydrogen OX is BoE (OZ and ¥ are Fy and Byllg respectively (@)Z ie potassium hydroxide. Which compound of boron is used as a mild antiseptic for washing of eyes under the name boric lotion ? aa You have either reached a page that is unavailable for viewing or reached your viewing limit for this book. aa You have either reached a page that is unavailable for viewing or reached your viewing limit for this book. aa You have either reached a page that is unavailable for viewing or reached your viewing limit for this book. CHEMISTRY OF NON-METALS—II (Bown, Sili~on, Phosphorus, Sulphur and Noble Gases 61. Which of the following is bleaching powder ? (a) CrO,Cly @ cachoc (ocaochs (@Ca0I0s, ‘Which noble gas is used for warning signals? (He Kr (Ne Xe Which of the following is called Ofeum ? (@)Porexymonosalphurie acid (®)Peroxydisulphurie acié (H28)07 (@)Both (6) and fe). Which statement about borax is incarmed™? 6. 309 | (@00n- (aN, In the reaction, I, +I” — I5", the species acting as Lewis acid is (@) Ipmolecules (6) ions (0 Ig ions (d) None of these, White phosphorus molecule does not have (0) Six P-P bonds () PEP angle of 60° (© Four P-P sigma bonds (@) Pear lone pairs of electrons. KF combines with HF to produce KHF2. The com- pound contains the species (a) tis used in the manutactame of 6ptcal glass were ona (OTs widely Ged as ue trmallurgical operations . " (Ibis used as water softener (OK, (APY ions (a) (SHY, F. (GVAII Gab stkteccieis a tescrnct: 70. An element was burnt in limited supply of air to give as ss . oxide A which on treatment with water gives an acid Orthoboric acid on strong heating to red heat gives B. Acid B on oxidation gives C which gives yellow (@) Borax. (0) Tetraborie acid. precipitate with AgNOs solution. A is (c) Metaboric acid (d) Boron trioxide. (a) 4g (b) S02, ‘Annealing of glass is done to make it (N02 (@) 805. (o}bettle ‘71. Awhite solid reacts with dil. HCl to give colourless gas_ (b) opaque that decolourises aqueous bromine. The solid is likely « tobe (transparent . (awanxteesa testeeg wine ()Satumcartonate (6) Sodium horde Which f the fllowing dooe not reac with AgCl pot. ? (© Sodium acetate (d) Sodium thiooulphate. ‘iain e woneoa 72. Zine and hydrazoic acid react to produce wranoy wnnyo (@) Zo(Nz, Ng, No (6) Zn(NOg)g, Ng (c) Zt 1» Ha, (d) Zn(NO3)2 and Hy. Which of the following is net pseudo halide ion ? (2 ZN gh, Ha, NE A ZNOwe * (@eno- @ cHRYCOO- ANSWERS QUESTION BANK (Level 1) Lo 2© 3.® 4 Be) 6. @) 1© aw 2 © 10. (a) 1. (@) 12, (6) 18.) 14. @) 15. (d) 16. (d) IT. (a) 18. (a) 19. (6) 20. (0) 21. b) 22, (a) 23. (b) 4. (a) 3. (@ 26. 27.) RG) 8H) BH) BL) 3.) 2. © 4 ©) OH) — BO — BHD = 4. 42. (@) 6B «6 68 0 @@ &@ aa You have either reached a page that is unavailable for viewing or reached your viewing limit for this book. aa You have either reached a page that is unavailable for viewing or reached your viewing limit for this book. aa You have either reached a page that is unavailable for viewing or reached your viewing limit for this book. CHEMISTRY OF NON-METALS—UI (Boron, Silicon, Phosphorus, Sulphur and Noble Gases) 313 | 17. (@) Factual question. 18. (e)lodine reset with NapS40p to form eslourieas Nal and Nap8,05. 20%ag603 +1z——# 2a # Nmg905 19. (a)All ble gases are monoatomic. 20, (6) HgPOs is monobasic acid i ° fi | H-0-P-H Loon 22. (6) Buchlorin is « mixture of C10. and Cla. 23, (c) When mistare is throwa in water, CaC produces CzEip which burns with lumineus fame and CagP2 produces PH which causes ignition on coming in contact with air 24-27. These are questions based on facts 28 (@)48+30,-22 5, 20405 ;28 + Ny—> 2BN burn 80. (b) The process refers to the isolation of boron by van Arkel’s method, BA. (€) Ni + 4CO— Ni(CO)g 88. (0) There aretwobridge H atoms in the molecule which aro held to two boron atoms by three centre bonds (B...H..B). 84. (€) 4BCly + SLIAIH, —> 2BpHe + 3AIC\g + SLiCL 88.) BFy+: Ny —> (PgR Ng) BF3+ FFB CF. 38. (6)Si+ 2HQ0(@) —> SiO) + 2H 99. (@)SiC is a network covalent solid. 40. (c) Charge of Li stomis + 1 and charge on each Al ato is +3. Thus, charge on two SiOs units must be — 4. Hence charge on one Si0; unit ia ~ 2. 42. (b) Factual question. 43. ©) 2Ne + Clp —> 2NaCl 2md — Lmol Here, sodium is limiting reagent. Thus, 1 mol of sodium ives 1 mol of NaCl. 44.) HF react with glass to form sodium and calcium fluoro silicates NagSi0y + 3H2F)—+> NagSiF + SHO CaSiOs + 3HaF2—+ CaSiF + 3H0. 45. (d)Fy +2HC1— 2UF + Ch, 41 “1 o 46. (c)NaClO + 2HC.—+ NaCl + 20+ Cla 47 (a) By + 2H Za + Hy mol 2mol B mot =0.2 mo $m =0.2 mol HOI givenis : 1.222.4= 0.5 mol HoreZn islimiting reagent. Thus, Hz produced at STP from 0.2 mol ofZn = 0.2 mol 202x224" 4.48 L 48, (@)Chlorinebeing ess clectropositive cannot replaceiodine from KIOs, 49. (4) Clz does not react with sulphuric acid. 50. (¢) Both Iand IV statements are correct. ao) 8H,804 + 8KC1Og—+3KHSO, + 20102 + HCIO, + HzO. 52. () Ky [ReION)s] + 6HyS04 + 61:0 + B80 5 + FeSO, + 3(NHg}g $04 + 660. 58. (a) The reaction involved is 2S02 +02 -> 280s. Thus, according to starting moles and also the number of moles ‘consumed and formed, A is S03 :B is SO, and C is Oa. 5A. (a) HyS + Pb(CHsCOO),— PbS + 2CHsCOCH Black 55. (8) Sifly 4209-4 Si0g « 280. Silane. 56, pron filings will completely dissolve in hydrochloric acid to produce Hy gas. 87-60. Factual questions. G1. ©) Oleumis HyS207 62. (@)Allthestatom Red 65.) HyBO;————+ 2B 209 + 64120. Bot 64, @) Factual question. 85. (©) NaNOy does not react with AgCI ppt. other reagents dissolve AgC] and form complexes. 68, () Factual questions. 67. G@) T ion is a ligand while Ip molecule is acceptor of lone pair from I in. te) Me statement C does not apply toP, molecule. 2a given is: 18 tz about the use of borax are incorrect. es. 69. (@) HF combines with F to form (HE). 7D. @)AinPsOg It dissolves in water to give HsPOs(B, which fives white pp. of Agg(PO4) with AgNOs, TL. d) Nay&_0g + HCI— NaCl + $02 +S i 802+ Bro-+ 2H0—+ 2HBr + HyS0,, 72. (¢) Zn + 3HNg— Ln(Ng)p + NHy + Ng ine azide aa You have either reached a page that is unavailable for viewing or reached your viewing limit for this book. aa You have either reached a page that is unavailable for viewing or reached your viewing limit for this book. aa You have either reached a page that is unavailable for viewing or reached your viewing limit for this book. | CHEMISTRY OF LIGHTER METALS Ul. ELEMENTS,THRIR OCCURRENCE AND EXTRACTION 317 SOME MISCELLANEOUS IMPORTANT POINTS + Cais the third mest abundant motal in th eerth’s crust. + Mgis harder than Na due to stronger malic bond. + Ca imparts brick red colour to the fame whereas Be end ‘Mg do not impart any colour tothe flame. + Soret cementin n mixtare o€ MgO axed MgCl. 1,0. + Mecan be obtained from son water by Dow's process. '* Mg0 is refractoryi.e., it can withstand high temperatures: without fusion or decomposition. + A mixture ofeamsic soda (NaOH) and quick ime (CaO) is known as soda-lime. + Milt of magnesia is sospeasion of MgyOH)g in water. + Lime water (Ca(OH? solution} and baryia[Ba(OT) eclu- tion} are used for detection of COp. + Cag on reaction with nitrogen yields nitro (a mixture of calcium cyanamide and carbon) Ca + Ny —+ CoC, +0 * Mgkeeps on burning in the atmosphere of C02 or SOz 2Mg + CO, —+ 2Mg0 + 2Mg + S0,—+ 2MgO +8 [Epsom salt (M504. 7H;0) is eMbrescent whereas mag- nesium chloride and aleium chloride are dliquescent. Chlorophyll contains magnesiam. (Calcium is present in bones as calcium phosphate, Precipitates of CaCOs, ebtained by adding sodium car. Donate to the solution of some soluble salt of ealeium, is known as precipitated chalk, {CaCl (a9) + NopO0) (og) —+ Ca005 ) $+ 2NaCI (ag). Corrundum, 03 is natural aluminium oxide. Diaspor, AlgO . Hp0 is natural hydrous aluminium oxide. Precious stones suchas sepphire, ruby, topes, et. are also ‘AlzO, containing other motal oxides. ‘Anhydrous aluminium chloride is a Lewis acid. ‘Anhydrous AlCl is covalent whereas hydrated aluminium ‘loride, AICs. 6110 is ionic aa You have either reached a page that is unavailable for viewing or reached your viewing limit for this book. aa You have either reached a page that is unavailable for viewing or reached your viewing limit for this book. aa You have either reached a page that is unavailable for viewing or reached your viewing limit for this book. L.CHEMISTRY OF LIGHTER METALS 11. ELEMENTS, THEIR. OCCURRENCE AND EXTRACTION ger 4. a. 7. 14. Which is the strongest reducing agent ? fa) Re @Na kK (Meg. Purification of aluminium, by electrolytic refining, is, known as (a) Hall's procoss ()Bacyer’s process {e) Hoope's process (a) Serpeck’s process. Bell metal is an alloy of (a) Cu+Pb )Cu+Sn (Casta, @cu+Ni Epsom salt is (a) MgSO, . THO (C0804 . 1,0 fe) MgSO4 . 2H0 (d)Ba8Og . 2490. Molecular formula of Glauber’s salt is (a) MgSO4.7H20 () e804. 1H20 (@ CaSO, . 5H (a)Nay804. 10120 ‘The metal that is extracted from sea water is. (a) Me Ca Ni (Fe. Electrolytic reduction of alumina by Hall-Heroult process is carried out (a) in the presence of NaCl. (6) in the presence of fluorite. ledin the presenes of ervolite which fermsa melt with lower melting temperature. (d)in the presence of eryolite which forms a melt with higher melting temperature.” Pyrolusite is an oxide ore of (a) Alumininin (b) Manganese (own (a)Gold Sea-weeds are a good source of (a) Chlorine (b)Bromine (o)Todine (a) Fluorine. ‘The slag consists of molten impurities, generally, in the form of (a) Motalearbonat (b)Metal silicate (€) Metal oxide (d)Motal nitrate, Gravity separation process may be used for the con- centration of (a) Chaleopyrite (c) Hematite (Bauxite (@)Calemine. 1. 76. 71. 78. 81. 82. In electro refining of metal the impure metal is made ‘the anode and a strip of pure metal the cathode during. the electrolysis of an aqueous solution of a complex salt. This method cannot be used for refining of (a) Silver () Copper (Aluminium (Gold ‘Which of the following metals is obtained by leaching its ore with dilute cyanide solution? @) Silver @) Titanium (Vanadium (Zine ‘The purpose of smelting an ore is, (a) to oxidise it ) to reduce it (©)to separate volatile impurities (@)to obtain an alloy. Sodium metal cannot be stored under (a) Rthanol ©) Water (©)Both of the above G@) None of these, Which of the following hydroxides is the least soluble inwater? (a) NaoH (CaO) () Mg(OIDy (@) Kon. Which of the following has the highest melting point? (a) NaF Nac ()NaBr @Nal. Plaster of Paris hardens by (a) giving of COz ) changing into CaCO (e) giving out water © combining with water. Which among the following has the smallest atomic radius? (o)Na OK og wen. Aibiteis a mineral of (a) Sodium () Alurniaiuen (© Both of the above (a) None of these. Aluminium chloride exists as dimer in (a) Vapour state () Inert solvents (c) Both of the above (@)None of these, ‘The percentage of CeO in Portland cement is ap proximately (a) 50-50% (€)30-40% (6) 20-30% (a) 70-80%, aa You have either reached a page that is unavailable for viewing or reached your viewing limit for this book. aa You have either reached a page that is unavailable for viewing or reached your viewing limit for this book. aa You have either reached a page that is unavailable for viewing or reached your viewing limit for this book. |. CHEMISTRY OF LIGHTER METALS 11, ELEMENTS, THEIR OCCURRENCE AND EXTRACTION. 57. 58. 6. (©) Plaster of Paris can be obtained by hydration of gypsum (a) Plaster of Paris is obtained by partial oxidation of eypsum. Sodium is made by the electrolysis ofa molten mixture of about 40% NaCl and 60% CaCl because (@) Caz helps in conduction of electricity (6) This mixture has lower melting point than NaCI (©) Ca®* ions can displace Na from NaCt (a) Ca” ions can redace NaCl to Na. Aluminium (II}) chloride forms a dimer because (a) Higher coordination number can be achieved by aluminium () Aluminium bas high ionization energy (© Aluminium belongs to third groap (aNTt cannot form n trimer, ‘The solubility in water of sulphates down theBe group is Be > Mg > > Ca > Sr > Ba. This is due to (a) High heat of solvation for mallor ions like Be™* (b)Increasing molecular mass (Decreasing Inttice energy (@)Increase in melting points. Sodium carbonate reacts with SO in aqueous medium togive (@)NaHSO; (NaHSO, (6) N80 (a) NagS0s. ‘The following compounds have been arranged inorder of their increasing thermal stabilities. Identify the correct order : I. K,CO,; Il, MgC03; III CaCOy; IV. BeCO; @l, AlO2", BaOz and NO", unpaired electron is present in (a) NOg" and Bay (@KOg only (©) KOg and A103 (@) BaGg only. Among the following statements, the incorrect one is, (a)Calamine and siderite are carbonates ()Argentite and cuprite are oxides 5. a7. 2. 325 (©) ine blonde and iron pyrites are sulphides (2) Malachite and arurite are ores of copper. The difference of water molecules in gypsum and plaster of paris is 5 5 we 1 wnt Heating an ore in the absence of sir below its melting point is called (a) Leaching (6) Roasting (e) Smelting (@Calcintation. ‘When a substanee A reacts with water it produces a combustible gas B and a solution of a substance C in water. When another substance D reacts with this solution of C, it also producos the same gas B on warming, but D can produce B on reaction with dilute sulphuric acid atroom temperature. Aimparts a deep golden yellow colour to a smokeless flame of Bunsen burner. A, B, C and D respectively are (@) Na, Hg, NoOH and Zn (©) K, Hy, KOH and Al ©) Ca, Hy, CafOH)y and Sa d) Caz, CoH, CalOH)g and Fe Cassiterite is an ore of a) Nn Ni Sb Sn. In the commercial electrochemical process for slu- minium extraction, the eletrelyte used ie (a) AWOH)g and NaOH solution (®) an aqueous solution of AlyiSO4)3, (©) a molten mixture of AlgO3 and NagAlFs (d) a molten mixture of AIOOH) and ANH). Which of the following substances can be used for drying gases ? (a) Calcium carbonate (©) Sodium bicarbonate Which of the following elements is (are) commercially recovered fram sea-water ? (@) Sodium carbonate (@) Calcium oxide. (2) Chlorine (&) Magnesium (@) Sodium (@)All ofthe above, Bauxite is leached with (a) KON @)Nacn 4) NaOH {d) NagCO3. aa You have either reached a page that is unavailable for viewing or reached your viewing limit for this book. aa You have either reached a page that is unavailable for viewing or reached your viewing limit for this book. aa You have either reached a page that is unavailable for viewing or reached your viewing limit for this book. OCCURRENCE AND EXTRACTION Iron is the second most abundant metal in the earth’s crust ‘The important ores of iron are : () Hazmatite, Fe20s (red oxide of iron) (ii) Magnetite, Foy04 (magnetic oxide of iron) (iti) Limonite, FeO: . 3Hy0 (hydrated oxide of iron) (iv) Iron pyrites, FeSp (v) Siderite or Spathie ore, FeCOs. EXTRACTION OF IRON ‘Tho cast iron is gonorally extracted from haematite (Fe,Os). The various steps involved in the process are as follows : 1, Concentration. By Levigation Process. 2. Caleination. 3, Smelting. After calcination the ore is subjected to reduction with carton in a blast furnace. ‘The calcined ore, coke and lime stone are mixed in the ratio of 8:4: 1and are fed into the furnace through cup and cone arrangement, at the same time hot air is blown in. Coke here serves as a fuel as well a reducing agent while lime is a flux. Commercial For (@) Cast iron or pig iron. It contains about 2 to 5% of carbon. of Iron (@) Steel. 1 contains 0.1 to 1.5% carbon and other im- purities. (©) Wrought iron. Itis the purest form of iron and contains carbon and other impurities less than 02%. feria ceenp tary and Formulae ii Heat Treatment of Sthel ‘The bardness of steel can be controlled by heat treatment ‘discussed below : Quenching. Ifa steel W and then suddealy coo the treatment is called qy this traatmentis hard an Annealing. [steel is he red hot and then cooled sl ing. The steel obtained by ‘Tempering. When the temperature of about 580 ide is heated to redness (1123 by plunging in water or oil, soching. The steel cbtained by brittle, fied to a temperature well below ly, the process is called Annesl- his treatment is soft. yuenched steel is heated to a and kept at that temperature for some time and tien coded slowly. The steel obtainedis, quite hard but less brittle. This treatment is called tempering. ‘The tempered steal is used for making razor blades, axes, Inives, swords, ete. Case Hardening. This jrocess is applied for low carbon steel artides. The articlep are heated with carbon just below the melting point. The carbon diffuses into the steal forming iron carbide at thp surface. Aa a result surface of the article gets hardened, Nitriding. This process ing ofartides ofalley ste! heated in the presence o! formation of nitrides of surface ofthe article employed for surface harden- In thie precoce, the articlesare ynmonia, which results in the tals like Mo, Cr, ete. en the 330 @) Copper pyrites, Cu (Gi) Copper glance, Cus (iit) Malachite, Cu(OH)CuC0 (iv) Ruby copper, Cu20) aa You have either reached a page that is unavailable for viewing or reached your viewing limit for this book. aa You have either reached a page that is unavailable for viewing or reached your viewing limit for this book. aa You have either reached a page that is unavailable for viewing or reached your viewing limit for this book. 334 con OBJECTIVE CHEMISTRY 3 4 1 Choose the correct answer from the four alternatives given in each of the, Which of the following is not an ore of iron? (a) Haematite (6) Limonite ()Siderite (d) Malachite. Silver ore dissolves in dilute solution of NaCN in the presence of air to form (a) AgcN (0) fAgOND2I (©) AgcNO (a agonal. Nitriding is a process of hardening steel by treatingit in an atmosphere of (@) NBs Na 05 (HS. Which of the following is used as antacid in medicines ? (a) Milk of magnesia () Milk of time (o)Lime water (d) Baryta. Quenched steel is (a) Hard (6) Brittle (©) Both of the above (d) None of these, ‘Which of the following statements is not correct ? (@) Wrought iron is ductile, soft and malleable (b) Annealed steel is soft (c) Tempering increases the hardness of steel (@) Hardness of stee! increases with increase in carbon Iron is protected from rusting by coating it witha thin layer of @Mg (ca (20 Kk Which of the following alloys of steel contains highest percentage of nickel ? (@ Stainless steel (6) Invar (6) Nickel steel (d) Permalloy. During clectrorefining of copper, gold may be recovered from (@) Electrolyte (©) Cathode (6) Anode mud (d) Cathode mud. 10. uw 12. 13, “4. 15. 16. 17. Iron is rendered passivp by treatment with (a) HyS04 (8) HgPO4 one CT ‘Cone. HNOs, ‘The formula of green v}triol is (@) FeSO, . 7H0 @)Cus04..5H,0 (©) FeSO, . 5H20 ‘The formule of ferric (@) FeSO, . (NH4), SOy (©) Fea(S04)9(NH1))2804 (d) Cu804 .7H90. (2) PeS04 . (NHy),80, ‘The chemical composit (@) Cus of malachite is (b) CuFeS, (d) Cu(OH)y . CuCOs. Which of the following metals occurs in nature in combined as well as in pative state ? (@) Zine (@)Mereury (©) Silver (Tin. Which of the followingmetals may be present in the anode mud during elecfrorefining of copper ? I. Gold ; IL. Iron ; IL. Sfiver!; IV. Maguosium. (a) andtt (e)Tand IV (and tt (Mana. Brass is an alloy of (@) Cu, Za.and Sn (®)Cu and Sn (©) Cu and Ni (d)Cuand Zn, ‘An oxide of lead whi teries, in safety mat agent, is is used in lead storage and is a powerful oxidizing (@) PbO @®PbO2 (©) PaO, (d)Pba0a, ‘Paris green’, which is|used to destroy agricultural pests is (a) Cu(CHjC00)y @) CuyAason2 aa You have either reached a page that is unavailable for viewing or reached your viewing limit for this book. aa You have either reached a page that is unavailable for viewing or reached your viewing limit for this book. aa You have either reached a page that is unavailable for viewing or reached your viewing limit for this book. 9 10. prs 18. ua 18. 16. rt (Sieg. (o) Gangue (© Metallic calcium (@) Calcium carbonate. Which of the following statements about ferric chloride is not correct ? () Its a covalent compound () Tt eublimes on heating (6) Its aqueous solution is basic (@) Anhydrous ferric chloride is dimer. Cyanide process is used for the extraction of Go) Ag Co au (@) Both (a) and (o. Which one of the following compounds does not decolorise an acidified aqueous solution of KMnO, ? (e) Sulphur dioxide (b) Ferric sulphate (© Hydrogen peroxide _(d) Ferrous sulphate. Which of the following elements is present in haomoglebin ? (a) Mg () Fe ocu (@) In. (On decomposition of anhydrous ferrous sulphate by strong heating, which of the following is/are obtained (a) Feg0g (®) 802 (©) 803 (2) All ofthe above, Which of the following metals bas maximum den- sity? (a) Iron (6) Copper (© Siver (@ Goa ‘Composition of the brown ringin the test for nitrateis (a) FeSO4. NO (8) e804. N20 (© FeNOgg @) FFeQz0)sNOISOg In the extraction of copper from copper pyrites, the flux used is @ Lime stone () Sitica (Quick imo (@) Slated Kimo. ‘Parting’ is the process associated with the purification of (@) Gold (b) Sitver (© Copper (a Mercury. ‘The solubility of AgBr in hypo solution is due to formation of (a) NaglAgtS203)2) (©) AgyS203 (by NalAgtS20s)1 (d) AgaS03. 19. 20, 21. 27, con OBJECTIVE CHEMISTRY ‘When quer ammontp a8.afided to an aqueous solution ofvopper sulphate, thp colour of solution changes from. lghtoeto deep The change indus to maton of (@) [Cu(NHg)aI* ion o [Ca(NHa)o}"* ion Coiewortigng™ion | lca tearing ion Which of the follovfing |compounds is soluble in (6) AgBr (a) Agr. ng ee is not black ia « (Cus oags © Augs (a) Hes. Silver chloride is sclyble in (@)Ammonia (#) Sodium thiosulphate (Potassium cyanide | (d) All of the above. Fineness is the torm eed to oxpross purity of (@)Gold (6) Silver (Copper Kd) Lead. The correct order of rpactivities of copper, silver, gold and iron is, (@)Fe>Cu>Ag> Av | (6) Cu>Ag>Fe> Au (e}Au>Ag>Cu> Fe (@)Fe> Ag>Ca> Au. ‘Which of the following: compounds is not black ? (@Cu0 (#) C0 (Cus Ags. Which of the followibg metals on reaction with hot (Hg Ag Fe wt. Which of the following salts is hexahydrate? (oy Blue vitriol 46) Groen vitriol (©)White vitriol (@Mobresalt. Which of the following alloys of iron contains highest percentage of nickel (@) Nickel steel ‘@) Stainless steel (olavar ( Alnico. Ferrous oxalate on Hiberates (@)COponly (@)0 only (©COzandco (@) Neither C02 nor CO. aa You have either reached a page that is unavailable for viewing or reached your viewing limit for this book. aa You have either reached a page that is unavailable for viewing or reached your viewing limit for this book. aa You have either reached a page that is unavailable for viewing or reached your viewing limit for this book. oa et ey 57.) 58. (©) 5.0) HL) fo OLS) 65. (©) 66. (©) 6.) 8) OH) TM) = LH) HY 73. 6) 4. ©) 75. (ch HINTS/SOLUTIONS QUESTION BANK (Level D 1. (d) Malachite, CulOH). . CuCO}, is an ore of copper. 28, (a) Factual questions. | 2.) AggS+ 4NaCN—+ NalAg(CNyy] + NagS 24, (@)Allthe three coinage nptale react with chlorine toform their corresponding NagS +209 + Hg0 + NagS09+ 2NaOH Se ass ‘Tho complex NalAg(CN)p] contains lAg(CN)gI” ions 38, (2 CaSO, SHO ae Be 3 O80 ec - Hy 8. (a) Nitriding’s the process employed for surface hardening i pedicel nenodeteia eiites core 36 Zine eaibits only +P oidation state Zn): (he heated in an atmosphere of ammonia, which razults in the 3a, formation of nitrides of metals suchas Cr, Mo, ete onthe 27 (c) Factual question. surface, 28 (@)Lithopone isa Jof BaS04 and ZnS. Itis used as 4. @) Suspension of MeCOs (milk of magnesia) is used as ‘white pigment in paints antacid. 29. (6) Wrought iron isthe mst pure form of iron, 5. (©) Quenched stecl has high carbon content and hence is 0, (d) Factual question very hard and britle. 0 bp Pace seme . e enrages te —_ of ee 82, (a) AgCl + 2NasS20,—4 NaglAg(S>0s)o] + NaCl. 7. (0) Iron is pr sm rusting by coating it with a layer of sine The process is known ar yalvanisation, ap menen - 8. (@) Permalloy contains about 78% nickel It is used in the (224 carat gold is 100 po cent gol. preparation of electromagnets. 8% (0) Pectoel question. 9. () During sectrorefning of copper, the leas reactive metals -«96_(c) FaeBual question. such as silver and gold fall below anode inthe form of anode «38 (2) Factual question. sand 38. (o) Galenais PIS. ). (a) Factual question. 40. (©) Iron does not form amdlgam with Hg, -_ (a) Factual question, 41, (@) Solder contains more than 60% (= 67%) tin 2. () Pactual question. 42. (©) Factual question 3. (d) Factual question. 43. (8) NAOH + Zn —> NaginOy + Ho (© Silver ours in combined state (as argentite, AgyS and 44, (€) Zine, because of its hither oxidation potential than iron 15. horn silver, AgCl) as well as in native state. (©) Anode mud contains less reactive metals such as gold andsilver whereas more reactive metals such as Feand Mg 45-46, Factual questions, undergoes oxidation in from misting (sacrificial go into solution as their ions. = Segue 16. (d)Factual question. Erasscontainsabovt 60% copperand «48. (8) CullT has partially filled d-orbital 40% zine 650. (é) Pig iron contains tage of carbon. 17, (6)Factual question. 51. @ Malachite is CuC03 . COE 18. (©) Double salt copper acctateopper arsenite is used as 52, &) Copperexiss in combfued state as well as nativestate agricultural pestzide and is known as Paris green. innature. 20, (d) CuCl is soluble in water. 58. (d) Sn-+ 2NOH + HOt» NagSn0s + 24g 21, (@) AuCl—> AUCL+ Gly Ph+ 2NaOK—> Neghb0y + He 22, (@) Mercury is alec known as quich aver Zn ¢ 2NaOH— Ns aa You have either reached a page that is unavailable for viewing or reached your viewing limit for this book. aa You have either reached a page that is unavailable for viewing or reached your viewing limit for this book. aa You have either reached a page that is unavailable for viewing or reached your viewing limit for this book. (CHEMISTRY OF REPRESENTATIVE ELEMENTS 347 GENERAL PHYSICAL CHARACTERISTICS Atomic Radii. ‘The atomic radii of alkaline carth metals are quite larger but are smaller than those of alkali metals. Density. ‘The densities of alkaline earth metals are larger than those of alkali metals. Alkaline earth metals are harder than alkali metals due to stronger metallic bond. Melting and Boiling Points. The melting and boiling points of alkaline earth metals aro quite ow but are higher than those of alkali metals. Ionization Energy. ‘The alkaline earth metals owing to their large size of atoms have fairly low values of ionisation energies ax compared to the p-block elements. However, within the group, the ionisation energy, decreases as the atomic number increases. Itis interesting to note that although first LE. of alkaline earth metals are higher than those of alkali metals, the second LE. values are much lower than those of alkali metals, Electropositive Character. Alkaline earth metals are feirly electropositive in nature, how- ever, due to their greater ionisation energies these are less electropositive than alkali metals. ‘Oxidation States. The alkaline earth metals have two electrons in their valence shell and by losing these electrons, these atoms acquire the stable noble gas configuration. Thus, alkaline earth metals exhibit + 2 oxidation state in their compounds. Moo > MM ee [Noble gas! ns? (Noble ash Reducing Properties. All the members of group 2, owing to their low ionisation energies have ‘a tendency to lose their valence electrons and thus act as strong reducing agents. Reducing character increases on moving down the group from top to bottom. Flame Colouration. Among the members of this group except Be and Mg all other members give characteristic colouration to the flame. ca Sr Ba Ra Brick red Crimson red Grassy green (Crimson red Solubility in Liquid Ammonia. All these metals dissolve in liquid NH. Dilute solutions are bright blue in colour while concentrated solutions are bronze coloured. These solutions on ‘evaporation yield hexammoniates af metals which slowly decompose to give metal amides M(NH3)5—> M(NHo)2 + 4NHs + Ho. CHEMICAL CHARACTERISTICS Alkaline earth metals are less reactive than alkali metals. Reaction with Water. Ca, Sr and Ba react with cold water, liberating hydrogen gas Ca + 2H,0 —> Ca(OH), + Ey Magnesium decomposes in hot water Mg + 2Hg0 —> Mg(OH), + He Beryllium docs not react with water. Order of Reactivity: Ba > Sr>Ca>Mg Reaction with Oxygen. All the alkaline earth metals burn in oxygen to form oxides Be, Mgand Ca form oxides whereas Ba and Sr form peroxides. Hydrides. All the members of the group, except beryllium combine with hydrogen on heating to form metal hydrides, My. nea +H; —> ME, aa You have either reached a page that is unavailable for viewing or reached your viewing limit for this book. aa You have either reached a page that is unavailable for viewing or reached your viewing limit for this book. aa You have either reached a page that is unavailable for viewing or reached your viewing limit for this book. 352 CONPREHENS (@ Ionic or salt like carbides. These are carbides of highly electropositive elements. BegG, CaCg, AlyCgete. (Gi) Covalent carbides. These are carbides of elements with equal or slightly lower el Examples are : B,C and SiC. (iii) Interstitial carbides, These are carbides of transition elements, Examples are 6. Silicates. It is general term given to solids with Si—O bonds. Some exampl (Gi0,) ; Asbestos (Ca MgSiz0¢), Feldspar and Zeolites (NagAlzSiz0y -xH0). Silicates are complex network solids having silicate (SiO,)* as the basic structural, can be further classified into chain silicates, ring silicates, cyclic silicates, shect sili dimensional silicates. 6 SILICONES. These are polymeric organcsilicon compounds containing Si having general formula (R,SiO)q ; R, here may be CH" or CoH," group @)Short chain silicon are oily liquids ; medium chain silicones are viscous oils,, chain silicones are rubbery elastomers and resins. (2) Silicones are water zepellants, heat resistant, chemically inert and resistant ait acids. ‘SOME MISCELLANEOUS IMPORTANT POINTS + Bucky Ball. Areceatly developed alletrope of C with formula Cay and is called buckminst + Diamond is insoluble in all the solvents. + Onheating diamond to 1860-2000°C graphite is formed. + Meat impure form of carbon is wood ehercoal and purest form is anthracite. + Interparticle forces in graphite are covalent bonds and Vander Wal forces. ‘+ Graphite can be oxidised by conc. HNO, to give graphitic acid. * Glass dissolves in hydrofluoric acid forming hydrofluoroslicie acid (HySI) NagSi0, + 3HP-—> 2NaF « H,SiF + SH,0 ‘Thia property ia made use of in etching of glass. ‘+ Mixture of O, and 5-10% CO, is called carbogen and is given to pneumonia patients poisioning for artificial respiration. Some Fire Extinguishers ‘© Dry powder extinguishers contain sand and baking soda (NeHHCO3) ‘+ Baking sode, sulphuric acid type extinguishers contain NaHCOs + HS0y + Foamiteextinguisherscontain baking soda and oluminium sulphate and are used in # Meta fires are put off ether by sand or pyrene (OC\)or fea type extinguishers. ‘Some Important Gascous fuels # Water gas (CO + H,) It is formed when steam is passed over red hot carbon. + Carburetted water gas [H, (94-38%) + CO (23 ~28%) + saturated hydrocarbons (17 ~21%) + unsatarated hydroca 4 C0, (0.2 = 2.28) + Ny @— 583) + Producer gas : (CO +No) + Semi water gas : [CO + Fy + Na; with CH (1 ~ 1.5%) ; 00, (~ 5%] * Olgas: (CH, +H, +00 + (Op) Its obtained in lab by burning kerosene ail + LPG: outane + isobutane) + Bio gas or gobar gus CH, (55%) ; Hy (7.4%); CO, (25%) ;Na= (2.6%) © Coal gas: CH (8296) ; Hy (4) ; 00 (9) ; CaHy (2%), CoH (3%) Ny 456); CO, 1%) ‘© Natural gas (mainly methane) : CH, (70-90%) ; CaM (5—10%) ; Hy (9%) ; CO + COp nit. Silieates tes and three victims of CO il fires (33-16%) aa You have either reached a page that is unavailable for viewing or reached your viewing limit for this book. aa You have either reached a page that is unavailable for viewing or reached your viewing limit for this book. aa You have either reached a page that is unavailable for viewing or reached your viewing limit for this book. CHEMISTRY OF REPRESENTATIVE ELEMENTS 357 (HO3S - 8, ~ SOgH) etz. The various allotropic forms of the clement contain (S), rings or chains of different size © Allotropy. Allelements exhibits allotropy. CHEMICAL CHARACTERISTICS 1. Formation of Hydrides () Allthe elements of this group form hydrides of the formula HM where M is central atom, (0, H,S, Se, HzTe, HP). (di) All these hydrides have bent shape and angle around the central atom decreases as : HO (104.59 ; HpS (92°) ; HySe (91°); HyTe (90. (iii) All the hydrides are volatile except H0. H,0 has exceptionally high melting point/boiling. point due to H-bonding. The boiling points of these hydrides varies as Hy0 (373 K) > HaTe (269 K)> HySe (232 K) > HS (213 1. (iv) These hydrides are weakly acidic and acidic nature inereases from H, -> HyTe. (v) Thermal stability of these hydrides decreases from HzO — H,Po due to weakening of M—H bond from © — Po. : (vi) Reducing nature of hydrides inereases as HO < Ha8 < HySe < HsTe, (vii) Dipole moment of these hydrides decreases as HzO > H,S > HySe > HzTe. 2. Formation of Halides (® Oxygen forms only dihalides (OF, Clz0, Brz0, ete.) because it cannot expand its octet due to non-availability of vacant d-orbitals. These dihalides are V-shaped with O assuming sp* hybrid state, (@ Other clements form tetrahalides and hexahalides in addition to dihalides. This is because of their ability to expand their octets. (iii) Tetrahalides have see-saw shape with central atom assuming sp°d hybrid state. They ean set as Lewis acids as well as Lewis bases. (iv) Hexalides of these elements have octahedral shape with central atom assuming sp*d* hybrid state. (o) In general, the thermal stability of the halides for the same central atom is, fluorides > chlorides > bromides > iodides. 8. Formation of Oxides (These elements form variety of oxides in different oxidation states ranging from +2— +6. (i) Acidic strength of oxides (in same oxidation state) decreases as we move down the group. $02 > Sez > TeO2 > PoOs Similarly, SOg> SeO3 > TeOs. iti) For a particular clement, the acidic character increases with increase in oxidation state. For ‘example, acidic character among oxides of sulphur increases as 4244 46 50 Se > TeOs > FoOs (v) Gaseous SQ, is a bent molecule with S atom assuming sp” hybrid state SeO snd TeOs are solids having polymeric chains. (vi) Gaseous SOs is a trigonal planar molecule with 0 - S~O angle = 120° and S atom in sp” hybrid state. Solid $0; exist in trimeric eyelic form or polymeric chain of SOs units 4. Structure of Oxoncids of Sulphur. The structure of some oxoacids of sulphur are as follows aa You have either reached a page that is unavailable for viewing or reached your viewing limit for this book. aa You have either reached a page that is unavailable for viewing or reached your viewing limit for this book. aa You have either reached a page that is unavailable for viewing or reached your viewing limit for this book. CHEMISTRY OF REPRESENTATIVE ELEMENTS 365 (@)NaF @Nacl (ONaBr Nal. 20. Which of the following halides has the maximum covalent character? @cat ox (Nat @uit. 21. The electrical conductivity of sodium, dissolved in liquid NH, is due to (a)Ammoniated Na’ ions (6) Ammoniated electrons (© Both of the above None of these. 22. Which of the following allali metal ionsin selution, is the best conductor of electricity ? @)Lit W) Net ox wes’, 28. Alcohol dissolves in (@)Lich (@) NaCl (KCL (@ Rech. 24. Which of the following does not illustrate the anomalous behaviour of lithium ? (@) Lithium reacts with nitrogen to form a nitride (@)Lithium is the hardest alkali metal (©) Lithium reacts with oxygen to form normal oxide only (a) Lig003 decomposes on heating. 25. The correct order of stability of hydrides of alkali metals is, (q) LiH> NaH > KH > RbH (O)NaH > KH > RbH> Lint (c)RbH> KH > NaH> Lil (q) LiH{> RbH > KH > NaH. 26. When potassium superoxide is dissolved in water, the products obtained are (a) KOH and H20. () KOH, H202 and Oz (KOH and O2 (@ KOH, H0 and Os, 27. Which of the following compounds has maximum covalent character? @LF (Nat @ Lil wee, GROUP 2 ELEMENTS 28. Which of the following is the most abundant alkaline earth metal ? (@)Be Mg ca sr, 29. Which of the elements of groups ITA has the highest value of [Ey ? (a) Mg (Be (Ca @sr. 30. ‘Which of the flowing is the strongert reducing agent ? (a) Be oy Mg (Ca sr. 31. Which ofthe following elements has the highest value of IE? (a)Na OMe OK (Ca. 82. Which of the following elements has the highest den- sity? Mg @)Na OK Rb. 33. When calcium is heated in the flame of a Bunsen burner, the colour imparted to the flame is (@) Golden yellow (b) Brick red © Crimson red (a) Grassy green. 34. Which of the following oxides is amphoteric ? @) C20 (®)Ba0 ©@Beo (@) MgO. 35. Which of the following hydroxides is most basic ? (a) Mg(OH)y (b) CaF S102 (@BaOHy, 36, Which ofthe following hydroxides is least soluble in (@) Mg(OH)2 (8) Ca(OH)2 ©S1OH (a) Ba(OH)g. 37. Which of the following is the moststable towards heat @) CaC0s (b) Bao © S:003 (a) MgO5. 88. Which of the following is most soluble in wator ? @)Ca80, (0) S804 ©) MSO. (a) BaS0.. ‘39. Which of the following exists as polymeric chains in solid state? @ SiC (©) Bac (MgC, (@ Bech! aa You have either reached a page that is unavailable for viewing or reached your viewing limit for this book. aa You have either reached a page that is unavailable for viewing or reached your viewing limit for this book. aa You have either reached a page that is unavailable for viewing or reached your viewing limit for this book. 371 137. 138, 139, 140. M1. 142, 143. 44. ‘Among chaleogons, the highost tondeney to form X?- ion is exhibited by (a) Oxygen () Sulphur © Petlerium (Selenium. In which of the following compound, oxygen does not exhibit the oxidation state of — 2? (@)803 (8) 80 (HO (@)H0e, ‘The binary compounds of oxygen and fluorine are called fluorides instead of oxide because (a) They contain fluoride ions in their molecules ()Oisless electronegative than fluorine (©)Size of oxygen is larger than fucrine (d) Reducing power of oxygen is smaller than fluorine. Which of the following is not responsible for an- amalous behaviour of oxygen as compared tothe other members of the family ? (a) Absence of vacant d-orbitals (@)Smallest atomic size Ability ofpx-prbonding (a Ability of exhibiting allotropy. Sulphur dees not exist as S, molecule because (a)itisless electronegative (Ovi is not able to constitute pr — px bond (it has ability to exhibit eatenation (@of tendency to show variable oxidation states. Inwhich of the following pair of species, the oxidation states of oxygen and sulphur is same ? (HLS, OF; (8) HS, 1,02 (Fes, NazOz (@) FeqOo,FeSp ‘Which element of group 16 cannot form compound of the type XF? (@)Seleaium (Sulphur Oxygen (@Both oxygen and tellurium. Inwhich allotropicform of sulphur, puckered Sg rings are not present ? (@) Plastic sulphar (©) Monoclinic ovlphur (6) Rhombie sulphur (@ Flowers of ealphar. 148. 146. 147. 148, 149. 150. 161. 182. 158. Which of the following is least acidic but possess highest thermal stability ? @ HO (o)HyTe ons (@UySe. What is same in various hydrides of chaleegens ? (e) Molecular shape (@) Reducing nature (0 Central bond angle (@ Magnitude of interparticle forces. Which of the following sulphur compound act as anti- chlor? (a) NagSO2, (6) Nag&O3 (©) Nags (@) All ofthe above. Which dioxide of group 16 elements not solid at room ‘temperature ? () 8eO2 (6) Te03 ©802 (a) Both S02 and 8003. Jn which of the following species, S atom assumes sp" hybrid state? 80s (D ; HgS (ID ; CS» (II) ; $5 1) @1n ou omn omy. In thiosulphuric acid, the number of S—S and O—H links are respectively @1,2 21 23 @13. In which of the following oxo acid, poroxide link is present ? @) y805 ) H2804 (©) H,803 (a) H807. What is true about HpS207 ? (a) Ibis obtained by dissolution of S05 in water. )Itis obtained by dissolution of 605 in cone. HS04. Iti called poroxydisulphurie acid @) Itcontains one O—O bond in its molecule. An clement X burns in air to form corresponding dioxide A. Onbubbling hydrogen through molten state of X, a gaseous hydride B is formed. A and B react to sive back X. A and Bare respectively () Hy8, $0 (6) 803,50 (©) 802, HS. (€) 003,084 aa You have either reached a page that is unavailable for viewing or reached your viewing limit for this book. aa You have either reached a page that is unavailable for viewing or reached your viewing limit for this book. aa You have either reached a page that is unavailable for viewing or reached your viewing limit for this book. CHEMISTRY OF REPRESENTATIVE ELEMENTS 375 @XeP, xe © Hadium (Neon. (oXeRs @) All ofthe above. 215. The boiling point is least for 214. Which of the following is used in weather balloons and @) Hy (©) He air ehips? oe jae (a) Argon (6) Krypton QUESTION BANK Level Ii Choose the correct answer from the four alternatives given in each of the following questions : 1. Which of the following compounds is(are) not scluble _-—-&- Which ofthe following metals has the highest melting inwater? point ? (@)LigCOg LiF @L (Be (Ligh, (@)Allof the above. Na Mg. 2. Which of the following represents the correct order of ‘9 The thermal stability of carbonates of alkaline earth basic strength ? metals is in the order (@)LiOH > NaOH > KOH > RLOH (a) MgO > CaCO > $xC03 > BaCOg (@RHOH> NaOH > KOH> LO (©) BuCO4 > MOOS > CuC05 > 81005 (@RDOH > KOH > NaOH> LiOH (©) BaCOp > S120 > CaCO > MgCO3 (@)LiOH > RLOH > KOH > NaOH. (d) BaCO3 > CaCO; > $1003 > MeCOs, 8. Which of the following compounds is stable to heat? 10. Which of the following is the strongest base ? (@)LisCOs 6) MrCO, (@) Mg(OH)2 () NaOH (©)Both ofthe above. __(d) None ofthese. CXR, (a) KOH. 4, Which of the following compounds has the highest 11. Carbonates of alkaline earth metals dissolve in water melting point ? in the presence of CO2 because @Lict Neo (@) offormation of bicarbonates Ke ce ©) of formation of hydrexides 5. Which of the following metals on reaction with excoss (offormation of complex oxygen forms superoxide? Gd) acidic nature of CO and basis nature of carbonates. @Li (Na 12. A fire-work gives green colour. It possibly contains OMe @K feos call of 6. Among the following the most stable hydride is pokes oe ” Goneenr Se eae © Strontium (@ Potassium. = % 13. Sapphire is a mineral of (oKH @ RoE. one om 7. Alkaline earth metal that resembles aluminium more than its congeners is os av. @Be OMe 14. Which of the following represents the correct order of first ionization energies ? (@) Mg> Al> Na>B (®)B>Al> Mg >Na (@)B>Mg> Al>Na (@) Mg>B>Al>Na, Ca @sr, aa You have either reached a page that is unavailable for viewing or reached your viewing limit for this book. aa You have either reached a page that is unavailable for viewing or reached your viewing limit for this book. aa You have either reached a page that is unavailable for viewing or reached your viewing limit for this book. CHEMISTRY OF REPRESENTATIVE ELEMENTS 381 121. @) 122. (@) 128. @) 12.) 188.) 188.) AT.) 128. (8) 129. (©) 180. (a) 181.) 18%) ~— 1S. ISK. @) 185.) 188.) 137. @) 138. (@) 138.0) 140.) AL) KL), (0) 145, (@) 146, (a) 147.) 148) 14. 18D, fa) BK (@) 18.) 188. © 154. 155. @) 156.) 1ST. SB) 159. @) 160.) 161. (©) 162. (6) 163.) 16) 185.) 16K.) 167.) 168, ) 169. (6) 170. ©) 1.) 17) @) TH) TB) 1TH.) 177. (@) 178. ©) 179. (6) 180. (©) 181. (&) 18. a) 183. @) 184.) 185. (@) 186. (a) 187. @) 188. (@) 18.) 1) 192) 1093. (e) 194.) 105. (6) 196. ()—10T. (o)—198. a) 198.) 200.) 201. ©) 202. (b) 203. (a) 204.) 205. (cd) 206. (a) 207. (e) 208. (a) 209. (©) 210. (@) ML @) = Ad) BAe) 1B. QUESTION BANK (Level II) L@ 2@ ad) 40 5. @ 6 ®) 7 @ o 2©) 10. (a) 1 @) 12.) 18. 14. (@) 15. (a) © 17. @) 18. © 12. @ = «2 @) 22. (b) 23. (a) %. (0) 25, (a) 26. (a) 27. (©) 28.) 8. 0. fa) 81. (a) 32. (©) 33. 34. (@) 35.) 98) TO) 38. (e) 38.) 40.) 4.© 42. ©) 8 44) 8 46.) 4.@) 48 @) 49, 50. (a) 51. (©) Bd) 5B) 54.) 55. (a) 56. (a) 87.) 58. (c) 59. (@) 60. () GL. 62. (c) 63. (a) @ 65. @) 66. «@) 67. OB) 70. aL@ Be 73. () 74.) Bb) BM md me 79.) 80.) aL ©) 82. d) 83. (0) 84.) 85. () 86. (a) 8.) 89.) 90. (@) 2) 8.) = 95) 8.) 9. (b) 98. (6) 99. (a) 100. (OL. (o) 102.) 108, 104. (a) 108. (a). HINTS/SOLUTIONS QUESTION BANK (Level 1) 1. (a) Lithium is the lightest metal (d = 0.54 g em™®), 2 (a) LE. decreases on moving down the group from top to bottom. 8. (d) Li has the strongest metallic bond and hence has the highest melting paint among group 1 elements. 4-6, Self explanatory and Factual question. 7% (b)In BCC lattice each atomis incontact with Bother atoms directly. 8 (d) Lithium is the only alkali metal that reacts with nitrogen to form nitride. 8. cv 12. 13. “a (d) Arcong alkali metals reactivity increases from top tobottom. (c) See Important terms, facts and formulae, (q) Lattice energy is more. (6) LAF has very high lattice energy and hence is least soluble in water. (a) On reaction with excess oxygen lithium forms normal ‘oxide, sodium forms peroxide while Rb and K form super- oxides. aa You have either reached a page that is unavailable for viewing or reached your viewing limit for this book. aa You have either reached a page that is unavailable for viewing or reached your viewing limit for this book. aa You have either reached a page that is unavailable for viewing or reached your viewing limit for this book. INTRODUCTION + The transition elements may be defined as the elements whoseatoms or simpleions contain Partially filled d-orbitals. This definition of transition elements does not cover zinc, cadmium ‘and mercury which are, however, included in the transition elements. There are three series of transition elements = + First transition series. From scandium (Z = 21) tozine (Z = 30). + Second transition series. From yttrium (Z = 39) to cadmium (Z = 48). ‘Third transition series. The first clement of this series is lanthanum (Z = 57). It is followed by fourteen elements called lanthanides which involve the filling of 4f-orbitals. The next nine elements from hafnium (Z = 72) to mercury (Z = 80) belong to third transition series. * The general electronic configuration of transition elements is (m~ 1) d'~1 ns7~*, PHYSICO-CHEMICAL PROPERTIES 1, Metallic Character All transition elements are metallic in nature. They show gradual decrease in electropositive character in moving from left to right. 2, Melting and Boiling Points ‘The melting and boiling points of transition elements are generally very high. In moving along the period from left to right, the melting point first increase to maximum then gradually decrease towards the end of the period. 3. Ionization Energy ‘The ionization energies (IE values) of transition elements are higher than those of «-bleck elements bat lower than p-block elements. In a particular transition series, ionization energy ‘increases gradually as we move from left to right. 4, Atomic and Ionic Radii ‘The atomic and ionic radii of transition elements ard smaller than those of »-block elements. The atomic radii first decrease till the middle, become almost constant and then increase towards the end of the period. The elements of second and third transition series belonging to a particular group have almost equal atomic radit. This is because of lanthanide contraction. All transition metals exhibit a great variety of oxidation states. The highest oxidation states are found in compounds of fluorine and oxygen. Some noteworthy features of oxidation states of the transition elements are : * In cach group, the highest oxidation state increases with inerease in atomic number, reaches a ‘maximum in the middle and then starts decreasing. + Fortheelements of first transition series (except scandium) + 2oxidation state is the most common oxidation state. 387 aa You have either reached a page that is unavailable for viewing or reached your viewing limit for this book. aa You have either reached a page that is unavailable for viewing or reached your viewing limit for this book. aa You have either reached a page that is unavailable for viewing or reached your viewing limit for this book. ‘TRANSITION METALS INCLUDING LANTHANIDES 391 6. Magnetic behaviour. The lanthanide ions (M*) generally show paramagnetism due to the ‘unpaired electrons in forbitals. 7. Atomic/ionic radii and Lanthanide contraction. The atomic radii and ionic radii of tripositive lanthanide ions (M®) show a steady and gradual decrease in moving from La to Lu. ‘The steady decrease in the size of lanthanide ions (M°*) with the increase in atomic number is called lanthanide contraction. Consequences of Lanthanide Contraction ‘The atomic radii of second row and third row of transition elements are almost similar. * Separation of lanthanides. Separation of lanthanides is also possible due to lanthanide contraction. + Variation in basic strength of hydroxide. The basic strength of hydroxides decreases from La(OH), to Lu(OH), 8 Oxidation States. The lanthanides exhibit a principal oxidation state of + 3. Although a few Ianthanides exhibit oxidation states of + 4 and + 2, yet they have the tendency to attain the oxidation state of + 3 because the +3 oxidation state is most stable state for all lanthanides. ‘Uses of Lanthanides: ‘Some common uses of lanthanide and their compounds are listed below (@ Mischmetal, an alloy which contains rare earth elements (94-95%), iron (5%) and traces of sulphur, carbon, silicon, calcium and aluminium. It is pyrophoric and is lighters, tracer bullets and shells. The main rare earth elements present in this alloy are cerium (© 40%), lanthanum and neodymium (~ 44%). (ii) An alloy containing magnesium and about 3% mischmetal is used in jet engine parts. (id) Corie sulphate is used as oxidising agent in volumetric analysis, iv) Cerium salts are also used in dyeing cotton, in lead accumulators and also a catalysts. (v) Oxides of neodymium and prascodymium are used for making coloured glasses for goggles. ACTINIDES + The series of elements involving the filling of 6f- orbitals is called actinide series. They follow Ac (89) and include elements from Th (90) and Lw (103). * Allthese metals are radioactive and therefore, their accessibility for laboratory investigations is limited. The elemente beyond uranium are all man-made elementeandaremade bynuclear-chemi- cal methods. Physico-Chomical Properties of Actinides 1. Oxidation states. The common oxidation stato of these cloments ia + 3. However, they also exhibit oxidation state of + 4, +5, +6. 2, Physical appearance. Actinides are silvery white metels. They get tarnished when exposed to tho attack of alkalies. 3. Density. All the actinides except thorium and americium have high densities. 4, Colour. ‘The actinide ions in general, are coloured. The appearance of colour depends upon the number of 5f electrons. The ions coataiaing 5f° and 6f7 configuration are colourless. While those having 3 to 6 electrons in 5fshell are coloured. ‘U% (Gf) : Red, Np (f4) : Bluish, Pu ** (6f5) : Blue, Am** (5/5) : Pink 5. Ionisation energies. LE values of actinides are low. 6, Electropositive character. All tho known actinide metals are highly electropositive. They resemble lanthanide series in this respect. 7. Magnetic properties. The actinide elements are paramagnetic due to the presence of unpaired electrons aa You have either reached a page that is unavailable for viewing or reached your viewing limit for this book. aa You have either reached a page that is unavailable for viewing or reached your viewing limit for this book. aa You have either reached a page that is unavailable for viewing or reached your viewing limit for this book. 396 001 isIVE OBJECTIVE CHEMISTRY ataniat al (yard? aa? 67. Which of the followidg species has (have) 5 unpaired electrons ? | (o) tae) 38 4? (a) tanga! 4s? ; (a) Mn®* ) cr 60. Which of the following statements is not true? or Fe, (a) An acidified solution of KaC1207 liberates iodine from iodides 68. Which ofthe following ionsis (are) likely toform white (©) In acidic solution dichromate ions are converted to complexes? ehromate ions ca va (©) (NH4)_Crg07 on heating undergo exothermic decomposi- fa oe 6 a tion ta give Cr203 one wor. (d) Potassium dichromate is used as a titrant for estimation 69. Which of the foll 1g complexes is expected to be te a diamagnetic? a. 66. Which of the following transition metals doos not occur in nature ? (a) Te er (Mo (ane. ‘Oxidation state of chromium in chromic acid is (@)0 +7 ta+3 re Which of the following compounds is (are) coloured due to charge transfer spectra and not due to d-d transitions? (@) RyCn 07 (2.6103 ()KMn04 (a) Allof the above, ‘When a small quantity of KMnQ, is added to con- contrated H,SO4, a green solution is obtained. Tho green colour is due to (a) KyMn0, ()Mnd. (© MnOg* (@Mn0,", When a large amount of KMnO, is added to con- centrated H,SOq, an explosive compound is formed. The formula of the compound is, (a) Hng07 (o)Mng04, fe) Mins (a) Mand", Oxidation number of chromium is + 6 in (a) C1203 (8) C105, (©) CrOCly (a) KyCr04, 70. 1. 72, "4. 15. (2) [CwHEZ0)4)?° ©) Lagi)" Ge) [ConNtHgI* @) (SeQO)g)™. Which of the following species has (have) O-—O bond? (a) Cra (6) COs (ei Cr? wood. species has (have) only one ost shell ? wor won. Which of the follow}ng oxides is (are) acidic in na- ture? (a) Mng0s (©) MnO, (e)Mn0z () MngQ0r, In alkaline medium] KMnO, oxidises iodides to (@) lode acid yTodates (e)lodine (q) Periodic acid. Which of the followipg oxides are acidic in nature ? (a) V205 () Mao (016r03 (@) NinOy Which of the followibg sulphides is black in colour ? (Nis 6) 208 (orn «cas. aa You have either reached a page that is unavailable for viewing or reached your viewing limit for this book. aa You have either reached a page that is unavailable for viewing or reached your viewing limit for this book. aa You have either reached a page that is unavailable for viewing or reached your viewing limit for this book. g R OBJECTIVE CHEMISTRY pee 10, a At 15 16, “1 a. RRR OR BE #888 ©) 2Mn0, + 4KOH + 0)", 216)Mn04 + 2830, Pactual question. (©) Coating with zinc is called galvanisation. (©) 20 +11,0 + 00, + 02+ CXOM)y . CuC0s. (Green mass () Cu) has 3d” configuration, Sod-d transitions are not possible init (@) Scandium has density of 3.0 g em” ® whereas most of tho transition clements have density greater than 8g é“9. (@) Melting point depends upon the strength of metallic bond in metals. (©) Pectosl question. (@) Ma? has 5 unpaired electrons in 3d-eubshell (@) Za has greater LE. than alkali and alkaline earth aetals and heace its base would be weak. (i) Za.+ 2NAOH —> NazZinOg + Hg. (2) Agia less reactive than Cx (xen electrochemieal series) (©) Se, Ty and V belong to first transition series (©) AgNOs is sensitive to light. (Fe and 04" ions cannot be oxidized further because Fe and § are in their highest oxidation state. (@) Fe is the most abundant element among ¢-block ele- ‘ments and it belongs tothe frst transition series (@) ded transitions became possible in incomplete d-sub- shell, (©) Oxidation state of Nn is + 7 in KMnOg. Upto Mn the hhigheot oxidation state shown by a transition element i equaltothenumber of electronsinns and (n ~ 1) dorbitas. ) Peramagnetic nature is due to unpaired electrons. (@) Innewirat and oltatine medium. ‘MaOq” + 2Ez0 +96" —>Min0g + 4017 In acidie mediam : MnO," + 8H* +5e” —> Mn™ + 41,0. © Melting point along a series of transition elements ‘increases upto chromium sub group, ©) Factual question. ) Lotetium is the last clement of lanthanides, Due to lanthanide contraction it has the amallaataiza, ) The oxides with intermediate oxidation states are amm- hotericin character. @) Pactual question. @ Thorium. ©) (NE)s Crg07 —+ Org03 + No + 4120. @ 2Mn04" + 5803 + 6H —> 2Mn* + 502 + 3120. ee & ? i & ‘is alkali metals < alkaline earth 34. Mercury elon 35. (2) Osnium has deine configuration 5° 67.1 ex 86. (€) Dichromete ion. ‘There are 8 Cr—0 bonds. 87.88, Factual question. 99. (d)Unlikep-block eleme} ‘an element differ by on 49. (2) 0rg07?" + 411202 + 9 9 26105 + HO. (Blue coloar in ether) aL BR BR BES 61. aa You have either reached a page that is unavailable for viewing or reached your viewing limit for this book. aa You have either reached a page that is unavailable for viewing or reached your viewing limit for this book. aa You have either reached a page that is unavailable for viewing or reached your viewing limit for this book. aa You have either reached a page that is unavailable for viewing or reached your viewing limit for this book. aa You have either reached a page that is unavailable for viewing or reached your viewing limit for this book. aa You have either reached a page that is unavailable for viewing or reached your viewing limit for this book. aa You have either reached a page that is unavailable for viewing or reached your viewing limit for this book. 410 OBJECTIVE CHEMISTRY QUESTION BANK Choose the correct answer from the four alternatives given in each of the, can be termed as mixed com- (©) [CuQNHs)) $04 (@) KaF ey. Which among the following represent chelating, ligands? @ son- won- ©2,2Dipyriayt won How many ions per mole should be produced in the solution when potash alum is dissolved in large excess of water ? (a) 4 mols (©) 8 mals 6 mols (@) 10 mols. ‘What is false about glycenato ion ? (a) Its formula is HpN ~ CH) - COO™ @) It is a symmetrical bidentate ligand © Itis a chelating ligand (4) Its unsymmetrical bidentate ligand. In the test for nitrate ion, the brown ring formed has a formula [Fe(H;0);NO]S0,, The oxidation number of iron in this complex is, @e+t +2 Ora * wo. on number of copper in cuprammoniam, (6) [Co(NH)4NOzCICL sulphate is we 4 os (as. ‘Which of the following ligand gives chelate coraplexes ? (@) Sen ()2, 2Dipyridyl (© Pyridine (a) NH ~ NH. ‘What is the co-ordination numberof Ni in nickel-DMG complex? @2 3 wos ae ing questions: ym dctacyanomolybdate(IV), of Mois, 6 2. 10. In the complex (Pt (py)}{Pt Cl), the oxidation num- bers of Pt atom in form@r and later part of the com- pound are respectively (@)0 ando @)+4and+2 (+ 2and +2 (@Oand+4. 11. Inthe complex [Co(NHy) ber of cobalt is + 8. The ws 2 12. The co-ordination m [Cofen}gBr1Cl, is @2 [CdCl] the oxidation num- ae oF x is w4 8. of cobalt in. wa ws 18. Which statement about tion number of a cation is true? (@) Metal ioas exhibit only ingle charactristic coordina tion number (6)'The co-ordination numtr is| equal to the umber of tigands bonded tothe (©The co-ordination numbpr is determined solely by the number of empty d (@CN is equal io the numbfrof coordinate bonds between ‘metal cation and 14. The ligand shown here is| (a) Tridendate (1, 10 phenanthrine 15. In which of the following complexes Ni metal is in highest oxidation state? (@) NCO (©) [Ni(NH3)6] (BF2 aa You have either reached a page that is unavailable for viewing or reached your viewing limit for this book. aa You have either reached a page that is unavailable for viewing or reached your viewing limit for this book. aa You have either reached a page that is unavailable for viewing or reached your viewing limit for this book. 416 co sIVE OBJECTIVE CHEMISTRY (a) ee called Zeine’s eat © [CHOXaI*, Cog (itis shonded complex Ween (Oxidation number of tis +4 zon fs ough wieroastil patos stom 14, Which of the follovfing matchings is incorrect ? 6. What is the value of x in the complex ? Se) Feeney vale On (@) Secondary valend onnumber - (© Secondary valene a number OG SO er ON |COM (@) Primary valencied hold ionisable species. 15. Onemoleofthe compound PtCl, . 5 NH; in its aqueous if oxidation state of cobalt in the complex is same as solution gave 3 mofes of silver chloride precipitate on that offiron in Ke [Fe(CN)e] adding excess of qlver nitrate, The formula of the 1 we comaplex 38 ws wo. LEHI os 7. In the complex trans-bis(ethylene diamine) chloro- DD Cd es hhydroxocobalt-(I1D) cation, the charge on the ion and NH the co-ordination number of cobalt are respectively mene bee ‘The erystal fild spits for octahedral 5 e exystal fiold slitting energy for octahodral com- +34 +26. Jjedral complex (dy) are related as 1B Which of the following is flexidentate ligands ? (a) Oxalate ion @) Hab, (0) A= 05.40 (EDTA (a) Ns. (© 2¢= 0.88 49 @ Bao 8. Which ofthe following system has unpaired maxi rich ofthefollowingsrstemhesunosiredmaximam 7, rar which type of elempents, the tendency to form ; oy complexes is maxihum ? (ad (octahedral high spin) (6) d® (octahedral) (@) Normal elements (Oe (tetrahedral) (@) d* (octahedral, low spin). (2) Transition a 10. In the formation of tetraammine zine (II) cation, the {6 Becta contain hoy Aled derbitets hybrid orbitals used by zincatom are (@ Inner transition ements. fw’ cat 18. Which ofthe followpng is antidote for lead poisoning ? om ds? (@) Cobly (6) Cisplatin 11. Which of the following complex has highest stability QEDTA (@) Ding. at 298 K? 18, Which ofthe folloning is referred to as Berlin green ? terica cu @ica@rpi* - fo teary @ Ica eng (a) FelFe (CN)g) {b) [PUNH)4) [PtClg) 12, Which ofthe following compound is used for the treat. © Cr0xCh, (dK 1CoNO2g} aneat of éaneet 20. Which of the follfwing will not show geometrical (@) Potassium sulphocyanide isomerism ? (©) cis-Diehlorodiammine platinum 1) (@) (CMON (6) (Cofen)y Cl) CL pepe Dick bored inant aterm 1) (6 ICHINHy)4 ClgIC1 (d) Both b) and (e. INH] NOs. anes 0 21, What is not true alfout Nag {Co(NOz)g] ? 13. Which of the fallowing pair contains optically active compound/species ? (@) [Cofen)g)”, trans-{CoClg (en) {b) |CofOX)g)*", [Co(NHs)¢) Cly (0) Its called sod () Its TUPAC name: (o Its aqueous solu (@) Its TUPAC name: cobadtinitrive sodium hexanitrocobaltate (111) nis slightly basic in nature sodium hexanitritocobaltate (ID. aa You have either reached a page that is unavailable for viewing or reached your viewing limit for this book. aa You have either reached a page that is unavailable for viewing or reached your viewing limit for this book. aa You have either reached a page that is unavailable for viewing or reached your viewing limit for this book. 420 OBJECTIVE CHEMISTRY con 70. n. n. ‘The total number of possible isomers of the compound iCall (NH)4] Pt" Cll is 3 5 we ws A co-ordination complex of cobalthas molecular formula containing five ammonia molecules, one nitro group and two chlorine atoms for one cobalt atom. One mole of this ‘compound produces three mole ions in an aqueous solu- tion. On reacting this solution with excess of silver nitratesolutions, two mcles of AgCl get precipitated. The ionic formula of this complex would be (a) [CoNHg), NOQCIl ((NHg)CIl (6) [Co(VHg)5Cll (CKNO2)} (©) 1Co(Nsg (NOD) Cy @ {Costs} NOD)2 Cl) Silver chloride dissolves in excess of NH,OH. The cation present in the resulting solution is (a Ag? (0) (Ag(NHs)2I"” (c) [Ag(NHo)41” (d) [Ag(NHg I". ‘The complex pentacarbonyliron (0) is diamagnetic. ‘The hybrid state of iron is (a) sp* apa (ods? (a) dep. ‘The hybrid state of Ni in Ni(CO) is (a)sp® tb) sp? pd Wade. ‘The magnetic moment of d® ion in square planar ligand field is (@)1.73BM (6) 2.83 BM (3.87 BM. (@)4.90 BM (©) Zero. ‘Which compler is formed when silver chloride dissol- ves in hypo solution (a) Nag [Ag (S203)4) (c) NaglAgt$z0g)o) (6) Nag {AgiS205)] (4) Nag (AgtS205)6- a 1". B. al. ‘he geometries ot Ni( (a) both square planar (e) beth tetrahedral (@) square planar and tetephedral. Which of the following complexes represent the ex- ample of dsp” hybri nm (a) [Fe(CNg)™ (B) INWCN) (€) ZeNHtg)gl”* (Ferg. Which of the foll ‘is an example of octahecral complex? (a) FeFg™ (©) (Zn 4)"" © wveng™ (dh iCagntty ai” CuSO, dissolves in. (@) CuOF, @cuo tc) [Cu(NH3)4I804 (d) \Cu(NHgig (OF) al. The shape of [(Cu(NHa)fI"* ip (a) Tetrahedral (6) Square planar (c) Pyramidal (d) Octahedral. The correct structure of Fe(CO)s (Fe, Z = 26) is (a) Octahedral (6) Tetrahedeal (©) Square pyramical |) Trigonal bipyramidal. Which of the following frill exhibit optical isomerism 4a) (Crven) (HzO) @) [Crcemgl* @) (CriNHgigl** (a) trans-(Crien) Clg (NHgal*. complexes will have four (a) (Co(en) (NHg)2 Cla} (©) {Co(en}s Cly) (@) (Co@PPhg}a (Ng) Cla} CL (4) [Cofen)a Cg} Cl. no d-electron in the central (8) [Co0Hgie1** (d) (Cx(Hg0}g)*. (a) (Min, c) [Re(CNgh* aa You have either reached a page that is unavailable for viewing or reached your viewing limit for this book. aa You have either reached a page that is unavailable for viewing or reached your viewing limit for this book. aa You have either reached a page that is unavailable for viewing or reached your viewing limit for this book. aa You have either reached a page that is unavailable for viewing or reached your viewing limit for this book. aa You have either reached a page that is unavailable for viewing or reached your viewing limit for this book. aa You have either reached a page that is unavailable for viewing or reached your viewing limit for this book. aa You have either reached a page that is unavailable for viewing or reached your viewing limit for this book. aa You have either reached a page that is unavailable for viewing or reached your viewing limit for this book. aa You have either reached a page that is unavailable for viewing or reached your viewing limit for this book. aa You have either reached a page that is unavailable for viewing or reached your viewing limit for this book. aa You have either reached a page that is unavailable for viewing or reached your viewing limit for this book. NUCLEAR CHEMISTRY 437 65. 67. 70. ‘The mass number andatomic number of the resulting atom will be lw) 19 ands (18 ands (0) 17 ana 6 4) 18 and. ‘The blank in the expression, 80 Gna) ania ‘ "te w"e wo tw wo ®n In the equation given below, Wg —s M64 In the bombarding particle is ote tne ot oe ‘The activity of 1 rd is equal to (a) 10° Bq (0) 10! Bq (010° Bq (a) 10° Bq. Radioactive decay follows (eoPirstorder ingen (0) Third order Knute (c)Zero order kinetics __(d) Second order kinetics. ‘To which part of the atom radioactivity is aesodated (a) pretonsonly () nncleus (e) neutrons only (@) extranuclear pert. ‘The mean life time of a radioactive element is equal to rte worsen @og (oes, For aradioactive olement,a graph of log 3 v time has aslope equal to 1. 72. 13. 4. 6. i (+2903 2 ©- a5 a Ors (@-2.503% The halt-life period of a radioactive element depends ‘upon (a) temperature (©) pressure (© the amount of element precent (@) nature of the clement. ‘An old piece of wood has 25% as much MC asa fresh one has today. The approximate age of the wood is (tof MC is 5770 years) (a) 11540 years, (©) 1440 years (e) 2880 years (28770 yours. A radionuclide has an initial activity of 8.00 10" Bg, After ten days the activity is reduced to 2.00 x 10* Bq, ‘The activity after 20 days (from the start) would be (a) 100 x 10 Bq (6) 1.00% 108 Bq (©) 500 x10" Bq (d)2.00x 10° Ba. A rare earth clement is radioactive, It and its daughter elements decay by emitting two alpha and two beta-particles in succession. The daughter ele- ment formed will belong to group @2 @i6 4 @u. 2p decays with emission of a beta-partice. The resulting nuclide emits an alpha-particle. The final nuclide formed is (a) "NgPb 7B: oe co ANSWERS QUESTION BANK (Level I) 2a) 3.(@) 4@ &@) 6a 2@) ae 10.) 1.6 12@ 18. wd) 16.) 16.) 18.4) 20.(e) 21. 22.) 23.(¢) 24.0) 26.6) 28.) 29. (a) 30.) 81.6) 92. (a) 34.06) 36.6) 31.) 38.(d) 38.) 40. (6) 42.(¢) 44.(@) 45.) 48.6) 42.0) 48. (0) 50.(a) 52) 53. (0) 54. (c) 55. (d) 56. (a) aa You have either reached a page that is unavailable for viewing or reached your viewing limit for this book. aa You have either reached a page that is unavailable for viewing or reached your viewing limit for this book. aa You have either reached a page that is unavailable for viewing or reached your viewing limit for this book. NUCLEAR CHEMISTRY 4aL 16, (b)Cadmium or Boron are used in control rods as they can absorb neutrons. mu, Bova, ~* any = Ban ‘gb——> A> gi B—— aah Pb and Chave sare atamienos, s0 they are isotopes. 16. (e) "IN JHe—> 1X4 yp Aa san asi Anig-1017 1422200 eo. 22. (b) Mass numbers of 0 and #!"Rn when divided by 4 fave same remainder 24, (e)Halflio radioactive notope doce net depend upon the amount of the substance 25, (by The daughter element ies two places lett the parent clementin the period table, Therefore, itbelongsto group 1A 29, (¢)Fromatonienumbers and mass numbers itis dear that ‘themissing particle ins maso no, zero and atomic number 4, (+t) 30. (ci praye are electromagnetic radiations BI. (0)Emissionof lu-patticledeereases the proton number by 2 whereas emission of 2 rparticles increases the proton number by 2 32, (a) Mass defect = 21.0087 + 21.0073) 4.0015 = 0.0306 ams Binding energy = 6.0805 x9815 = 284. 34. (c) Sines mass numbers and atomic numbers are already balanced, X must be yrays 26. 1) "Maa ta ot no atone and a na tne ss Soe aly he evo ad diac (4) 2 39. (a) “Bi aa ort aan tne Bt ‘Themissingpariaia ‘Ho wd ha symbol a, "oe Hn Comparing the mass nunbors and charge numbers, the imissingpartice is "ON, BR a. a1 at, 5, 58. sr. 58, slope yemete. (©) Beyond bismuth (Z.= 83), all nuclides are radioactive (d) Here noutron ie the bombarding particle and c-rays or partiteles are produced. (a) Emission of 1a and 2}-perticles reduces mass number by 4 but atomic number remains same (because decrense in atomic no. due to emission of w-particle is cancelled by increasein atomicnumber duetoomissionof2 ft particles) 034,92 82, aoa toa (@ AandB are ixoliaphers as they have same diflerence ‘fno, of neutrons and no. of protons. (d) Bnission of two a-particles decreases atomic number by 4 and emiasion of four P-particlos increasoa the atomic number by four. Therefore, atomic number of the daughter nuclide would be same as that of parent nuclide. (a) Rate of nuclear disintegration is diractly proportional (o the no. of atoms. 45210 602 x10 For the formation of 0.75 mole He 0.75 mole, of the radionuclide would decempose. Thus, starting with 1 mole ‘of radioactive substance, the time required for lecomposi- tion of 0.75 mole would be twice of¢1/2. (@) In three years intensity is reduced from Jy t0 19/3. In ‘next three years intensity ie further reduced to one-third 1 Ip To te, 3.2 (d) Decay constant does not change with time. (@)No. of moles of helium formed 18 (@) Rate of radioactive disintegration does not change with temperature or pressure. (Mass ofa radioactivesubstance is reduced to1/8 in three haltive 1 1 1 138 hour = 5 halfatves ta ta ta fa aa (@) 100 50—) 25 — 12.5 -—9 6.259 8.12 Ion wm woeov re = 8 ss o Supls Bat, pL, Be aa You have either reached a page that is unavailable for viewing or reached your viewing limit for this book. aa You have either reached a page that is unavailable for viewing or reached your viewing limit for this book. aa You have either reached a page that is unavailable for viewing or reached your viewing limit for this book. L QUESTION BANK Choose the correct answer from the four alternatives given in each of the, Which of the following is a most suitable method for the separation of 1:1 mixture of ortho and para- phenols ? @) Crystallisation (@) Sublimation (Steam distrillatim __(d) Separating funnel ‘The purification of benzeneamine is done by (@) Steam distil (©) Chromatographic techniques (© Fractional crystallisation (@) By adding oxalic acid ‘Simple distillation can be used for the seperation of a mixture of liquids whose boiling points differ at least by @s5c (6) 30—80°C (© Below 15° (d) Above 100° Steam distillation is used for the extraction of (@) Essential oils, (©) Higher alkynes (© Mineral oils (a) Fatty acids. ‘The separation an organiccompound from its aqueous solution can be achieved by : (q) Distillation (6) Steam distillation (@Soventextraction _(d) Fractional distillation. Chromatographic technique was iatroduced by (2) Dama (1803) (B) Kjeldahl (1903) (@ Tewett (1908) (d) Lavosier. ‘The separation of the constituents of a mixture by column chromatography depends upon their @) Difforoat eolubilitios 6) Different boiling points (© Different refractive indices (@ Differential adsorption. In paper chromatography, (@) the mobile phase isa liquid and the stationary phaseis, solid (6) the mobile phase isa solid and the stationary phase is a Tiguid (© both phases are liquide (@) both phases are solids. 9. 10. uM. 12. 13. 14. 15. 16. Ua lowing questions : The technique of, tography is suitable for compounds which (a)are liquids (©) have very low boi (are soluble in water (a) vapourize without dfcomposition A mixture of camphor and benzoic acid can be separated by which of the following technique ? (@) Chemical methods (@) Sublimation (©) Fractional distillatio (a) Bxtraction with a sobvent, Separation of two sybstanoos by fractional crystal- lisation depends upod their difference in (@) Densities (0) Sotubitities (© Melting point (a) Doing points ‘Sugar containing an fmpurity of common salt can be Purified by crystallizdtion from (a) Benzene (6) Ethanol {a Water, id acid and naphthalene can be (©) Petroleum ether separated by crystallization from (a) Hot water {b) Cold water (©) Benzene (Ether. ‘When a solid vapouriges directly without melting, the process called (@) Evaporation (© Saponification Naphthalene can be 1 (@) Sedimentation (@ Sublimation. rified by (q) Sublimation ( Crystallization (c) Bxtraction with a solfent (d) Distillation. Sublimation cannot be used for purification of (@) Naphthalene ©) Camphor ()Bonzoie acid (Urea, ‘The boiling point of a gompound doesnot depend upon (a) Sotubility of the comsound in water (&) Hydrogen bonding (Size ef melecules (@ Polarity of the mole aa You have either reached a page that is unavailable for viewing or reached your viewing limit for this book. aa You have either reached a page that is unavailable for viewing or reached your viewing limit for this book. aa You have either reached a page that is unavailable for viewing or reached your viewing limit for this book. PURIFICATION AND CHARACTERISATION OF ORGANIC COMPOUNDS: 457 ‘A gaseous hydrocarbon having empirical formula (CH, has density of 125 gdm” atS.T-P. The molecular formula of hydrocarbon is ‘Astudent performed Lassaign test for four different compounds. For which of thom, the test is not poisi- tive? QUESTION BANK (Level I) 33. (c) ‘34, (d) 35. (b) 36, (6) 37. (©) 38. (d) 39. (a) 40. (a) 81. (a) 82. (c) 83. (c) 84, (a) 8. @). QUESTION BANK (Level II) L©@ 2. ©) 3. @) 4. 6) & ©) 6. (c) 7. (b) 8b) a2 @ 10. (©) 11. (d) 12 (@) B. ©) I ‘15. (b). 16. (a) 57. (6) 5B. (c) 59. (b) 60, (a) 61. () 62. (b) b). HINTS/SOLUTIONS QUESTION BANK (Level 1) 1. (©) Tho higher volatility and lower solubility ofo-nitrophenal in water enables it to be easily sepreated from p-nitrophenol by steam distillation. 2. (a) Benzeneamine is steam volatile. Hence this method is suitable fr its purification. 3. (6)Ifis a factural question. (@) Only essential cils are volatile while others are not. Essential oils are volatile oils isolated from sap and tissues ofcertain plants and areextensively used in perfumery. (©) Organic compounds are usually more soluble in organic lvents than in water and hence can be extracted by vent extraction with awater immiscible erganie solvent such as CgHg, CHCls, CCly ether ete. aa You have either reached a page that is unavailable for viewing or reached your viewing limit for this book. aa You have either reached a page that is unavailable for viewing or reached your viewing limit for this book. aa You have either reached a page that is unavailable for viewing or reached your viewing limit for this book. Da EUNDAMENTAL CONCEPTS OF aa ORGANIC CHEMISTRY Important Terms, Facts and Formulae * Vital force theory was proposed by Berzelius. ‘* Urea was the first organic compound synthesized in laboratory. It was synthesized by Wohler by heating potassium cyanate and ammonium chloride, * Carbon exhibits catenation to maximum extent among all the elements. * Carbon atom can undergo sp, sp” and sp° hybridisation. * Carbon atom is sp hybridised ifit is directly attached to two atoms, sp? hybridised i atoms and sp” hybridised if it is directly attached to four atoms, * According to Huckel rule cyclic planar systems containing a total of (4n + 2) delocalized x electrons possess a high degree of stability and behave as aromatic compounds. Thus, benzene molecule having six x electrons (aromatic sextet) behaves as an aromatic compound. is attached to three HYBRIDIZATION Carbon in its compounds undergoes three types of hybridization. These are sp? (in alkanes), sp? (in alkenes) and sp (in alkanes) leading to C-C, C = C and C= C bonds respectively (@ The C-C, C= C and C=C bond lengths are 1.54 A, 134 A and 1.20 A respectively. (it) More is the s Character in an orbital, lower is its energy. Thus, the relative energy of the s, p and different hybrid orbitals follows the following order p > sp* > sp* > sp > s. (iii) The hybrid orbitals are stronger than the unhybridized orbitals. (iv) The strength of bonds varies as sp* > sp’ > sp. Intermolecular forces in covalent compounds ‘The molecules are held together through very weak forces which are of three types : (a) Dipole-dipole interaction (b) Hydrogen. bond and (c) London (Vander Waal’s) forces. (i) The relative order of attraction of the three intermolecular forces is Hydrogen bond > dipole-dipole > Vander Weal’s (Wi) The Physical properties depend upon the nature of the intermolecular forces. (a) Covalent compounds melt (or boil) at low temperatures, (b) Solubility. Covalent compounds are governed by solubility rule “like dissolves like”. ELECTRON DISPLACEMENT IN REACTIONS Polarization effect (Permanent) 1 Inductive 1. Inductomeric 2 Mesomeric 2. Electromerie 3. Hyper-conjuctive 463 aa You have either reached a page that is unavailable for viewing or reached your viewing limit for this book. aa You have either reached a page that is unavailable for viewing or reached your viewing limit for this book. aa You have either reached a page that is unavailable for viewing or reached your viewing limit for this book. aa You have either reached a page that is unavailable for viewing or reached your viewing limit for this book. aa You have either reached a page that is unavailable for viewing or reached your viewing limit for this book. aa You have either reached a page that is unavailable for viewing or reached your viewing limit for this book. aa You have either reached a page that is unavailable for viewing or reached your viewing limit for this book. aa You have either reached a page that is unavailable for viewing or reached your viewing limit for this book. L 476 COMPRENENDIVE, OBJECTIVE CHEMISTRY 9. What is incorrect about alkyl carbocation ? 19. C—H bond length is rhinihum in a) It ean undergo the attack of nucleophile wos, @ cat, (6) it ean undergo rearrangement OCH, (Coie () It can be attacked by carbene 20. The bond order of infividual CC bonds in ben- (2) It can undergo less of H* to form aliens. zene is 10. A nucleophile is called an ambident nucleophile if G)one (a) it is capable of acting as nucleophile as well as an (6) two | electrophile (c) between one and (6) its attacking atom has two lone pairs Gian uae lew (c) it has two possible electron donating sites 21, C,Hay is the general fhrmula used to represent (4) ie can act Levis acid. aaa avaiiseas 11. Which of the following is the correct order of () Alleynes (ch Banaancida. nucleophilic 22. A compound with mdiecular formula C,H, must (@) CHiy < NH < OF Br > Cl” > F. The correct nucleophility order in protic solvent is (a) Br'> Or> FT (o> Br>cr>F The shortest C—-C bond distance is found in (6) F-> Cr >Br or @)Or>F>Br or. (a) Diamond (8) Ethane (©) Benzene (2) Acetylene. Number of sec-H-atoms in iso-butane is @)1 9 wa (@) none, ‘The tetrahedral structure of carbon was proposed by (a) Kekule () Wohter (©) Vont Hoff and LeBel (d) Berthlo:. ‘Marsh gas mainly contains (a) CoH, &) Clty ©) CH, CEs. C—C bond length is minimum in (@) CH, wok, (©) GH, (2) He. 21, 30. ‘Which of the following is the most stable ? (a) CH, = CH, (@) CH,CH = CHCH, (©) CH,C = CH—CH,CH, (d) CH,CH,CH = CH,. \ cH, ‘An example of conjugated diene is (@)1, 4Pentadiene (6) 1, 2Butadiono (8) 1. S-Hexadiene (d) 1, 3-Peatadione. ‘The hybridization of girbon atoms in C—C single bond of H,C = CH—CH = CH, is (a) op? - op? (©) op? op* (©) sp? — ap (d) ep — ap. Which of the followinghas zero dipole moment ? (a) 1-Butene (6) cis-2-Batene (@) trans.2-Butene ‘What is not true abor (@) All the members har (6) They hove identical (d) 2.Mothyl-1-propane, homologous series ? similar chemical properties ical properties by a general formal iffer in molecular mass by 14, ‘The number of pi bonds in isoprene is @2 ws 1 (ao. ‘The hybrid states of tHe four carbon atoms in 1, 2- butadiene are (ap, sp* pop? 2p, sp spp The C—C single bond (@) Propane ()Propene aa You have either reached a page that is unavailable for viewing or reached your viewing limit for this book. aa You have either reached a page that is unavailable for viewing or reached your viewing limit for this book. aa You have either reached a page that is unavailable for viewing or reached your viewing limit for this book. 480 COMPREHERSIVE OBJECTIVE CHRMISTRY (@1, 2adicyano ethene) (B) but-Q-ene-, 4 75. The TUPAC name of the compound CO is (e)dicbat-2-en-1, 4-nitfile (@) Nene of these. 5 ‘0 81. The IUPAC name of Fhe compound (a) 3, 2, 8-tri-formy! propane CH, = CH-ON is | @) S-forny!-, pentane dial (@) Vinyl eyanide © acrylonitrile (c) Propane-t, 2, 9-trial (©)prop-2ene nitrile | @) 2-propene-2-nitrle. (d) Propane-t, 2, 3-triearbaldohyde. 76. ‘The correct IUPAC name of the following compound is (@) Ethyl tri-amine ON, N, Netri-ethyl aspine (ON, N-diethyl ethanfmine + (@)None of the above. | J 88. The IUPAC name of fhe | (a) Tethyl-2, 4, 5, G-tetra methyl deca-t, Bdiene Me Fe- pound is (b) ethyl6, 6, 7, 9tetra methyl deca2, diene coon (c) 2, 4, 5, Gtetramethyl-7-ethyl-deca-, 7iene (a) 1-amino-2-hydroxy foroppnoie acid (4) None of the above. () 2-amino-2-carbory pentanol ‘71. The TUPAC of the compound (©) 2-aminod-hydrary] ° out NOs (@ V-hyaroxy-2-amin 4 H,C-C-CH-CH-CO,H is (a) 2-nitro-3-hydroxy-4-oropentancic scid 84. ‘The compound Hage (8) 2nitro-3 hydroxy 4-pentanon-L-oie acid TUPACname (©) 4-amino-3-hydroxy-2-cxopentanoic acid (@)1, Spentadiyne | (t) penta-t, 4tiyne (2) 2-Hydrony-2-nitro-4-oropentanoic acid. (a pentane-1, 3-di:yne. 78. The correct IUPAC name of 85. ne of tartarie acid is CH, (@)1,4 H. C08, _0-0H,cH,On is (a) 2ethyl prop-t-ene-2-01 (6) 2-thyl prop-2-ene-1-o1 (©) 2allyl-1-butanol 86. TUPAC name of is (a) 3Bthyl but-3-en-1-<. 79, The IUPAC name of the compound (a) Bieyelo (4.2.0) oe CH, 0 (@)Bieyelo 16.2.2} oe 1 1 (6) Bicyelo 16.2.0 oe H,C = C—CH,—C—OC,H, is (2) Bicyelo (4.2.2) deedne. (a) Ethy| aceto ethanonte 87, TUPAC name of DVT is (8) Bthy! methyl butenoate (a) Dichlorotipheny! tfichloro ethane (©) Ethyl serylote @)Dichlorodipheny! thichlore benzene (d) Ethy| 3-methylbut-3-en-oate. (€) Dichlorodipheny! fFichlore propane (@1, 1, LTrichloro-2|2-bis (p-chlorophenyl) ethane. HL fan 88, Apsirin is 80. The TUPAC name of the compound x is (a) Methyl salicylic add (8) Acstyl salicylic acid (©) Acetyl salicylate | (q) Phensl salicylic acid. =N aa You have either reached a page that is unavailable for viewing or reached your viewing limit for this book. aa You have either reached a page that is unavailable for viewing or reached your viewing limit for this book. aa You have either reached a page that is unavailable for viewing or reached your viewing limit for this book. 484. COMPREH OBJECTIVE CHEMISTRY 25. () 26. () 97. 98. (a) 29. @) 80. (e) 33. (6) 34. (a) 35. (a) 36. (a) 31. (0) 38. (a) 41. ©) 42. ©) 43. d) 44. (@) 45. 46. (c) 49. (e) 50. (a) 51. 6) 52. (©) 538. (6) Ba. (eo) QUESTION BANK (Level I1) L© 2. (e) 3.) ) 5. (a) 6. (a) 9. ©) 10. (@) i. @) © zo) 14. @) 17.) 18. (6) 19. ©) ©. @ 22. (d) 2. (c) 26. (c) a7. (d) . (a) . ) 30. (c) 33. (a) 34. ©) 35. () © © 38. (c) 41. @ 42. (a) 43. @) ® = Ce) 46. (a) 49. (a) 50. (c) 51. 6) (@) @ 54. (a) 87. (e) 58. (c) 59. (a) (a) (o) 62. (c) 65. (a) 86. (e) 67. ®) @ ® 70. () 73. (d) 74. (@) 15. d) = (a) @ 78. (d) 81. (c) 82. (e) 83. (c) ® @) 86. (a) 89. (a) 90. (a) 91. © ® @ 94. (a) 97. (a) 98. (a) 99. (a) (a) - ©. 102. (d) 105. (d) 106. (a) 107. (©) =e) ©) 110. (6) 13. (6) 14. ©) 18. ©) @ @ 118. (6) HINTS/SOLU' i) QUESTION BANK (Level 1) ® ) © (@). (@) @ ) © © @ ( ) @ @ @) (a) ®) ) @ 24, RSSSISSSSS BES 88 . (a) . Ce) . d) @) © @ @) @ @) © @ @ ) 6) @ @) @ 1 6) 1. (&) 6x,- GH= CH-Ga,01 (Crotyl chloride) 1-Chioro-2-butene 2. (a) Factual question. 8. (c) Factual question. H i 4. (6) CH,=CH CH= CH-C=0. 5. (a) H-C = C—CH,OH Prop-2yn-l-ol 6. (b) GF, in tabied by sonance wheres FC—C+ is destabilized by detron withdrawing effet off C— group. F. +7 Di Dei >» ’ ’ | RC 7. (6) Physical prope regular gradation, 8. (a) Self explana 9. (6) CH,CH,COOH Propancic acid 11. (@) Self explana cH, 12. @) CH,=CH- CH soprene) 2Methyl-t, hydrogen atoms. , facts and formulae. 15. (a) ep hydribized cafbon atom is more electronegative an sp? hybridizall carbon which in turn ia more tive sp* hybridized carbon. More ‘Secommodates the negative aa You have either reached a page that is unavailable for viewing or reached your viewing limit for this book. aa You have either reached a page that is unavailable for viewing or reached your viewing limit for this book. aa You have either reached a page that is unavailable for viewing or reached your viewing limit for this book. 484-D 64. (cd) Like group like treatment rule. 66. (c) Long chain tule, 67. (6) CHO is taken as substituent, G8, (a) Locant rule is followed. 9. (b) Longest chain of five carbon is taken. 70. (c) Like group like treatment rule, 72. (di The parent functional greup is amide, 7. (d) Smaller groups are treated alike and also substituent 3-nitre propyl should get lowest position. 74, (d) Smaller groups are treated alike. 75, (d) Like group like treatment. 76. (a) Longest chain is taken as with both the double bonds in the chain, 80. (b) The carbon of both CN groups is counted in chain. 82. (c)'Twoethyl group are taken ax substituent on the N- atom. 83. (c COOH group is parent and longest chain of three carbon atoms is taken, s CHIOH)COOH 85. (bi Tartaric acid is | CH(OH)COOH 86. (a) It is bieyclie compound with bridges having 4, 2 ‘and 0 carbon atoms, a ey" 9—cocK, Coon 88. 0) O Acetyl salieylic acid Aspirin} oH mao WO [Pieric avid} 91. 92, 93, 94, 95. 105, 106. 107. 108, 110. 112, 4, 6. 17. 48, ne. 120. fe) (2) Mesitylene is cH (d) The OK groups afe given preference. (d) Longest chain of } carbons is taken, (d) The benzene ring is taken as substituent on the aliphatic chain, (d) Cyclohexyl grouphis substituent on the chain. (d) Benzene ring iq taken as substituent on the aliphatic chain. i (a) As aliphatic chai bas more no. of carbon atoms than the ring therefpre ring is taken as substituent. on the chain. (c} The carbon of eno. 1 (b) Two six memberdd saturated rings fused together are named decalin if TUPAC system. (a) Cumens is isoprfpyl benzene. (6) Benzene is taken} (@) In structure I alkylated and thus ture [is least alkyl substituent on aliphatic chain. wuble bended carbons are highly has maximum stability. Strue- sd and thus has lowest stability. ble for radical among these be- ifeative effect of CH, group and R. (b) Electron witha group will create el wing effect of ~ CF, and ~ NO, on deficiency at ortho and para positions in these structures is on density and thus undergoes wing effect of two nitro groups to a greater extent causing diffi- culty to electrophille attack. (b) Choice ' is cordect. It is because of electron with- drawing effect of Cl= O group attached to ring 1. aa You have either reached a page that is unavailable for viewing or reached your viewing limit for this book. aa You have either reached a page that is unavailable for viewing or reached your viewing limit for this book. aa You have either reached a page that is unavailable for viewing or reached your viewing limit for this book. a ‘Some examplesare being given below: 1 =a a i ony _ ‘The groups of higher priorities are on opposite sides, Hence it is (E) — 3-Bromopent-2-ene Geometrical isomerism in cyclic compounds Cyclic compounds also exhibit geometrical isomerism because cyclic systems alfo prevent free rotation abcut the carbon-carbon single bonds. The substituents on the came side pire cis-isomers while the substituents on opposite sides represent trans-isomers. Some examples aie: i : ¢ » G » H a H Br cis-1,2-Dibromocyclohexane —_ frans-1, 2-Dibromocyslohexane 492 con SSIVE OBJECTIVE CHEMISTHY aa You have either reached a page that is unavailable for viewing or reached your viewing limit for this book. aa You have either reached a page that is unavailable for viewing or reached your viewing limit for this book. aa You have either reached a page that is unavailable for viewing or reached your viewing limit for this book. 498 iSIVE OBJECTIVE CHEMISTRY H iG Lo cH-¢—cH,cooH on 2 (¢) CHg~CH—CH—CHy 1 CH CHa ‘ © ooo a 5 (8) The four alkyl radicals for C4Hy— are nButyl i) sec-Butyl (Git iso-Butyl dv) tert-Batyl, 6 (c) Only compounds with 4 or more carbons can exhibit chain isomerism. 7. (CH{CH,COCHs —-CHyCH;-0—CHyCH Butanone Diethyl ether CHO C4Hy90 18 woeenencuen Ww cHeHCHCH, OH i Os oe enon on, ) cH, ono cH, ro 13. @) CH,—CH- Br (©) There are the thre (@ CHyCH,CHACI CH, ynes are possible. aa You have either reached a page that is unavailable for viewing or reached your viewing limit for this book. aa You have either reached a page that is unavailable for viewing or reached your viewing limit for this book. aa You have either reached a page that is unavailable for viewing or reached your viewing limit for this book. CHEMISTRY OF I'YDROCARBONS The product actually obtained is the one which obeys Markownikoffs rule which is stated as : Electrophillic addition in unsymmetrical athenes takes place in such a way 0 that the negative part of the addendum attaches itself to the carbon which carries smaller no. of hydrogen atoms, Kharasch effect deals with anti-Markownikoff addition of HBr across unsymmetrical alkene in the presence of peroxide Itmay be noted that H’ and HCI donot follow free radical mechanism even in the presence of peroxide. ‘They add by ionic mechanism-guiding products in accordance with Markowaikof?s Rule. Addition of Hypohalous Acids (Formation of Halohydrins) Cly/H,0 CH, - CH=CH, + Hoc! _, cH,—cH-cH,c1 Propene | ou 1-Chlor-2-propano! 3. Addition of Water (Hydration) A water molecule gets added to alkenes in the presence of acids to form alcohols. Addition of water to unsymmetrical alkenes follows Markownihoff rule. During hydration of alkenes many times rearranged products are obtained. 4, Hydroboration-Oxidation In this reaction alk: is (reated with diborane followed by treatment with alkaline solution of 12 when alcohol is tormed. H0, BCHyCH = CHy + |) Jig ——>(CH,CH.CH,),B ——-—+ 3CH;CH,CH,OH + 1,803 Propene Diiveane ‘Tripropst decane — OH H22 1 Propanol In this method addi‘un of water oceurs in an anti-Markownikoffs way 5. Oxidation / i dit di CHy—CH = CH, — +510} —+ CHy—COOH +002 + 1,0 No to, Ms ctty—cHcry 525K racly 2 Hy + Op ——» 2CH,CHO Wacker Process Cub, HCL 6. Allylic Substitution 00 K (CHa= CH -CHy+Brz ———+ CHy =CH- CH ~Br + HBr Propene Allyl bromide ‘The allylic substitution can also be carried out by treating the alkene with N-bromo succinimide (NBS) METHODS OF PREPARATION OF ALKYNES 1, By Dehydrohalogenation of Alkyl halides 503 aa You have either reached a page that is unavailable for viewing or reached your viewing limit for this book. aa You have either reached a page that is unavailable for viewing or reached your viewing limit for this book. aa You have either reached a page that is unavailable for viewing or reached your viewing limit for this book. 21. B OBJECTIVE CHEMISTRY | ‘Daring electrophilic substitution in benzene, the in- termediate species involved is (6) Corbocetion (6) Carbenion (6) Fees radical (a) None of these. Ozonolysis of p-xylene give (@)Gayoxat (er Both of those In the reaction, FeCl, © + Clyp——> \—Cl+ HCI the attacking species is (&) Methyl glyoxal (d) None of those. ek wer wor (a) Feely 2,3-Dibromobutane, when heat with zine dust, yields (@)2-Butone (@)2Butyne (2) 1-Batene (@) Butane, Alkanes mainly show reactions having (@) Free radical formation © Ionic elimination (olTonic formation (@ Feeat/photochemical substitution, Olefine can be hydrogenated by (@) Zine and HO. (b) Nascent hydrogen (©) Raney Ni and Hy (@) Lithium hydride in ether. ‘To prepare a pure sample of n-hexane using sodium metal as one reactant, the other reactant will be (@) Ett! chloride and n-butyl chloride (Q) Methyl chloride and n-pentyl bromide (© Ethyl bromide and n-butyl bromi (@ n-Propyl bromide. ‘Towards electrophilic substitution the most reactive will be S (@) Nitrobenzane () Anitine (@ Aniline hydrochloride _(d)N-Actylaniline. Aqueous sulphuric acid reacts with 2-methyl-1- butene to give predominantly (@) iso Butyl hydrogen sulphate ()2-Methyl-2-butanol (€)2-Methyl-1-butanol (@) see Butyl hydrogen sulphate. 31. 34, 35. 36. a7. ‘The olefin which on eonolysis gives CHyCHgCHO and CH,CHO is (@) 1-Butene (®)2-Buteno (c) -Pentene (d)2-Pentene. Dilute aqueous KMaQj at oom temperature reacts with R- CH= CH -R}o give (a) R-CHO (R-CooH, ()RCHOK)—CHOWR] (4) CO + F20. ‘When acetylene is igh dil. HyS04 in the presence of HgSO,, thq compound formed is (@) Ethanol @) Acetone (© Aeotie acid (@) Bthanal, Select the true stathment about benzene from amongst the following! Ls (aq) Because of unsaturatjon benzene easily undergoes ‘addition reactions. () There are two types of(—C bonds in benzene molecule. () There is a cyclic delo (d) Monosubstitution of bpazene group gives three isomeric substances, Which one of the fallo hydrogenation per mol jing has the smallest heat of (@) 1-Butene ©) trane-2-Butene ()cis-2-Butene @1, 3 Butadiene. ‘The catalyst used in the manufacture of polyethene by Ziegler method is (a) Titanium tetrachlori ) Titanium tetrachle ©) Titanium dioxide @) Titanium isopropoxid and tripheny1alumintum and trimethyl aluminium gh propene in the presence of benzoy! peroxide, it fives (a) n-Propyl chloride (@) ico Propyl chloride (©) Allyl chloride @ Noreaction. Benzene reacts with n-prop}l chloride in the presence of anhydrous AICI, to give predmoninantly (a) n-Propyl benzene (0) iso-Propyl benzene (6)4-Propy)-1-chlorobensdee (@) No reaction, Which of the following resections would yield 2, 2- dibromopropane ? (@)HC=CH+ 2Br aa You have either reached a page that is unavailable for viewing or reached your viewing limit for this book. aa You have either reached a page that is unavailable for viewing or reached your viewing limit for this book. aa You have either reached a page that is unavailable for viewing or reached your viewing limit for this book. CHEMISTRY OF HYDROCARBONS 513, 31. 32. Which of the following hydrocarbons has the highest value of heat of combustion ? (0) C4Hio (6) CoHag (0 Csttag (a) Croan Which of the following compounds has the highest octane number ? (a) 2, 2, 4Trimethylpentane (6) 2,2, 3-Trimethylpentane (02,¢Dimothylhexane (4) Octane. Innis the number of carbon stoms in the potassium salt of a carboxylic acid, then the alkane formed on electrolysis ofaqueous solution of this salt would have carbon atoms equal to @n On-1 (@2n-1 @%-D. Benzene on oronolysis yields © @yoxal () Acetone (@ Bthanal (@)Methanal. ‘An organic compound which readily decolouris bromine water and forms an anhydride on heating could be coon 7 0008 @ | ) cHK cooH cooH coon coon Re Bei o 4 @ fl °. ¢ u~ coon Hooe” ‘Mesitylene can be obtained by polymerization of (a) Ettyne () Ethene (oPropene (@) Fropyne. Among the following, the compound that can be most readily sulphonated is (@) Nitrobenzene ()1,2-Dinitrobengene © Chiorobenzene (@ Toluene. Ethene is shaken with aqueous solution of Brz and NaCl Which of the following is not the possible product ? (@) Cy -Br @) CH-Br il 1 Cp - Br CHy-Cl (© CH-c1 (@) CHg-Br 1 1 CH -c1 (Cig OH. Sulphur moncchloride reacts with ..... to give mustard eas. 41. 42. 43. 48. 47. (a) Ethane (®) Ethene (o) Bthyne (@ Methane. Ethyne can be prepared in a single step from (@) Caletum carbide (®) Bxhylidene bromide (© Bihylene bromide @) All oftheve During nitration of benzene, the electrophile is (a) NOs (NOS noe (@)NOz ‘Addition of HBr to iso-butylene mainly gives (a) ito-Butyl bromide (6) n-Butyl bromide ()tert-Buty! bromide (d)see-Butyl bromide. A group which deactivates the benzene ring towards electrophilic aubstitation but directs the incoming group towards o- and p-positions (a) NH cl (NO, Cs. Starting from which of the following compounds can methane be prepared in one stop? (a) Sodium acetate (@) Aluminium carbide (©) Methyl magnesium bromide (@) Mloftheas. Starting from which of the following compounds ethaneas well as ethane can be preparedin separate ingle steps? (@) Bthyl magnesium bromide (6) Methyl bromide (© Sodium propancate (@ Bthylaleohol. 1, 1-Dibromoethane when heated with zinc dust produces (a) Bthyt bromide (®)Ethene (© Vinyl bromide (d)2-Butone. Propene can be converted into 1-propanol by (@) Hydration (6) Hydroboration-axidation (©) Reaction with alkaline KMnO, (@) Reaction with dil. NAOH solution, ‘Combustion of which of the following compounds (in the presence of excess of oxygen) doesnot result inthe change in hybrid state of the carbon atom (a) Cy (CH=CH, (©) CH -CHy (@HC= CH. aa You have either reached a page that is unavailable for viewing or reached your viewing limit for this book. aa You have either reached a page that is unavailable for viewing or reached your viewing limit for this book. aa You have either reached a page that is unavailable for viewing or reached your viewing limit for this book. CHEMISTRY OF HYDROCARBONS 517 a “$8. Since react wh anmonical AgNO soution emu ou bea terminal alkyne. Hw 1 44. (d) Boiling point of hydrocarbons increases with increase (CHa ~ CHa ~ C = CH + HO —+ CH - CH ~ C - CHg jin molecular mass. CHgCHgCHgCHs has the least mole- 1 1 cular mass. ons tty cH Additin follows MaskownikofPs rue oe +: one 45. (© Clly-CHsC-CHs CH, 29, wor cH dT - city i crgoHo + encmjono oe 0} | 29, (c)R—CH = CH-R + H,0——> R-CH-CH—R (CHjCHO + O= C- CHO+ CHQ0. 14 48, (b) Silver alkynides are explosive when present in dried OH OH form. Trin hyn 4 0) Tai hyrtratien sation recim, Here ater 2 Nee Hyon eHOR soles add to ote nd in anantownos il, HySO, 48, 6) Daring acid catalysed dehydration of 3, 3-dimethyl-2- ‘Tautomerization butanol, the intermediate carbocation rearranges through tg 12+ methyl shift CH CHs 81. (Pactual question. ee) 4 88. (d)Conjugned double bond being nos stable hasthe last eajuen-tiea, | ag Beles, heat ofhydrogenation. Teme \ 34. @)Faciual question. OH city Cily 85. (6) Addition obeys Markownikot?s rule Kharasc effect is ‘ot applicable to addltion of HCL. 2, 2emethyl ‘86. (b)n-Propyl carbocation formed as intermediate, rearran- shift ges through 1, 2-hybride shift to form more stable iso- CH; CH propyl enrbecaiion. 1a i eo - Clig-C=C~ city —— city CHC -CHly CHgCH,CH,Cl + AlClp —+ CHCHACH, + AIC, i mS + Ltlyiride —g Cy cH cHycHiciig"—— > clly-CH- city 2, 8DimethyL2-botene ee tarbedion doesnot show geometric isomerism). 87, (6) HBr adds to propyne to give 2, dibromepropane. 49. 6) Pactoatquetion 39, (d) This alkene has an unsymmetrical double bord. eee 40. (2) CoHsMeBr + D0 —+ CaHD + MgBr(OD). ssnor con ‘dine 80, (@)H-CaC-H———+ H-0-C-H a ws ‘ceteme— > bi cag city CH, -11,0 1 4 (CHg ~ CH - CH - CHg + 2NaBr 1 Oxidation Nao Ly 42, (6) CgHsCEs ———> CgHsCOOH——— = | ct @ do Soda ime 58. (a)—COOH group meta directing. CeligcOONa—— Cg GL) Ca0p = Cy a © G4kg — 28ke aa You have either reached a page that is unavailable for viewing or reached your viewing limit for this book. aa You have either reached a page that is unavailable for viewing or reached your viewing limit for this book. aa You have either reached a page that is unavailable for viewing or reached your viewing limit for this book. CHEMISTRY OF HYDROCARBONS 521 71. (@)NBS brings about allyliebrominaion a cy NBS NS o CHyCH,CH = CHy — CHg - CH - CH = CH cag Sony 4 42, 9, 4, 5-Tetramethyl-2, 4-hexadiene ‘ale. KOH e —— a-cu-cn-cn, Osonaiyais a @ oO ee cx 72 (a) Bensylc hydrogen e more eusily substituted cHO CH CH, CICHCH; GA, (Factual question. ssa 67.) truns-2-Dutene is mst stable among the given alkenes +e snd hencehas least valve of heat of hydrogenation. 68, (@) Among alkynes, ethyne is the strongest acid, Sub- inn St, un S)-coon, cohen a carbanion, formed by loss of H*, through electron releasing inductive effet of ally groups. substitution towards meta position. on cad 1 0. (a) CHgCH2C = CHy + Ha0 —— CHaCHg - C- CHa I i cH cHs 1n oboys Markownikof"s rule, aa You have either reached a page that is unavailable for viewing or reached your viewing limit for this book. aa You have either reached a page that is unavailable for viewing or reached your viewing limit for this book. aa You have either reached a page that is unavailable for viewing or reached your viewing limit for this book. ORGANIC COMPOUNDS CONTAINING HALOGENS (Alkyl Halides and Aryl Halides) 1. a. Geminal dihalides on hydrolysis give (a) Ketones (b) Aldehydes (¢) Kotones and aldehydes (d) Alechols. When ethyl iodide is treated with moist silver oxide, the product formed is (a) GH0H (e) Oily -CHy NS When chloroform is exposed to air and sunlight, it ives (6) CgH30C)H5 (a cHgCHO. (a) Mustard gas (by Lewisite {) Carbonyl ehloride (a) Carbon tetrachloride. When chloroform reacts with acetone, the product is (a) Choral ()Dichloroscetone (© Chloropicrin (a)Chleretone, Pare chloroform is obtained by treating {a) Ethanol with bleaching powder {) Acetone with bleaching powder (6) Chloral with NaOH (d) OC, with moist iron. For the reaction CoHgOH + HX —> GylgX +H,0, the order of reactivity i (a) HCI> HBr> HI (o)HI > HBr > HCL (o) HBr >HC1> HI (q)HI> HCl> HBr. ‘Treatment of ammonia with excess ethyl chloride will yield (q) Triethyl amine (6) Diethyl amine (© Ethyl amine (d) Tetracthylammonium chloride Ethyl bromide reacts with Zn-Cu couple in the presence of ethy! aleohol to form (a) Butane (Diethyl ether (©) Methane (@Bthane Identify Z in the following sequence KON ConeHC] ail HCH HCH — x yz (a) cco. (8) CHsCONHa (©) CHigcooH. (@CHsCH,COOH. When tert-butyl chloride is made to react with sodium. methoxide, the major product is 82. 35. sa] (a) Dimethyl ether (6) Ditert butyl ether (c)Tert-Butyl methylether (d)Isobutylene. Which ofthe flowing compounds ean exhibit optical (a)Chiorocyelopropane ()eis-1, 2-Dichlorocyelopropane (trans-1,2-Dichlorocyelopropane @trars-1, 3-Didhlorecydobutane. ‘Which of the following is not a primary halide ? ()Leobuty! bromide (6) Neo-Pentyl chloride (c)lsopentyi bromide —_(d) All are primary halides. The reaction given below is known as OgHI + 2Na + ICHg —+ Celis - CH + 2Nal (a) Wurtz reaction (6) Fittig reaction (Wartz-Fittig reaction (4) Uimann reaction, NBS is a specific reagent for (a) Nudeophibic substitution (6) Electrophilic substitution (6) Allylic substitution (d) Blectrophilic additi ail. NAOH H—C«C_—H+2HCI— Bc ‘The product in the above sequence (a)Ethylidene dichloride (6) Ethylene glycol (Ethanal (d) Ethyl aleohol. CHC, —— > X.TheproductX is AIC, (6) Benzyl cnloride (6) Benzal chloride (©)Bento chloride (@) Diphery! methane. TUPAC name of Gammexane is (a) Hexachlorobenzene (@yBenzenchexachloride ©1,2,3, 4,5, 6Hexachlerocyelohexane (d)None of these. Chloral on treatment with aqueous NaOH produces ? (@)Ethanal () Propanol (Chloroform (@)Chloroethanal, ‘Pyrene’ is the trade name of ..... whon used as fire extinguisher @)CO, (©) CHCIs wcck @ CHC). aa You have either reached a page that is unavailable for viewing or reached your viewing limit for this book. aa You have either reached a page that is unavailable for viewing or reached your viewing limit for this book. aa You have either reached a page that is unavailable for viewing or reached your viewing limit for this book. 532 IV OBJECTIVE CHENISTRY (© Chloral and benzene (@ Chlorobenzene and chlorine, 26. Which of the following is called Sandmeyer reaction ? Neon coanono °F, cryons Hoon exci © xr SS, 1 ny @ © son, Hs ou 60, @ on i. (O)-coon Tal 21, "The major product of reaction of aleoholic silver nitrite with ethyl bromide is (o) Ethane (b) Ethene (ONitroethane (d) Ethyl nitrite. 28. Which of the following is obtained when chloral is boiled with NaOH ? cet cnet, (ccy (a) None ofthat. 29. Which of the following is used in fire extinguishers ? (CH, (CHCl, (0 CHCl, (a) 001, C—C! bond of chlorobenzene in comparison to C—Cl ‘bond of methyl chloride is (@)longer and weaker _(b) shorter and weakor (© shorter and stronger _(d) longer and stronger. ‘31 Replacement of Cl of chlorobenzene to give phenol requires drastic conditions but chlorine of 2, 4- dinitrochlorobenzene is readily replaced because (a) —NO2 makes the ring electron rich at ortho and para positions (@) —NOg withdraws electrons at meta position (© -NOg donate electrons at meta-position (4) NOg withdraws electrons at ortho and pare positions. 92, A solution of (+}2-chloro-2-phenylethane in toluene racemises slowly in the presence of small amount of SbClg, due to the formation of (0) Carbene (a) Corbocation. (@) Carbanion (© Pree radical 36. 36. 37. 41. ic alkyl halides possible for oe we. ‘Which of the following alkyl halides can exhibit onan- tiomerism ? | (a) n-Butyl chloride (©) see-Butyl chloride @)iso-Butyl chloride (tert Butyl chioride. The best reagent foy converting alcohols to alkyl chlorides is (a) Pls @Pol, ( HCV2nC, @s0ck, pls + HNO + Ip ‘The major organic profluct in the above reaction is (a) Nitrobensene @ytedodentene (©) Benacic acid (@) p-lodonitro benzone Which of the following halides is not a primary halide? (@)iso-Pentylehtoride | (b)neo-Pentyl chioride (©/ino-Butyl chloride || (aiso-Propyl chloride. When O,HsBr is treated with limited amount of ale. NH, the major prodyet obtained is (@)Bthylamine (©) Diethylamine (o/Triethylamine (d)Tetraethylammoniuny broride ACh © + CHsCH,CH,C1—— X + HCL ‘The compound X (tho fasjor product) is (a) nPropyl benzene | (b)is0-Propyl benzene (n-Hexane (@Diphenyi. Which of the following factors does not favour Sy mechanism ? (a) Strong nucleophile | ()Polar solvent fe) Low cane. of nucleophfle («l) 8° Halide Which of the follow#eg nucleophiles would react through Sy? mechanisfn most readily? (@):08 yHL6 (o Rig 08 aa You have either reached a page that is unavailable for viewing or reached your viewing limit for this book. aa You have either reached a page that is unavailable for viewing or reached your viewing limit for this book. aa You have either reached a page that is unavailable for viewing or reached your viewing limit for this book. 542 001 SIVE OBJECTIVE CHEMISTRY (CHEMICAL PROPERTIES OF PHENOLS +a +Na0H + CHyCOCLPyridine He + (CHsCO20/11,804 = CiiyCOOH + Ogtiscoci ‘iq, NaOH 4+ NHy/ZaCh, Heat 2n/h NaOH NaOH Bra/ag) Bra/CS 73K Fuming HNOs il. HNO 298K chigcl | aniayd AICI Collen: 270-218 K, ph= 10 Gi) HO", 340K CeHsONa + Hy C6llsONa + 120 CgHsCOOCHs: ‘CeHsCOOCHs CeH,COOC SH, As Acid Acylation Benzoylation (Schatten Baumann reaction) Reimer-Tiemann aa You have either reached a page that is unavailable for viewing or reached your viewing limit for this book. aa You have either reached a page that is unavailable for viewing or reached your viewing limit for this book. aa You have either reached a page that is unavailable for viewing or reached your viewing limit for this book. 550 COMPREHENS|VE OBJECTIVE CHEMISTRY 19. a. Phenol, p-Methylphenol, m-Nitrophenol and p- Nitrophenol follows order ofincreasing acidicatrength (a) Phenol, p-Methylphenal, p-Nitro phenol, m-Nitrophenal (©) p-Methylphenol, Phenol, m-Nitrophenol, p-Nitrophenal (€)p-Methyiphenol, m-Nitrophenol, Phenol, p-Nitrophenol (@) m-Nitrophenol, p-Nitrophenol, Phenol and p-Methyl- phenol Alkene R—CH = CH, react with BgHls in the presence of Fig0z to give (@) R-CCHy ()R-CH—OF 4 | | ° ou on (e)R-CHg- CHO (@) R- Cl - CHg- OF. When3, 3-_Dimethyl-2 butanol is heated with H_SO,, the major product obtained is (3, 3Dimethyl-1-buvene (@)2, 3Dimethsl-2-buteno (612, 3.Dimethyl-1-butene (@ cis and transisomers of () Which of the following will not be soluble in sodium carbonate solution ? on nN, NO, COOH @ ‘oO o or Nop OH NO» © @ Which ofthe following reagents will convert propanoic acid to 1-propanol ? (a) KMinOg @) MnO2 (¢} Crg03 (a) Lalit ‘The order of reactivity of halogen acids for reaction with CoH;OH is 80,00 (@) HC > HBr > HI () HI> EBr> HC! (¢) HBr > HI > HCL (@) HBr > HC1> HL. Identify the final product Op Ag ank (CH, = CH; —— ®)—+ Steer (Ethanol (6) Bthanal (©) Bpoxyethane (d) Bxhylene glycol. 31. ‘on treatment with acidified (dP which reacts with Tp form triodomethane, The ‘An organic compound KyCr207 gives a com; and sodium carbonate compound ‘X could be (a) CHgOH (0) CHyCHO (© CHACHORCHS (a) CHCOCHs. ‘A compound is soluble in conc, H2S04. It does not decclourise bromine in ¢Cl, but is oxidised by chromic anhydride in aqueoud sulphuric acid within two seconds, turning orangp solution to blue green, then opaque. The original sofution contains a/an (a) secondary alechol (b) alkene (ether (@ primary alcohol. RCH,CH,O8 can be eqaverted into (@) PBrs, KCN, Hy0* (6) PBry, KCN, Ho/Pt (c) KCN, HO” (d) HCN, PBrs, H30", On distilling 2-1 benzoic acid with Zn dust, it gives (a) Phenol (@)Benaoie acid (c) Benzaldehyde ‘(d)a polymeric compound. ‘What amount of bromine will be required to convert 2.g of phonol into 2, 4, B-Tribromophenol ? (@)400 6.00 022 20.44, Which of the following compound is known as oil of winter green? (a) Phenyl benzoate (®) Phenyl salicylate (c) Phenyl acetate (d) Methy! salicylate. Which of the followin, react with water ? @) CHOI, (@)ClgC.cHO fe) CCl, (@) CICHg -CHg-CL ciganafanieaingubecan asia gra) | (ota Methane cd | (c) Ether (d) Acetone. | In the Libermann’s nifroso reaction, sequential chan- ges in the colour of phpnol occurs as (a) Brown or red —> grden—> deep blue (6) Red —» deep blue aa You have either reached a page that is unavailable for viewing or reached your viewing limit for this book. aa You have either reached a page that is unavailable for viewing or reached your viewing limit for this book. aa You have either reached a page that is unavailable for viewing or reached your viewing limit for this book. ALDEHYDES AND KETONES ‘Various reducing agents employed and the specific names associated with them are Zn-Hg/HCl (Clemensen's reduction), NEiy ~ NHly/KOH, (470 K) (Wolf Kishner reduction) ; Nalaleohol Hired P. 2n/g (CHy),C =O ———s CH,CH,CHig + 1,0 Acetone Hel Propane ° t Zoe c—CH, ———+ (ChigCHs + #20 Acetophenone NC! plipbenzene (c)Bimolecular reduction of acetone OH OH (OMe- He, Cae to %CHg)0 =0 ——————+ (CH)y-C —C{CHy) Acetone ano Pinseal 7, Condensation Reactions oO. oH u Dil. NaOH | Heat CH;—C—H + H—CH,CHO————+ CH, —-CH—CH,CHO ——_> CHs—CH = CH—CHO t nD. (croton aldehyde Acstaldehyie Aectal OH Ba(OH), i Heat 2CHyCOCH; ————> (CHg),C—-CH;COCHg ——__—+ (CH), —C = CHCOCH, Acetone “Ho Mocityl exide (i) Aldolcondensation. This is shown by aldehyde/ketones having at least one a-H atom. Here, ‘&=C of one molecules joins with earbonyl carbon of the ather (i) Crossed aldol condensation. It involves condensation between different molecules or molecules having a—H atom with aldehyde or ketones having noa—H atom. 9 oH i DaNaoH, | aie GgH,—C-H+ H- cH,cHo PEMOK, to, C—cH,cHo| —8°> o,H,cH = cH—cHO | ‘Cinnamaldehyde H Unstable ‘This type of condensation is called Claisen Schmidt condensation, (i) Condensation of ketones in acidic medium ‘Three molecules of acetone condense in the presence of éry HCl to form phorone CHs ° CH CH, oO CHy I i | Dry HCI | 1 i CHy—C = 0+ HyCHC—CHy + O= C—CHs ————> CHy—C = CH-C_cH =C_CHy =28,0 Phorone aa You have either reached a page that is unavailable for viewing or reached your viewing limit for this book. aa You have either reached a page that is unavailable for viewing or reached your viewing limit for this book. aa You have either reached a page that is unavailable for viewing or reached your viewing limit for this book. aa You have either reached a page that is unavailable for viewing or reached your viewing limit for this book. aa You have either reached a page that is unavailable for viewing or reached your viewing limit for this book. aa You have either reached a page that is unavailable for viewing or reached your viewing limit for this book. aa You have either reached a page that is unavailable for viewing or reached your viewing limit for this book. aa You have either reached a page that is unavailable for viewing or reached your viewing limit for this book. aa You have either reached a page that is unavailable for viewing or reached your viewing limit for this book. aa You have either reached a page that is unavailable for viewing or reached your viewing limit for this book. aa You have either reached a page that is unavailable for viewing or reached your viewing limit for this book. OBJECTIVE CHEMISTRY 2. By reaction of acids with diazomethane. RCOOH + CH;N,; ———+ RCOOCH,+N; ‘Acid ——-Diiazo methane Boter ‘8. By Tischenko reaction, ° AVOCA, " CHyCHO+0=CHCH, ————+ CHs—C—ocH, Bithanal Ethyl aostate CHEMICAL PROPERTIES Some important chemical properties of esters are given below : 107,804 i >———______ R-co-onl + rps NOHO |_________, r-co-on + ot Salt of aid pt + Itmay be noted that acidic hydrolysis of esters is revprsible whereas basic hydrolysis of eters is irreversible and ia alzo called shponification. NE | _________*_, r_co—wiiy + x’bit oO PCs ‘Amide I |__—_________* _, x “coc + Rc} + Pocls . ‘BOC, Aid chloride R—C—OR’ |______*_, r“co“a: + roi} + so, Ester Aid ehloride aa RCH,OH + ROH LAH, or NAEtOH Alcohols ‘+ Reduction with Na/ethancl reagent is called Ut Blane reduction. OMex R” R RMX 1 HOH | i amet, | P-C-OR') ——> Bc=0 R-C_OH eel -WOMGX Keine | cH), | moe R R ‘Aleobol |—Claisen condensation. This reaction is given Hy esters having o-H atom. ° ° i) ° 1 a tl Na0CB, " " CHs—C—00,Hy + HCH, -C—0CzH1 CHy—C—CHly~C—003Hs ~CiHOH) thy! acotoncetate (Acetoacetic exter) aa You have either reached a page that is unavailable for viewing or reached your viewing limit for this book. aa You have either reached a page that is unavailable for viewing or reached your viewing limit for this book. aa You have either reached a page that is unavailable for viewing or reached your viewing limit for this book. aa You have either reached a page that is unavailable for viewing or reached your viewing limit for this book. aa You have either reached a page that is unavailable for viewing or reached your viewing limit for this book. aa You have either reached a page that is unavailable for viewing or reached your viewing limit for this book. aa You have either reached a page that is unavailable for viewing or reached your viewing limit for this book. aa You have either reached a page that is unavailable for viewing or reached your viewing limit for this book. aa You have either reached a page that is unavailable for viewing or reached your viewing limit for this book. aa You have either reached a page that is unavailable for viewing or reached your viewing limit for this book. aa You have either reached a page that is unavailable for viewing or reached your viewing limit for this book. aa You have either reached a page that is unavailable for viewing or reached your viewing limit for this book. Tae | 7. 30. aL. 32, 37. a a1. 49, Heat (@ CHsCOONHy———> CH,CONH + #10. ‘Acetamide (©) Ammonium cyanate on heating gives urea, an organic ‘compound. (@) Hig0 reacts with acetamide to form mereury salt. (©) CHyCOCI + NaOCOCHs — (CHaCO90 + NaCl mo (202 g) 10.2 g of ethanoic anhydride is produced from acetylehloride = 0.1 mal. (@) COOH Heat I —— HCOOH + 602 coon Oxaliencd COOH Hoa Hoc —— CHjCOOH+ co, coon Malone wid CHy—COOH — Heat om - CON a 6. (CHg- COOK CH, -CO—~ Suetnie acid Suerte anhydride Ona (@ cHBr,cB,coon ——" _, (i) NaOHICAO CHBryCHBr + Nay0O3 1,1,2, 2-Tetrabrome ethane (©) Dilute acidic KMnO, oxidises formie acid but does not oxidise acetic acid {d) CHgCHCC1,COOH is strongeat acid among the given compounds dae tothe presence of 2 CHycl——+ CHYCN Ho CHsCOOH, (Ht converts R—COOH to RCH. (d) CHaFCOOH is strongest among the given compounds and thus, has lowest pk, HNO, COOH ©CiaHn0n 1 4.00) + 20. (©) Coon Rh @) CHgOH + Co —+ cHsCooH. 53, 68. (d) NaBH, reduces > C| 70. ° 1 HN - CO-NH -C-NE. Biuret oH NO, (a) Stypheraic asid is on NOz #30" (a) CH;COOH + HIN: (CHsNHy + No + C02, Methanamine (a) CHyCH- COOH. CHa CH - COC] 1 ! oH on ~ (d) CHyCOOAg + Ip CHgl+ Agl. CHsI + AgOCOCH, CH;OCOCHs. ‘Methylacetate (¢) Isoamylacetate is alfo called banana cil. o NHCONHs + NH - _- coon wom: (CHg—COOH + CO», N coon ‘Acotic acid (6) CHCOCI CH;CHO CHsCH OAH a ‘Acetal (© GHly - CH ~ COOAgs Bp — ! CAs (CHsCH - Br + 002+ Agbr. CHs ‘Doromopropane (a) CO and NaOH re hydrolysis give (©) A and Bare non-suppr impossible mirror images. reduce COOCHs, (@) In CHgCHyCH,CH(P)COOH, Cl- group which is ‘electron withdrawing ghoup js nearest to—COOH group. onesie acid sirengthpe mai (d) CHjCONH) is least Fenctive towards acyl substitution. aa You have either reached a page that is unavailable for viewing or reached your viewing limit for this book. aa You have either reached a page that is unavailable for viewing or reached your viewing limit for this book. aa You have either reached a page that is unavailable for viewing or reached your viewing limit for this book. aa You have either reached a page that is unavailable for viewing or reached your viewing limit for this book. aa You have either reached a page that is unavailable for viewing or reached your viewing limit for this book. aa You have either reached a page that is unavailable for viewing or reached your viewing limit for this book. aa You have either reached a page that is unavailable for viewing or reached your viewing limit for this book. [ ss2 COMPREHENSIVE OBJECTIVE CHEMISTRY . Which substituted amine out of the following is most 56, Aniline + COCl,—> {fhe product of the réaction are basic? (@) Phenyl anate ) Phenyl isocyanate (a) FyC—CHyNH © FyCH—CHNH, Gil Ap rhe epee, (©) YOR,CH NE (@ CHsCHANH, 87. Which catalyst is ufed fo prepare aniline from jon with ia? any, iy RG, _ Syl sertncoet by re Pic cecie 49. CcHsCOOK—__—> A> B50 D ‘a a0 x ON Pr. In the ebove sequence D is 58, ‘Trimethyl ethyl jum iodide is allowed to react (@)Aniline (©) Benzylamine with silver hydroxidp and the product formed ariaa ee heated strongly at abopt 400 K. The final products are — " “ (6) CHa = CHg and (CHgpCECN 50. Which substance on acidic hydrelysis produces formic acid and a secondary amine? (@) CHgCHZCN and OgFKNH, (a)N,NDimethylirmamide (b) Ethy\formate (ON-Bthylformamide —_(@) None of these. 51. p-toluidine is treated with an ice cold solution of sodium nitrite and hydrochloric acid. The erystalline precipitate so formed are boiled with water. The resulting product is (@)totuie acid () proto (@ phenol @ prritro toluene. 52. Which of following compound is inscluble in acid and base and also has dipole moment equal to zero @ Op 0 ox-()-non 7 NOz © Or @ a {O)-r02 58. R—-N=C +Hg0 >A +Hg,0. In the above sequence Als @)1 Amine (2° Amide (c) Alkyleyanate @) Alkyl isocyanate. 54, cHNo, =", A+B +. In this reaction, the products are (a) CHy+ N+ 00 (6) 00,4 8H +3 Ny (©) Cplg + Ny + Hy (©) 602+ H+Ne. 85, Tertiary butyl amine on treatment with KMnO, gives (oy Tertiary butyl alechol (2) Iscbutylene (@Nitroethane (@) None af these. 58, (0) CHg = CH and (CH, (@) (Citg)gC = Clg and GgHgNH + NOCI. 18 +A+B Aand Bin the above (@) Npand CoHsCl (b) NHg end CpH,0C1 (©) (CiH),N and Ny (@) CyHy+ Ny, Dipole moment of eygnides and isocyanides are re- lated as (RON = RNC @ RON> RNC RON = RNC=0 | @ECNcRNC. Which of the following amine can exhibit optical ac- tivity? action are (@) CHsN-H @ CHgCHNH: | 1 Cais Coes (© (CHg)2CNFy (@ CHs—N—Cgl, 1 1 Cols Cally ‘Which of the following product is formed when benzyl amine is oxidised wit KMnO, ? G@) p-enzequinone @) toluene (phenol (@benzcie acid. Which of the followigg does not produce salt with hydrochloric acid ? G) Ethylamine ()Acetamide Aniline is (a)a stronger base than liphenylamine () weaker base than dihenslamine (@ stronger bace then acptamide @) stronger base than bfnzylamine. Aniline (@) Phensl. aa You have either reached a page that is unavailable for viewing or reached your viewing limit for this book. aa You have either reached a page that is unavailable for viewing or reached your viewing limit for this book. aa You have either reached a page that is unavailable for viewing or reached your viewing limit for this book. [ese te 88. 68, 69, 10, nL Hy ‘KMn04 ! (©) CHly—CH—NHp 5 CHly—C= NH Keuimine Hy 0" ' —cty-c=0 Ketme Br Bry (@) NHp ——+ Br Nh. =a Br 70. mn. 2. 73. 1“. 75. 76. (a) Selfexplanatory. HNO? NH, — (a) Self explanatory. () For M-mass 72. (a) (CHig)2C = 0+ H vale of nin CyHan 4 NH is 4. (OH —> (CHy)gC = NOH wo LAH, —S (CHy,cH= NH, 7. Natom gets delocalised with the x 78. (@)Seosummary. 79, (@)Selfexplanatory. | | (@) CHyCH,CH)—H is least basic ameng these. 80. (a)See Important tens, fats and formulae. (@) See summary. QUESTION BANK (Level 11) reduction P10s () R-N=C—___5 R_N—CHy 12, (© CelisC0CI. (CsHsCONHp ——s CaHigCN 2 Amine (a) 8 nitro compounds donot have a atom. ni Ee cgutgcHtaNtie anon @R-N=2C——", HCOOH + NH: Benzyl amine 1 Amine 18, (5) CaHlgNHlg + HONG ——+ C2HOH + No +H20 LAI, (mol) | (mob (0) (CH_CH = NOH (Clg 0H - NH ‘Thos, 1 mol of CoHsNHl gives out Np = 1 mol = 22.4 Lat postorime 1 Amine STP. (b) Self explanatory. 14. (a)—NOg groupis: directing. :._m-Dinitrobenzene is (©) Cu®* + 4C)HgNHg——>(Cu(CgHgNHq)4)" produced. The of two deactivating NO2 group in Bluish ‘Tetrakis {ethylemine) medinitrobenzene farther nitration. copper (II) ion 15. (d) Alkyl cyanides on! give carboxylic acids. ‘Deep blue complex) a | LiAIE, es 16. cgughiga |CetigcN——*> CgligCHgNHlg ©) Ogi NH, +8-C-S——2s cgttg-N=C-8 ee Phenyl isothiocyanate —— > CeHsCH 0H (@) Aldehydes and ketones react only with primary amines (Bena) alcohol) snd form Schiffe bases, mw @) Direet treatment of aniline (without protection) with nitrating mixture leads to the formation ofa blac tarry | NOH sniture becauee many cnidation preducts areformed. CeligNHy Cag —— Cells ONa (a) N is surrounded by 3 bond-pairs and one lone pair. « it rid state should be sp*. Hence its, hybri a? barcoosns (©) See basicity of amines in Important terms, fats and pain formulae, aa You have either reached a page that is unavailable for viewing or reached your viewing limit for this book. aa You have either reached a page that is unavailable for viewing or reached your viewing limit for this book. aa You have either reached a page that is unavailable for viewing or reached your viewing limit for this book. aa You have either reached a page that is unavailable for viewing or reached your viewing limit for this book. aa You have either reached a page that is unavailable for viewing or reached your viewing limit for this book. aa You have either reached a page that is unavailable for viewing or reached your viewing limit for this book. aa You have either reached a page that is unavailable for viewing or reached your viewing limit for this book. aa You have either reached a page that is unavailable for viewing or reached your viewing limit for this book. aa You have either reached a page that is unavailable for viewing or reached your viewing limit for this book. eC 43. 45. 46. 41. 49. bi. Calorific values of carbohydrates (), fats UI) and proteins (Iif) are in the order : (@t>U>m (I> M51 (> I>m (@M>M>1. ‘The main structural feature of proteins is (a) Peptide Linkage () Glycoside linkage (©) Bther linkage (@) Allthe thros. Which of the following is not proteinous? (a) Woo! (o) Hair (Nails (@) DNA. ‘The amino acids are the end products of the digestion of (a) Lipies (oy Fats (©) Proteins Which of the following a-amino acids is not optically active? (@) Enzymes. (@)-Alanine () Glycine (©) Phooylalunine (2) Allare optically active ‘The name of the dipeptide H,NCHCONHCH,COOH is | CH (@) Glyeyletycine @) Glyeylalanine (© Glycine alanine (@) Alanylalycine. ‘The force of attraction between the neighbouring pep- tide chains is (o) yan der Waal's force (@) Covalent bond (c) Hydrogen bond (@ Peptide linkage B-Pleated structure of proteins is (@)Primary structure —_(b) Secondary structure (Tertiary structure (a) Quaternary structure. Denaturation of protein does nothave any effect on its (a) Secondary structure (6) Tertiary structure (Quaternary structure (q) Primary structure. ‘The proteins with a prosthetic group are known as (a) Complex proteins ©) Conjugated proteins (©) Secondary proteins _(d) Essential proteins. Peptides on hydrolysis give (a) Ammonia (oy Amines (©) Amino acide (a) Hydroxy acide ‘Which of the following is a test for proteins ? 52. 58. 58, OB/ECTIVE CHEMISTRY (6) Molinch’s test () Beiletsin tont (o Biuret test (d) Benedict's test. ~ Enzymes are | (o) Steroids |(®) Proteins (o Fatty acids (d) Nucleic acids. ‘The destruction of tHe biological nature and activity of proteins by heat of chemical agent is called () dehydration (©) denaturation (© denitrogenation (@) deammination. ‘The aon-proteinous} substances which certain en- aymes require for their activity are called () Catalysts () Inhibitors (0Coenzymes (a) Bpimers, ‘An example of zwittgr ion is (© Glycineby je (6) Ammonium acetate ( Aniline (2) Alanine. At isoelectric point, the amino acid has (0) Least viscosity (6) Maximum surface tension (2) Allof the above. the detection of (6) Proteins id) Starch, ‘ax crystalline ionic solids and have high melting pdint due to the presence of (@) “NH group @) —COOH group (@ Both NH and— $00H group (4) None ofthese, Tnanq-amino acid, @—NHy (6) COOH (oN (a) COO”. Tn an c-amino acid, fhe acidic group is (@)—CooH ©) NH, (o—NEs* td) COO”, Which of the follovfing is an example of globular protein? (@) Myosin () Catlagen (@ Keratin {d) iaemoglotin. Which of following iq en example of fibrous protein ? () Insulin (@) Haemoglobin (0 Fibroin (d) Glacogen. aa You have either reached a page that is unavailable for viewing or reached your viewing limit for this book. aa You have either reached a page that is unavailable for viewing or reached your viewing limit for this book. aa You have either reached a page that is unavailable for viewing or reached your viewing limit for this book. BIO MOLECULES AND BIOLOGICAL PROCESSES L@ 2 6) a @ 6 a0 2@ a @) 2 10. (@) Le vm ete © 15. @) 16 (a) 1m & 18. 12. 20, a moO 2 @) 2. (0) 28.) 28. am 28, 20 6) aL a2) 38. (@) 84. (0) Oe Oc a a AC ee 41. @ 42. () &. ©) 40) 46) 4) 48.) 4. ©) 50. (6) SL @ 5) BB.) 54. ©) 55.) 56, Ob) 52. ©) 38.) 58. (@) 6) GML) 4. 8. ©) 98. (a) aL GB), 70. 2) 7% & 3. @ 4. 15. (2) 7) Th. 78.) 78. (@) 80. (@) 81. (@) 82. (©) 88) 8. (e) 85.) 88. @) 82 (@) 88. (a) 29. @) 90. (a) 91. fe) 92. (¢) 98. (a) M4. 85. (©) 96. (a) 97. @) 98. (6) 98.) 100.) 10. (@) 102 @) =O) 104.) 105. 100. «@) 02%) = 108 @) 108%) BY) a, @) 118. (@ 14. @) UW. © UeE@ UL 186 Ua 12. 8 121. 122. (6) 123.) 12) 18) 128) A) 8.) 129. (@) 130. (d) 181.) 182 @) 188. @, QUESTION BANK (Level Il) L® 2@ a@ a@ a) ad 1 L@) ad 10. @) w@ me 1. @ 40 B@ 16 @) 1 @) 18, 2%) 2) LH eC) 24. (a) 28. ©) 2a. (by mh MWe wm . @) aL@ a2 ©) 38. () 84. (a) 35.) = 8 @) 0 BLY) mo) 4) 41. @ 2. (@) BO 4@ 6 @ 66 aL © a 49. (b) 50. (a) SL 52. ® 54.6) 85.) 58d) 57. @) 58. (a) 58. () 6. (@) © 2 ©) 0) 64 ©) os. 96. () a. 8. (c) - d) Be 6H Be 3.) 4 (@) TH.) 8.) 7. 78. 6) 79. (@) 80. @) + BL. @ 82. () BR fe) BA. (@) 85. (b) 86.) BZ.) 88, () 89. 90. (&) 91 © 92.) 98.) 46) 95. @) 98). aa You have either reached a page that is unavailable for viewing or reached your viewing limit for this book. aa You have either reached a page that is unavailable for viewing or reached your viewing limit for this book. aa You have either reached a page that is unavailable for viewing or reached your viewing limit for this book. ENVIRONMENTAL CHEMISTRY 681 QUESTION BANK Choose the correct answer from the four alternatives given in each of the following questions : 1 5. Ozone hole refers to (a) Black hole () decrease in thickness of ozone layer in stratosphere (c) decrease of thickness of orone in troposphere (d) increase concentration of ozone in the atmosphere. Photochemical smog is related to pollution of (a) Air (0) Water (©) Soil (@) All the above. ‘Most hazardous metal pollutant of automobile ex- haust is (@) Mercury (6) Tin (©) Cadmium (a) Lead. Which of the following play significant role in de- pletion of ozone layer ? (a) Oxides of nitrogen (b) Oxides of carbon -"” () Oxides of sulphur (d) None of the above. Which among the following is secondary pollut- ant? (@)co (@) 00, (©) PAN (a) Aerosol. DDT is (a) Biodegradable pollutant (®) Nondegradatle contaminent (©) Air pollutant (d) An antibiotic. Peeling of ozone umbrella is due to (a) CFCs (@) PAN © CO, (4) Cosl burning. Ozone layer of stratosphere requires protection from indiscriminate use of (a) Fungicides, insecticides, bactericides and medicines (b) Aerosols and high flying jets (©) Atomic explosions and industrial wastes (d) Weather balloons, 10. 11. 12. 13. 14, 15. Environmental pollution affects (a) Biotic components (6) Plants only (c) Humans only (4) Both biotic and abiotic components of environment. Water pollution is due to (a) Agricultural discharges (0) Sewage and other wastes (c) Industrial effluents (a) All the above. ‘Water is often treated with chlorine to (a) Increase oxygen content (©) Kill germs (c} Cause sedimentation (d) Remove insoluble impurities ‘The presence of which of the following in drinking water is responsible for mottling of teeth (a) Mereury () Iodine (c) Chlorine (@) Fluorine. Photochemical smog is generally formed (@) in early hours of winters (®) around mid day in summer months (c) When intensity of solar radiations is very low (d) When concentration of particulate matter is very low. ‘Which of the following reacts with haemoglobin of blood and produce toxic effect. (a) Carbon dioxide (®) Carbon monoxide (c) Oxygen (@) Carbon suboxide. Which of the following is major sink for carbon monoxide ? (a) Water (6) Soil (c) animal respiration (@) Salts dissolved in ocean water. aa You have either reached a page that is unavailable for viewing or reached your viewing limit for this book. aa You have either reached a page that is unavailable for viewing or reached your viewing limit for this book. aa You have either reached a page that is unavailable for viewing or reached your viewing limit for this book. aa You have either reached a page that is unavailable for viewing or reached your viewing limit for this book. aa You have either reached a page that is unavailable for viewing or reached your viewing limit for this book. aa You have either reached a page that is unavailable for viewing or reached your viewing limit for this book. aa You have either reached a page that is unavailable for viewing or reached your viewing limit for this book. aa You have either reached a page that is unavailable for viewing or reached your viewing limit for this book. PRACTICE PAPERS C7 (2x10 @) 1.414 x10" 4x10 (@) unpredictable. 15. Which statement about aniline is not correct ? (a) It is loss basic than etiylamine (Item be steam distilled (C)Itishighly sotuble in water (a)Itcan act as acid during its reaction with metallic sodium. 16. A mineral having formula AB crystallises in cubic close packed lattice with A atoms occupying the lattice sites. The co-ordination number of A atoms, that of B atome and fraction of tetrahedral citee occupied by B atoms are respectively (a) 8, 4, 100% ©) 2,6, 75% (3, 1,25% (d) 6,6, 50%, 17, Which of the following method cannot be used for the preparation of esters ? (a) RCOOH + ROH + OF —> (pyridine) (®)RCOCI + ROH () RCOOH+ ROH + Ht — (pyridine) (@ (RCO,,0 + ROH 18, The hybrid state of C in COg is same as that of C in (@ HCHO @) C037 (Carbon 2 of propadiene (d) Carbon 3 ofpropadiene. 19. Which of the following statements is true ? (@) AG of photochemical reactions is not always negative (6) Different carboxylic acids are produced when toluene and ethyl benzena are oxidised with KMnO (c) The colour of old Jead paintings can be restored by wash- ing them with HyS04 (2) Ontongstanding, bromine water loses its colour duc to atmospheric oxidation of bromine. 20, Which of the following statements shows difference between two functional isomers of formula C3HgO ? (a) They differ in their ease towards oxidation (®) They have different reaction with NaHSO3 (c) They roact with Tollen’s reagent in similar way (@) One of them react with HzO while the other does not. 21, Which of the following substances will be able to reduce hydrogen peroxide? (a) Og 6) Pos (©) Ag0 (a) NOs" ions. 22, Which combinations can produce phosphene gas ? (@) Cakium phosphate + HCL a7. (b) Calcium phosphide + Nig (o) Sodium hydroxide +P, (d) PCls + H30. Which of the following conditions are not suitable for spontaneity of a process ? (a) AG, p>0 thats 0;a8<0 (©) AH > 0;8S<0 High temperature (2) AH. <0; AS <0; Low temperature. p-chloroaniline and anilinium chloride can be distin- guished by (a)Sandmeyer reaction (6) NagSO4 (©) AgNOg (d)Carbylamine test. Which of the following will not undergo aldol conden- sation ? (a) Acctaldehyde (b)Propionaldchyde (©) Bensaldehyde (a) Trideutroacetaldehyde. A sclution contains « g of glucose and b g of urea in one kg of water. If total mass of solutes and total ‘morality of solution ore w andm respectively, thontho ratio 7 terms of wand m is Gq) B= 320 (4) w= 180 180m—w (60m - w) ‘Siw — 60m) © te0m a (a) (180m ~w). ‘XeF, and SF, react to form (a) Xe, SF. (W)SF6,XeF5 (0) Fy, Xe @XeFs, 8. Silvor salt of certain diabasic acid contains 71.4% of silver, the molecalar mass of the acid is, as (088.5 () 100 (a 180, I-chlorobutane on reaction with alcoholic potash gives mainly (@)But-t-ene (@Bur-z-ene © Butan-1-01 (@)Buten-201 Pick up the correct statement (@) Benzene readily decolourises KMa0, solution (@) Dipole moment of p-nitrotsluene is more than p-chloronitro toluene (In Friedel crafts alkylation AIClg acts ns Lewis base (@ Toluene and p-Xylene are isomers of each other. aa You have either reached a page that is unavailable for viewing or reached your viewing limit for this book. aa You have either reached a page that is unavailable for viewing or reached your viewing limit for this book. aa You have either reached a page that is unavailable for viewing or reached your viewing limit for this book. C4 50, 51 52. 53. ‘Wastite is (Feo (6) Fe9.940 (0 Feage 0 (d) Feo. 96 0. ‘The compounds formed when noble gases are ent- rapped in the cavitios of erystal lattices of cortain organie/inorganic compounds are called (o) Picrates (0) Aerogens (@ Nobetites (@) Cathrates ‘The IUPAC name including the stereo chemical nota- tion (for the chiral C) of BT (6) ($)2 Bromo-Z)-hex-hen-t-one (6)(R}2 Bromo(E}-hex--en-t-one (ORH-Bromo(Z)-hex-d-ene2-one (6)(8)5-Bromo(B)-hex-s-ene-2-one, Which of the following reaction would result in the liberation of Clz gas ? I. MnO" + Cr +H*—> I Cr+h— MI. C10§+1;—> IV, CY +H80,—> on om 1m, (n,1v. ‘The IUPAC name of (CH, = ©) 1,2, 3rimethylene propane ()3-Methyledenepenta 1, 4-diene (0 Hoxa- 1,2, Striene (@) 2-ethenyl-1,-butadiene. ‘A piece of sodium metal is added of four different liquids given below. In which liquic the piece will sink ‘without undergoing any chemical change (6) prepanal (6) glyeorol (o)kerosene oil (@)tiquid ammonia, ‘What are the hybrid states of carbon atom bearing electrical charge in each of the following species respectivel, tHz~ CH” and CH, =CH’. (e) sp, sp (©)sp,sp* os?.sp (sp, wp. 56. a7. 58. 59. ot. Which matching are cbrrect ? I. Glucose, TL, Anisole, phenetol -Homologues TIL. p-methoxy toluere, phenetole—Isomers (a)Tonly a Lm @uu Ng In the given reaction dequence A(Ester) —» Bry ate, BC (CyHQN). alkali a) Bisal* Amine (6) Bis an amide (c) Acan react with ex (d) C canbe 2 or 3° If By for water is 5.11 freezing point depres: ick Up the correct statement of CHyMgt to give 1 aleahol i J? Ci, then the molality and Jon of a solution of 1.56 ¢ of 2 t 100 g of CgHg are respectively (2) 01 m0.812°C (6) 0.25 m, 12800 (©) 005 m, 0.256°C (@) None of these is correct ‘Theproduct formed byphe reaction of DCI with ethene (a) CHyCH,D (cHz-cHD (6) CHyD—CH,CI (CH,C1—CHAC1 ‘Therrisein temperatuypp on mixing of 200 ml of 0.05 M MHClis 7}. If0.1 sclutions of both the solutions iq same volumes are mixed, the risein temperature is @t= ont Hesafluorocobaltate ( complex, the probable hybrid state of cobalt in it is, ena by dep! Co) sp8c? op". Which halide the following can undergo nucleophilicsubsititiod most readily ® we{O)-« (a) CH, ci CHEMISTRY : © | AIEEE © oo | © 4S Soa a Ph.No.-23260190, 51791684, 23260248 | ds) yl 30520)

You might also like